patofiziology -testy full

ne zabud popolnit pojertvovanie na tel +380960034481 za polzovanie dannoj rabotoj, please


При абсолютному голодуванні єдиним джерелом води для організму є процес окислення органічних сполук. Яка з наведених речовин в цих умовах є основним джерелом
=Жири
Білки
Вуглеводи
Глікопротеїни
Ліпопротеїни

Після перенесеного сепсису у хворої 27 років з’явився бронзовий колір шкіри, характерний для аддісонової хвороби. В підвищенні секреції гормону полягає механізм гіперпігментації?
=Меланоцитстимулюючого
Соматотропного
Гонадотропного
B-ліпотропного
Тиреотропного

Хворому з ревматоїдним артритом тривалий час вводили гідрокортизон. У нього з’явилися гіперглікемія, поліурія, глюкозурія, спрага. Ці ускладнення лікування є наслідком
=Гліконеогенезу
Глікогенолізу
Глікогенезу
Гліколізу
Ліполізу

У жінки 46 років після операції на щитовидній залозі в невдовзі з’явилися фібрилярні посмикування м’язів рук, ніг, обличчя. Ці порушення можна усунути шляхом введення
=Паратгормону
Трийодтироніну
Тиреотропіну
Тироксину
Тіреотропного гормону

У пацієнта, який півтора місяця тому переніс інфаркт міокарда, діагностовано синдром Дреслера з характерною тріадою: перикардит, плеврит, пневмонія. Який головний механізм цього ускладнення?
=Сенсибілізація організму антигенами міокарда
Зниження резистентності до інфекційних агентів
Активація сапрофітної мікрофлори
Інтоксикація організму продуктами некрозу
Викидання у кров міокардіальних ферментів

У хворого з гіпертонічною хворобою виявлено значне збільшення маси міокарда лівого шлуночка. Це сталося
=Збільшення об’єму кардіоміоцитів
Збільшення кількості кардіоміоцитів
Розростання сполучної тканини
Затримки води в міокарді
Жирової інфільтрації міокарда

У дитини з геморагічним синдромом діагностована гемофілія В. Вона зумовлена дефіцитом фактора
=ІХ (Крістмаса)
ІІ (протромбіну)
VІІІ (антигемофільного глобуліну)
ХІ (протромбопластину)
ХІІ (Хагемана)

У хворого після оперативного удалення кисти підшлункової залози виник геморагічний синдром з вираженим порушенням зсідання крові. Розвиток цього ускладнення пояснюється
=Активацією фібринолітичної системи
Недостатнім утворенням фібрину
Зменшенням кількості тромбоцитів
Активацією протизгортальної системи
Активацією фактору Крисмаса

У хворого після травматичної перерізки сідничного нерва виникли трофічні зміни шкіри. Основним механізмом їх появи є
=Припинення аксоплазматичного току
Втрата нервом збудливості
Фагоцитоз нервових закінчень
Руйнування мієлінової оболонки
Пошкодження перехватів Ранв’є

У лікарню доставили хворого на цукровий діабет у стані непритомності. Дихання типу Кусмауля, артеріальний тиск 80/50 мм рт.ст., з запахом ацетону з рота. Накопиченням в організмі яких речовин можна пояснити виникнення даних розладів?
=Кетонових тіл
Модифікованих ліпопротеїдів
Молочної кислоти
Вугільної кислоти
Складних вуглеводів

У хворої з феохромоцитомою після психічного навантаження виникає тахікардія, підвищується артеріальний тиск, з’являється різкий біль у надчеревній ділянці. Ці приступи
=Масивним викиданням катехоламінів наднирниками
Звільненням норадреналіну симпатичними нервами
Активацією вегетативних ядер гіпоталамуса
Збільшенням секреції тиреоїдних гормонів
Підвищеним синтезом адренокортикотропного гормону

Електрокардіографічне дослідження пацієнта з гіпертонічною хворобою показало такі результати: ритм синусовий, правильний, частота серцевих скорочень 92/хв, тривалість РQ – 0,2 с, QRS – не змінений. У хворого є порушення
=Атоматизму
Провідності
Збудливості
Рефрактерності
Скоротливості

У пацієнтки, яка перехворіла на грип, шляхом електрокардіографічного дослідження виявлено таке: частота серцевих скорочень 140/хв, ритм синусовий, коливання величини R-R не перевищують 0,15 с, тривалість PQ – 0,2 c, QRS – не змінений. Ці показники свідчать про розвиток
=Синусової тахікардії
Синусової тахіаритмії
Фібриляція шлуночків
Пароксизмальної тахікардії
Мерехтіння шлуночків

У хворого виявлена аденома, що походить з клітин клубочкової зони кори наднирників. В результаті цього розвинувся первинний гіперальдостеронізм або хвороба Кона. На обмін якого іону впливає цей гормон?
=натрію
хлору
магнію
кальцію
заліза

У хворого має місце хронічна недостатність кіркової речовини надниркових залоз ( Аддісонова або бронзова хвороба). Недостатність якого гормону має місце при цьому патологічному процесі?
=Альдостерону
Інсуліну
Адреналіну
Тироксину
Вазопресину

Через 1 – 2 доби після видалення у собаки прищитовидних залоз спостерігались: млявість, спрага, різке підвищення нервово-м’язової збудливості з розвитком тетанії. Яке порушення обміну електролітів має місце при цьому?
=Гіпокальціемія
Гіперкальціемія
Гіпомагніемія
Гіпермагніемія
Гіпонатріемія

Хвора, 24 р., скаржиться на сухість в роті, зниження маси тіла, незважаючи на підвищений апетит, підвищення сечовиділення. Які дослідження для постановки діагнозу необхідно призначити в першу чергу?
=Визначення рівня цукру в добовій кількості сечі
Аналіз сечі по Зимницькому
Загальний аналіз сечі
Дослідження білкових фракцій сироватки крові
Коагулограма

Еритроцити – 3,0(1012/л; Hb – 90г/л; ретикулоцити – 0,5\%. В мазку: пойкілоцити, гіпохромні еритроцити. Залізо сироватки крові – 80мкмоль/л. Для якої патології це характерно?
=Залізорефрактерна анемія.
Хвороба Мінковського-Шоффара.
Залізодефіцитна анемія.
В12-дефіцитна анемія.
Серпоподібноклітинна анемія.

Загальна кількість лейкоцитів-90(109/л. В лейкоцитарній формулі: е-0\%, б-0\%, ю-0\%, п-2\%, с-20\%, лімфобласти -1%, пролімфоцити-2\%, лімфоцити-70\%, м-5\%,клітини Боткіна-Гумпрехта. У хворого збільшені шийні, підщелепні лімфатичні вузли. Для якої патології характерна така картина
=Хронічний лімфолейкоз.
Гострий лімфолейкоз.
Лімфогранульоматоз.
Інфекційний мононуклеоз.
Хронічний мієлолейкоз.

Хворий скаржиться на часті нудоти, які нерідко завершуються блювотою. Порушення якої із функцій шлунку найвірогідніше запідозрити у даного хворого?
=Евакуаторної.
Єкскреторної
Всмоктувальної
Інкреторної.
Секреторної.

У хворого діагностовано асцит. На жівоті виступають крупні судини синюшного відтінку. Ознакою якої гіпертензії є дане
=Портальної
Малого кола кровообігу.
Есенціальної
Церебро-ішемічної
Ниркової

Хворий, 35 років, скаржитьсяна біль в ділянці поперека, набряки під очима, підвищену втомлюваність. В сечі хворого виявлено білок – 0,99\%о. Артеріальний тиск – 160/110мм.рт.ст. Яку патологію з нижченаведених найвірогідніше можна запідозрити у даного хворого?
=Нефрит
Пієліт.
Цистит.
Уретрит
Нефротичний синдром.

У хворого М., 55р., виявили гіперплазію кори наднирників. АТ – 190/90 мм.рт.ст.; в крові - вміст глюкози – 20 ммоль/л, збільшення кортікотропину; в сечі - глюкозурія. Спостерігається ожиріння, гірсутизм. Для якої патології характерні виявлені
=Хвороби Іценка-Кушинга.
Хвороби Аддісона.
Синдрому Іценка-Кушинга.
Адипозогенітальної дистрофії.
Хвороби Барракера- Сіммондса.

Хвора, 28 р., скаржиться на в’ялість, швидку розумову та фізичну втомлюваність, диспептичні порушення. При обстеженні виявлено: позитивні туберкулінові проби, гіпоглікемія, АТ – 90/60 мм.рт.ст.,гіпонатріємію, пігментацію шкіри. При якій патології наднирників спостерігаються подібні явища?
=Хвороба Аддісона.
Синдром Іценка-Кушинга.
Гостра недостатність кори наднирників.
Гіпофункція мозкового шару наднирників.
Синдром Конна.

У фізично здорових молодих вояків після важкого фізичного навантаження при одноденному пішому переході на 50 км в сечі виявлено білок, рівень якого в середньому не перевищував 1 г/л. Який різновид протеїнурії, в першу чергу, мав місце ?
=Маршова протеїнурія
Дегідраційна протеїнурія
Аліментарна протеїнурія
Органічна протеїнурія
Несправжня протеїнурія

У хворого на ішемічну хворобу серця на грунті атеросклерозу коронарних артерій після коронарографії розвинувся тромбоз передньої міжшлуночкової вінцевої артерії. Який механізм в розвитку цього ускладнення є найбільш суттєвим?
=Пошкодження ендотелію судинної стінки
Сповільнення плину крові
Підвищення концентрації коагулянтів крові
Зменшення вмісту антикоагулянтів крові
Зниження активності фібринолітичної системи

У больного, длительно курящего табак, развился рак легкого. Какие из перечисленных канцерогенных веществ содержатся в табачном дыме и относятся к ПАУ?
=Бензпирен
Диметиламиноазобензол
Бета – нафтиламин
Диэтилнитрозамин
Ортоаминоазотолуол

В поликлинику обратилась женщина с жалобами на повышенную раздражительность, потливость, слабость, похудание, дрожание конечностей, сердцебиение, отмечается пучеглазие, субфебрильная температура. Какие метаболические нарушения в организме отражают патогенез
=Увеличение основного обмена
Увеличение синтеза АТФ
Снижение энергетического обмена
Ослабление активирования фосфолипазы
Снижение распада холестерина

У хворих на поворотний тиф виникає лихоманка, яка характеризується кількаденними періодами високої гарячки, що чергується з періодами нормальної температури. Така температурна крива називається:
=Febris recurrns
Febris hectica
Febris intermittens
Febris continua
Febris atypica

У чоловіка 49 років, який 12 років тому хворів ревматичним міокардитом та ендокардитом, є недостатність мітрального клапану. Дослідження показали, що запального процесу зараз не має, хвилинний об’єм кровообігу достатній. Якому поняттю загальної нозології відповідає дана умова?
=Патологічний стан.
Патологічна реакція.
Патологічний процес.
Типовий патологічний процес.
Компенсаторна реакція.

У хворого 54 років, який на виробництві багато контактував зі свинцем, виявлена гіпохромна анемія. Лікування препаратами заліза протягом місяця ефекту не дало. Встановлено підвищений вміст заліза в сиворотці крові. Чим обумовлений розвиток анемії?
=Порушенням синтезу порфіринів.
Дефіцитом вітаміна В12.
Дефіцитом фолієвої кислоти.
Гіпоплазією червоного кісткового мозку.
Дефіцитом белку

У хворого 20 років періодично з’являється жовтушність склер та шкіри, слабкість. Діагностована хвороба Мінковського-Шоффара. Що найбільш характено для картини крові при цьому захворювані?
=Мікросфероцитоз.
Анулоцитоз
Агранулоцитоз.
Макроцитоз.
Тромбоцитоз.

Хвора 45 років скаржиться на задишку при невеликому фізичному навантажені, набряки на ногах, в анамнезі часті ангини, хворіє на протязі двох років. Діагнастовано недостатність кровообігу. Який гемодинамічний показник декомпенсації серця спостерігається в даному випадку?
=Зменшення хвилинного об’єму серця.
Зменшення об’єму циркулюючої крові.
Зменшення венозного тиску.
Підвищення артеріального тиску.
Тахікардія.

У хворого на ішемічну хворобу серця раптово з’явився тяжкий приступ стенокардії. Про нього: обличча бліде, шкіра волога, холодна, АД 70/50 мм рт. ст. екстрасистолія. Діагнастован інфартк міакарду та кардіогенний шок. Назвіть первинний ланцюг патогенезу.
=Зменшення хвилинного об’єму крові.
Екстрасистолія.
Токсемія.
Больовий синдром.
Гіпотензія.

Хвора 44 років скаржиться на загальну слабкість, збільшення маси тіла, ріст волосся на обличчі, зупинку менструацій, АТ 165/100 мм.рт.ст. Що допоможе диференціювати хворобу Іценко–Кушинга від синдрому Іценко–Кушинга?
=Рівень кортикотропіну в плазмі крові.
Рівень кортизолу в плазмі.
Вміст 17 –оксікетостероїдів в сечі.
Ренгенографія черепу.
Кількість еозинофілів в крові.

У чоловіка 25 років виявлена недостатність митрального клапану без порушення кровообігу. Який негайний механізм забеспечуе серцеву компенсацію?
=Гетерометричний
Гомеометричний
Міогенна дилятація
Зменшення маси серця.
Посилення синтезу катехоламінів

У дівчини 15 років виявлен стеноз аорти, проте розладів кровообігу у неї не спостерігалось. Який негайний механізм забезпечує серцеву компенсацію?
=Гомеометричний
Підвищення АТ
Гетерометричний
Зменшення маси серця.
Міогенна дилятація

У дитини 5 років на ЕКГ спостерігалось порушення ритму серцевої діяльності.При затримці диханя ритм серцевої діяльності ставав правильним. Який вид порушень було
=Дихальна аритмія.
Миготлива аритмія.
Синусна екстросистолія.
Предсердна екстросистолія.
Поперечна блокада серця.

У чоловіка 50 років раптово виникло сильне серцебиття, біль у серці, різка слабкість, підвищення АТ, пульс неправильний з дефіцітом. На ЕКГ виявлено відсутність зубця Р і різні інтервали R-R. Яке порушення серцевого ритму у хворого?
=Миготлива аритмія.
Дихальна аритмія.
Пароксизмальна тахікардія .
Поперечна блокада серця.
Синусова екстрасистолія.

Чоловік 57 років скаржиться на біль в серці, який виник після тривалих негативних емоцій. Лікар швидкої допомоги встановив ішемічну хворобу серця, що проявилась стенокардієюя. Який механізм ішемії найбільш вирогідний?
=Ангіоспастичний

Облітераційний.
Компресійний.
Обтураційний.

У хворого з алкогольним цирозом печінки скарги на загальну слабкість, задишку. Встановлено зниження артеріального тиску, асцит, розширення поверхневих вен передньої стінки живота, спленомегалію. Яке порушення гемодинаміки спостерігається у хворого?
=Синдром портальної гіпертензії.
Недостатність лівого шлуночка серця.
Недостатність правого шлуночка серця.
Колапс.
Тотальна серцева недостатність

У больного вследствие хронического заболевания органов дыхания на фоне одышки, тахикардии и цианоза при исследовании газового состава крови выявлено развитие гипоксемии и гиперкапнии. Какое нарушение внешнего дыхания наблюдается у больного?
=Гиповентиляция
Гипервентиляция
Гиперперфузия
Гипоперфузия
Гипердиффузия

У младенца отмечается рвота и понос, общая дистрофия, гепато- и спленомегалия. При прекращении вскармливания молоком – симптомы уменьшаются. Какой основной наследственный дефект обмена веществ лежит в основе этой
=галактозы
триптофана
тирозина
фенилаланина
глюкозо-6-фосфатдегидрогеназы

Хвора 9 р. Протягом першого року життя знаходилась на природному вигодовуванні. В кінці першого року перенесла пневмонію у тяжкій формі, із затяжним перебігом. Ходити почала пізно. З’явились телеангіоектазії на шкірі і кон’юктивах. В крові – відсутність Ig A, знижений рівень Т-лімфоцитів. Яке з імунодефіцитних захворювань у дівчинки?
=Синдром Луї-Барр
Синдром Віскота-Олдрича
Синдром Чедіака-Хігасі
Імунодефіцит швейцарського типу
Синдром Ді Джорджі

У хворого виявлено порушення прохідності дихальних шляхів на рівні дрібних і середніх бронхів. Які зміни кислотно-основної рівноваги можуть розвинутись у пацієнта?
=Респіраторний ацидоз
Респіраторний алкалоз
Метаболічний ацидоз
Метаболічний алкалоз
КОР не зміниться

У дитини, хворої на дифтерію, розвинувся набряк гортані. Який розлад дихання спостерігається у даному випадку?
=Диспное (задишка)
Гаспінг-дихання
Апнейстичне дихання
Дихання Куссмауля
Дихання Біота

У хворої після пологів через декілька місяців почалося випадіння волосся, втрата ваги, млявість, випадіння зубів. АТ , температура тіла, рівень глюкози крові - знижені . При обстеженні рівень соматотропного і кортикотропного гормонів в крові знижений. Яке порушення функції гіпофізу у хворої?
=Пангіпопітуїтризм
Гіпофізарний нанізм
Акромегалія
Хвороба Іценко-Кушінга
Нецукровий діабет

Хвора В., 45 років, поступила в лікарню з повною втратою свідомості, арефлексією, випадінням зіничного і рогівкового рефлексів, періодичним диханням типу Кусмауля. АТ, температура тіла – знижені. Аналіз крові: заг.білірубін – 16,0 мкмоль/л, сечовина – 3,6 ммоль/л, креатинін – 10,8 мкмоль/л, глюкоза – 22 ммоль,л. Для якого виду коми характерна дана картина?
=Гіперглікемічна
Гіпоглікемічна
Печінкова
Ниркова
При недостатності наднирників

У больной, страдающей расстройствами менструального цикла, сопровождающимися продолжительными кровотечениями, выявлена гипохромия, снижение числа ретикулоцитов, микроцитоз, гипосидеремия. К какой группе по патогенезу относится описанная анемия?
=Железодефицитной
В12- фолиеводефицитной
Гипопластической
Гемолитической
Метапластической

У больного с инфекционным заболеванием температура тела через сутки повышалась до 39,5-40,5 0С и держалась на этой высоте около часа, а затем возвращалась к исходному уровню. Какой тип лихорадочной кривой описан в данном случае?
=Перемежающаяся
Постоянная
Послабляющая
Изнуряющая
Атипичная

У больного с инфекционным заболеванием температура тела через сутки повысилась до 39,5- 40,5 0С и держалась на этой высоте около часа, а затем возвращалась к исходному уровню. При каком заболевании встречается описанный тип лихорадочной кривой?
=Малярии
Туберкулезе
Гриппе
Перитоните
Бруцеллезе

После введения пирогена у больного А. повысилась температура, кожные покровы стали бледными, холодными на ощупь, появился озноб, увеличилось потребление кислорода. Как изменяются процессы терморегуляции в описанном периоде
=Снижается теплоотдача
Увеличивается теплопродукция
Теплоотдача равна теплопродукции
Снижается теплопродукция
Увеличивается теплопродукция

В 1910 г. Раус в эксперименте получил саркому кур путем введения им бесклеточного фильтрата, полученного из саркомы курицы. Какой метод экспериментального моделирования использовал автор?
=Индуцирования
Эксплантации
Изотрансплантации
Гомотрансплантации
Гетеротрансплантации

При микроскопии мазка экссудата, полученного от крысы с асептическим перитонитом, с добавлением в экссудат птичьих эритроцитов, обнаружены макрофаги, окруженные чужеродными эритроцитами. Какой стадии фагоцитоза соответствует описанная картина?
=Прилипания
Незавершенного фагоцитоза
Приближения
Погружения
Внутриклеточного переваривания

После снятия кровоостанавливающего жгута через 3 часа после его наложенияпоявился у больного появился резкий отек ткани бедра, частый пульс, холодный пот, резкая гипотония. Какой патологический процесс развился у больного?
=Токсемический шок
Анафилактический шок
Коллапс
Кардиогенный шок
Геморрагический шок

Хворий 23 років поступив у лікарню із черепно-мозковою травмою у важкому стані. Дихання характеризується судомним тривалим вдохом який переривається коротким видихом. Для якого типу дихання це характерно?
=Апнейстичного
Гаспінг-дихання
Куссмауля
Чейн - Стокса
Біота

На 10 сутки лечебного голодания больной почувствовал возбуждение, появилось глубокое, шумное дыхание, артериальное давление снизилось до 90/60 мм рт.ст., уменьшилось выделение мочи, моча с запахом ацетона. Чем обусловлены описанные явления?
=Кетозом
Негазовым алкалозом
Гипергликемией
Гипогликемией
Газовым ацидозом

Через 8 дней после облучения у ликвидатора ЧАЭС развились язвенно-некротические изменения в полости рта. В анализе крови: Эр-3,2.1012/л, ретикулоцитов 0,01\%, Hb-60г/л, лейкоцитов 2,3.109/л, тромбоцитов 50 тыс/л Для какого периода лучевой болезни характерны описанные изменения?
=Периода развернутых клинических признаков
Периода первичных реакций
Скрытого периода
Периода мнимого благополучия
Исхода болезни

Больной жалуется на тошноту, отрыжку, боль в єпигастральной области, запор, исхудание. При обследовании базальная секреция НСl - 6 ммоль/г, максимальная-60 ммоль/г. Каково состояние кислотообразующей функции у данного больного?
=Гиперацидное
Гипоацидное
Анацидное

Нормацидное

У больного, перенесшего травму и последующий геморрагический бурсит левого коленного сустава, при осмотре через 3 месяца отмечается ограничение объема движений в данном суставе вследствие образования рубца. Какой компонент воспаления является основой развития этого
=Пролиферация
Альтерация первичная
Альтерация вторичная
Экссудация
Нарушение микроциркуляции

Вследствие действия на организм электрического тока городской электросети в течение 0,1 сек в направлении “правая рука-голова” у пострадавшего наблюдалась остановка дыхания. Укажите наиболее вероятный механизм этого осложнения.
=Тотальный паралич дыхательного центра
Рефлекторная остановка дыхания [болевой шок]
Паралич дыхательных мышц
Эмоциональный стресс
Паралич центров вдоха

После длительной работы в горячем цеху у рабочего отмечено повышение температуры тела до 38,50С, тахикардия, нарушение ритма сердечных сокращений ,АД-160/85 мм рт.ст. Преимущественная потеря какого электролита обусловливает нарушения работы сердца?
=Калия
Кальция
Магния
Натрия
Фосфора

У мужчины после гипертонического криза отмечается отсутствие произвольных движений в правых руке и ноге, тонус мышц в этих конечностях повышен. Какой вид расстройства двигательной функции нервной системы
=Центральный паралич
Периферический паралич
Периферический парез
Рефлекторный парез
Центральный парез

Эпидемиологическое исследование распространения опухолей выявило высокую корреляцию развития опухолей легких с табакокурением. С действием какого химического канцерогена наиболее вероятно возникновение данного вида патологии?
=3,4-бензпирена
Ортоаминоазотолуола
Афлатоксина
Метилхолантрена.
Диэтилнитрозамина

В эксперименте у белой крысы моделировался отек легкого путем введения адреналина. Какой патогенетический механизм является ведущим в данном случае ?
=Гидродинамический
Токсический
Мембраногенный
Лимфогенный
Коллоидно-осмотический

У больного обнаружено ожирение, гирсутизм, *лунообразное* лицо, рубцы багрового цвета на коже бедер. АД 180/110 мм рт.ст., глюкоза крови-17,2 ммоль/л . При каком изменении продукции гормонов надпочечников возможна такая картина?
=Гиперпродукции глюкокортикоидов
Гипопродукции глюкокортикоидов
Гиперпродукции минералокортикоидов
Гипопродукции минералокортикоидов
Гипопродукции адреналина

Зміни на ЕКГ проявлялись спочатку подовженням інтервалу P-Q, потім випадінням поодиноких комплексів QRS, пізніше – збільшенням кількості випадаючих шлуночкових комплексів, і, нарешті, передсердя скорочувались з частотою 70 уд./ хв., а шлуночки – 35 уд./ хв. Описані зміни характерні для:
=Атріовентрикулярної блокади
Внутрішньопередсердної блокади
Внутрішньошлуночкової блокади
Екстрасістолії
Брадікардії

Больной В. 67 лет, страдает атеросклерозом сосудов сердца и головного мозга. При обследовании обнаружена гиперлипидемия. Какой класс липопротеидов плазмы крови имеет наибольшее значение в патогенезе атеросклероза?
=Липопротеиды низкой плотности
Хиломикроны
Альфа-липопротеиды
Липопротеиды высокой плотности


Больной М., страдает хронической сердечной недостаточностью. Какой из перечисленных показателей гемодинамики является ведущим признаком развития
=Уменьшение ударного объёма
Развитие тахикардии
Тоногенная дилятация
Увеличение периферического сопротивления сосудов
Повышение центрального венозного давления

При анализе ЭКГ у больного с инфарктом миокарда было обращено внимание на отсутствие зубца Р, присутствие волнообразной изоэлектрической линии со множеством мелких зубцов, частое и беспорядочное расположение комплексов QRS. Какому нарушению ритма соответствует описанная
=Мерцанию предсердий
Мерцанию желудочков
Идиовентрикулярному ритму
Пароксизмальной тахикардии
Желудочковой экстрасистолии

У больного на 2-е сутки после развития инфаркта миокарда произошло резкое падение систолического АД до 60 мм.рт.ст. с тахикардией 140 уд/мин, одышкой, потерей сознания. Какой механизм является ведущим в патогенезе развившегося шока?
=Уменьшение ударного объема крови
Интоксикация продуктами некротического распада
Снижение объёма циркулирующей крови
Пароксизмальная тахикардия
Анафилактической реакции на миокардиальные белки

Во время ЭКГ - исследования больной Р. выявлено периодическое появление желудочковой экстрасистолы. При этом установлено, что перед экстрасистолой отсутствует зубец Р. Какая наиболее вероятная причина его исчезновения?
=Невозможность ретроградного проведения через AV-узел
Блокада проведения импульса по предсердиям
Возникновение рефрактерного периода в желудочках
Блокада импульса в синусовом узле
Возникновение рефрактерного периода в предсердиях

У больного Ш. с гипертонической болезнью при ультразвуковом исследовании выявлена дилятация полостей сердца. Какой из нижеперечисленных признаков свидетельствуют о развитии тоногенной дилятации?
=Расширение полостей сердца с увеличением ударного объема.
Расширение полостей без изменения ударного объема.
Расширение полостей с уменьшением ударного объема.
Равномерное расширение границ сердца
Неравномерное расширение границ сердца

У больного В., 38 лет при исследовании ЭКГ обнаружили нерегулярные атриовентрикулярные экстрасистолы. Нарушение каких свойств миокарда составляет основу патогенеза экстрасистолии?
=Возбудимости
Автоматизма
Проводимости
Сократимости


У 48 летнего пациента после сильной психоэмоциональной нагрузки внезапно появилась острая боль в области сердца с иррадиацией в левую руку. Нитроглицерин снял приступ боли через 10 минут. Какой патогенетический механизм является ведущим в развитии этого процесса?
=Спазм коронарных сосудов
Расширение периферических сосудов
Закупорка коронарных сосудов
Сдавление коронарных сосудов
Повышение потребностей миокарда в кислороде

Больной К., 35 лет, предъявляет жалобы на постоянную жажду, сниженный аппетит. Количество выпиваемой жидкости за стуки 9 л. Суточный диурез увеличен, моча обесцвечена, относительная плотность - 1005. Наиболее вероятной причиной развития данной патологии у больного является повреждение:
=гипоталамических ядер
эпителия почечных канальцев
аденогипофиза
эпифиза
базальной мембраны капилляров клубочков

У больного В, 46 лет, выявлено непропорциональное увеличение кистей рук, стоп ног, носа, ушей, надбровных дуг и скуловых костей. В крови - гипергликемия, нарушение теста толерантности к глюкозе. Причиной развития данной патологии скорее всего является:
=гиперсекреция соматотропного гормона
гиперсекреция всех гормонов аденогипофиза
гипосекреция инсулина
гипосекреция вазопрессина
гиперсекреция глюкокортикоидов

Больному И., 45 лет с диагнозом “цирроз печени, асцит” произведено извлечения 5 литров жидкости, что вызвало развитие обморочного состояния, как проявления недостаточного кровоснабжения головного мозга. Какое нарушение кровообращения в брюшной полости имеет место в данном случае?
=Артериальная гиперемия
Ишемия
Венозная гиперемия
Тромбоз
Эмболия

У больного Н., 50 лет, с жалобами на потерю массы тела, слабость в крови обнаружена гипогликемия и гиперинсулинемия. При дополнительном обследовании выявлена опухоль островков Лангерганса. Усиление синтеза инсулина в данном случае расценивается как следствие
=функционального
морфологического
биохимического
физико-химического
иммунологического

Больная Л., 48 лет, с сахарным диабетом поступила в больницу в тяжелом прекоматозном состоянии. При исследовании КОС обнаружен метаболический ацидоз. Какой первичный механизм обусловил выявленные изменения КОС?
=Образование недоокисленных продуктов
Нарушение использования О2 в клетках
Нарушение буферных систем крови
Выведение щелочных компонентов с мочой
Снижение выведения СО2

Больной Ю., 40 лет, находится на лечении с диагнозом туберкулез легких. При обследовании отмечается акроцианоз, одышка, расширение границ сердца, повышение АД, увеличение числа эритроцитов, сгущение крови, нейтрофильный лейкоцитоз. Какие из ниже перечисленных симптомов хронической гипоксии относятся к долговременным компенсаторным механизмам организма?
=Дилятация полостей и гипертрофия миокарда
Сгущение крови
Лейкоцитоз
Повышение АД
Увеличение частоты дыхания

Больной Д., 29 лет, поступил в клинику с отравлением угарным газом. Объективно признаки тяжелой гипоксии: выраженная одышка, цианоз, тахикардия. Образование какого соединения имеет место при отравлении угарным газом?
=карбоксигемоглобина
метгемоглобина
карбгемоглобина
сульфгемоглобина
оксигемоглобина

В реанимационное отделение доставили водителя, который попал в автомобильную аварию. Он не сразу реагирует на вопросы, равнодушен ко всему, бледный, дыхание поверхностное, редкое, АД равно 75/50 мм.рт.ст. Назовите главное звено патогенеза указанной патологии.
=Торможение центральной нервной системы
Возбуждение ЦНС
Кровопотеря
Токсемия
Перераспределение крови

Больного доставили в клинику в коматозном состоянии. В анамнезе сахарный диабет 2 типа на протяжении 5 лет. Объективно: дыхание шумное, глубокое, в выдыхаемом воздухе слышен запах ацетона. Содержание глюкозы в крови 15,2 ммоль/л, кетоновых тел - 100 мкмоль/л. Для какого осложнения данного заболевания характерны такие
=Кетоацидотическая кома
Печеночная кома
Гипергликемическая кома
Гипогликемическая кома
Гиперосмолярная кома

При исследовании сыворотки крови больного обнаружено повышение уровня аланиламинотрансферазы (АЛТ) и аспатртатаминотрансферазы (АСТ). Какие изменения в организме на клеточном уровне могут привести к подобной
=Разрушение клеток
Нарушение функции энергообеспечения клеток
Нарушения ферментных систем клеток
Повреждение генетического аппарата клеток
Нарушения межклеточных взаимоотношений

С целью немедикаментозного лечения гипертонической болезни был проведен курс дозированных гипоксических тренировок. Какие наиболее значимые механизмы адаптации клеток включаются при данном лечении?
=Гипертрофия клеток
Повышение проницаемости мембран клеток
Коррекция ионного транспорта
Мобилизация пластических резервов
Снижение функциональной активности клеток

У пациента с хронической сердечной недостаточностью выявлено увеличение вязкости крови, при капилляроскопии обнаружено повреждение стенок сосудов микроциркуляторного русла. Какое из перечисленных нарушений возможно в данном
=Сладж-феномен
Тромбоз
Эмболия
Артериальная гиперемия
Венозная гиперемия

Хворому з цирозом печінки разом з лікарськими препаратами довенно ввели 500,0 мл 5\% розчину глюкози. Яке порушення водно-електролітного балансу найбільш ймовірно може виникнути у хворого?
=Гіпоосмолярна гіпергідратація
Гіперосмолярна гіпергідратація
Ізоосмолярна гіпергідратація
Гіпоосмолярна дегідратація
Гіперосмолярна дегідратація

Тварині, сенсибілізованій туберкуліном, внутрішньоочеревенно введений туберкулін. Через 24 години при лапаратомії виявлено венозну гіперемію та набряк очеревини. У мазках-відбитках з очеревини велика кількість лімфоцитів та моноцитів. Який патологічний процес у тварини?
=Алергічне запалення
Серозне запалення
Гнійне запалення
Фібринозне запалення
Асептичне запалення

У хворого, 70 років, перебіг атеросклерозу ускладнився тромбозом судин нижніх кінцівок, виникла гангрена пальців лівої стопи. Початок тромбоутворення найбільш ймовірно пов’язаний
=Адгезією тромбоцитів
Активацією протромбінази
Перетворенням протромбіну в тромбін
Перетворення фібриногену в фібрин
Зниженням синтезу гепарину

У юнака 20 років травмоване праве яєчко. Яку небезпеку це може становити для лівого (здорового) яєчка на 2-3 тижні після
=Демаскування антигену та виникнення ушкодження антитілами
Розвиток інфекційного процесу
Розвиток атрофії
Розвиток гіпертрофії
Не загрожує нічим

У хлопчика 5 міс., при дослідженні імунного статусу виявлено зменшення імуноглобулініві, особливо IgA та IgM. В крові та лімфатичних вузлах відсутні В-лімоцити та плазматичні клітини. Реакції Т-лімфоцитів збережені. Захворювання передається по спадковості як зчеплене із статтю. Який діагноз буде найбільш ймовірним?
=Хвороба Бутона
Синдром Луі-Барр
Синдром Віскотта-Олдрича
Імунодефіцит швейцарського типу
Рання гіпогаммаглобулінемія

У хлопчика 3 років з вираженим геморагічним синдромом відсутній антигемофільний глобулін А (фактор VIII) в плазмі крові. Яка фаза гемостазу первинно порушена у цього хворого?
=Внутрішній механізм активації протромбінази
Зовнішній механізм активації протромбінази
Перетворення протромбіну в тромбін
Перетворення фібриногену в фібрин
Ретракція кров’яного згустку

У хворого в результаті вогнепального поранення стегна пошкоджений сідничний нерв. Будь-який вплив на хвору кінцівку спричиняє жорсткий, нестерпний біль. Який механізм формування болісних відчуттів найбільш ймовірний у цьому
=Каузалгічний
Рефлекторний
Фантомний
Гіпофункція ендорфіну
Гіпофункція енкефаліну

Хворий, 64 роки, поступив у клініку зі скаргами на кашель з харкотинням, виражену задишку. Об’єктивно: положення вимушене, частота дихання – 32 за хв., в акті дихання беруть участь допоміжні м’язи. Рентгенологічно: легені підвищеної прозорості. Що є найбільш суттєвою ланкою в патогенезі дихальної недостатності у цього хворого?
=Зниження еластичних властивостей легень
Накопичення в бронхах харкотиння
Стоншення слизової оболонки бронхів
Порушення сурфактантної системи легень
Фіброз легень

У тварини в експерименті проведена перерізка обох блукаючих нервів. Як зміниться характер дихання у тварини?
=Рідке і глибоке
Часте та глибоке
Рідке та поверхове
Часте та поверхове
Періодичне

Хворий, 44 років, скаржиться на задуху, серцебиття, болі в правому міжребір’ї, набряки на ногах.. На ЕКГ – ознаки гіпертрофії обох шлуночків та правого передсердя. Діагностовано недостатність тристулкового клапана. Який патогенетичний варіант цієї недостатності?
=Перевантаження серця об’ємом
Перевантаження серця опором
Первинна міокардіальна недостатність
Коронарна недостатність
Тампонада серця

При обстеженні у юнака 16 років було виявлено прискорення серцебиття під час вдиху, сповільнене – під час видиху. На ЕКГ відмічалося: вкорочення інтервалу RR під час вдоху та подовження його під час видоху. Назвіть вид аритмії
=Синусова аритмія
Миготлива аритмія
Синусова тахікардія
Ідіовентрикулярний ритм
Синусова брадикардія

У хворого, 42 років, скарги на болі в епігастральній ділянці, блювоту; блювотні маси кольору “кофейної гущі”; мелена. В анамнезі виразкова хвороба шлунку. Аналіз крові: еритроцити – 2,8х1012/л , лейкоцити – 8х109/л, гемоглобін 90 г/л. Вкажіть найбільш ймовірне ускладнення, яке виникло у хворого?
=Кровотеча
Пенетрація
Перфорація
Пререродження в рак
Пілоростеноз

У хворого після отруєння грибами з’явилося жовте забарвлення шкіри та склер, темний колір сечі. Який пігмент спричинює забарвлення сечі у хворого на гемолітичну жовтяницю?
=Стеркобілін
Моноглюкуронід білірубіну
Некон’югований білірубін
Вердоглобін
Білівердин

Для моделювання виразки шлунка тварині ввели в гастральні артерії атофан, який спричинює їх склерозування. Який механізм пошкодження слизової оболонки шлунку є провідним в даному експерименті?
=Гіпоксичний
Нейродистрофічний
Механічний
Дисрегуляторний
Нейрогуморальний

Хвора на хронічний гепатит скаржиться на підвищення чутливості до барбітуратів, які раніше вона переносила без симптомів інтоксикації. З порушенням якої функції печинки це пов’язане у найбільшій мірі?
=Метаболічної
Утворення жовчі
Гемодинамічної
Гемопоетичної
Фагоцитарної

У хворого на бронхіальну астму виникла гостра недостатність дихання. Який тип недостатності дихання виникає в даному випадку?
=Обструктивне порушення альолярної вентиляції
Рестриктивне порушення альолярної вентиляції
Перфузійний
Дифузійний
Дисрегуляторне порушення альолярної вентиляції

Щуру в плевральну порожнину введено 0,5 мл повітря. Який тип недостатності дихання виникає в даному випадку?
=Рестриктивне порушення альолярної вентиляції
Обструктивне порушення альолярної вентиляції
Перфузійний
Дифузійний
Дисрегуляторне порушення альолярної вентиляції

У хворого на правець виникла гостра недостатність дихання. Який тип недостатності дихання виникає в даному випадку?
=Дисрегуляторне порушення альолярної вентиляції
Рестриктивне порушення альолярної вентиляції
Обструктивне порушення альолярної вентиляції
Перфузійний
Дифузійний

У ВІЧ-інфікованого хворого спостерігається пригнічення активності імунної системи. Ураження яких клітин найбільшою мірою обумовлює стан імунодефіциту у цього хворого?
=Т-хелперів
Т-супресорів
Макрофагів
В-лімфоцитів
Т-кілерів

При мікроскопічному дослідження пунктату з осередка запалення у хворого із абсцесом шкіри знайдено велику кількість різних клітин крові. Які з цих клітин першими надходять із судин до тканин при запаленні?
=Нейтрофіли
Моноцити
Базофіли
Еозинофіли
Лімфоцити

У хворого на атрофічний гастрит виник дефіцит вітаміу В12. Яка зміна лейкоцитарної формули є найбільш типовою для гіповітамінозу В12?
=Ядерний зсув вправо
Лімфоцитоз
Моноцитоз
Регенеративно-дегенеративний ядерний зсув вліво
Регенеративний ядерний зсув вліво

На п*яту добу післі гострої крововтрати у хворого діагностована гіпохромна анемія. Який головний механізм у розвитку
=Надходження з кісткового мозку незрілих еритроцитів
Порушення всмоктування заліза у кишечнику
Посилене руйнування еритроцитів у селезінці
Порушення синтезу глобіну
Підвищення виділення заліза з організму

При дослідженні стану імунної системи хворого із хронічними грибковими ураженнями шкіри виявлено порушення клітинного імунітету. Зниження яких показників найбільш характерні при цьому?
=Т-лімфоцитів
Імуноглобулінів G
Імуноглобулінів E
В-лімфоцитів
Плазмоцитів

У хворого після відкритої травми хребта виявлено розрив правої половини спинного мозку. Зникнення якого виду чутливості слід очікувати тількі з боку розриву?
=Пропріоцептиної
Температурної
Больової
Тактильної


В експерименті кролю ввели нефроцитотоксичну сироватку морськоі свинки. Яке захворювання нирок моделювалося в цьому досліді?
=Гострий дифузний гломерулонефрит
Нефротичний синдром
Гострий пієлонефрит
Хронічна ниркова недостатність
Хронічний пієлонефрит

У хворого після важкої травми грудної клитки розвинувся шок та з’явилися ознаки гострої ниркової недостатності [ГНН]. Що є провідним механізмом розвитку ГНН в даному випадку?
=Падіння артеріального тиску
Порушення відтоку сечі
Підвищення тиску в капсулі клубочка
Підвищення тиску в ниркових артеріях
Зменшення онкотичного тиску крові

У хворого із хронічною нирковою недостатністю встановлення зменшення кліренсу за інуліном до 60 мл/хв. З порушенням якої функції нирок це пов’язано?
=Клубочкової фільтрації
Канальцевої секреції
Реабсорбції в проксимальному відділі нефрону
Реабсорбції в дистальному відділі нефрону
Реабсорбції в збиральних ниркових трубочках

У хворого на хронічну ниркову недостатність з’явилися анорексія, диспепсія, порушення ритму серця, свербіння шкіри. Який механізм розвитку цих порушень є головним?
=Накопичення продуктів азотистого обміну в крові
Порушення ліпідного обміну
Зміни вуглеводного обміну
Нирковий ацидоз
Порушення водно-електролітного обміну

У хворого на цукровий діабет розвинулася діабетична кома внаслідок порушення кислотно-основного стану. Який вид порушення виник при цьому?
=Метаболічний ацидоз
Метаболічний алкалоз
Респіраторний ацидоз
Газовий алкалоз
Негазовий алкалоз

У дитини 6 років розвинулася гіперергічна форма запалення верхніх дихальних шляхів. З’явилася загорза серйозного порушення дихання, а тому виникла необхідність застосувати протизапальні гормони. Серед гормонів протизапальний ефект проявляє
=Кортизол
Адреналін
Соматотропін
Тестостерон
Інсулін

До лікаря звернувся чоловік 27 років. При огляді було виявлено збільшення кистей, стоп та нижньої щелепи. Крім того спостерігалась деформація суглобів (kiphosis), гормональні порушення (імпотенція, атрофія яєчок). Функції якої залози порушені?
=Передньої частини гіпофізу
Надниркових залоз
Шишкоподібного тіла
Щитовидної залози
Прищитовидних залоз

У хворого Д., 32 роки, гнійна рана у нижній трерині передпліччя. Хворому зроблено мазок із гнійного вмісту рани. Які клітини в основному виявлено при забарвленні мазку за Романовським-Гімзою?
=Нейтрофіли
Еозинофіли
Лімфоцити
Еритроцити
Базофіли

Під час бойових дій у госпіталь доставили солдата з тяжким осколочним пошкодженням хребта. У пораненого встановлено наявність перерізки правої половини спинного мозку (синдром Броун-Секара). Зникненням якого виду чутливости проявляеться цей синдром?
=пропріоцептивної - справа
температурної - справа
пропріоцептивної - зліва
больової - справа
тактильноі - справа

Хлопчик 1,5 років постійно хворіє на піодермію та тричі хворів на пневмонію. В крові знижена кількість имуноглобулінів G та
=Який вид імунодефіциту виник у дитини?
=Гіпогаммаглобулінемія Брутона
Швейцарський тип
Гіпоплазія вилочкової залози
Синдром Віскотта –Олдрича
Синдром Луи-Барр

Жінці поставлено діагноз ерозія шійки матки, яка є передпухлинною патологією. Який захисний механізм може попередити розвиток пухлини?
=Збільшення природних кілерів (NK-клітин)
Високодозова імунологічна толерантність
Збільшення активності лізосомальних ферментів
Спрощення антигенного складу тканин
Низькодозова імунологічна толерантність

Хворого було доставлено до лікарні у стані діабетичної коми. Дихає хворий шумно, після глибокого вдоху йде посилений видих. Який тип дихання спостерігається у цього хворого?
=Куссмауля
Чейна-Стокса
Гаспінг
Апнейстичне
Біота

У хворого на дифтерію розвинувся набряк гортані. При цьому спостерігається рідке та глибоке дихання з затрудненням вдоху. Який тип дихання спостерігається при цьому?
=Стенотичне
Куссмауля
Чейна-Стокса
Апнейстичне
Гаспінг

При моделюванні запалення на брижі жаби спостерігали крайове стояння лейкоцитів та їх еміграцію крізь судинну стінку. Який із перелічених факторів обумовлює цєй процес?
=Вплив хемотаксичних речовин
Збільшення онкотичного тиску в осередку запалення
Зниження онкотичного тиску в судинах
Збільшення гідростатичного тиску в судинах
Зменшення гідростатичного тиску в судинах

Під час огляду шкіри лікар помітив у хворого нагнійний процес у вигляді круглих підвищень червонуватого коліру, оточених зоною гіперемії. Які медиатори запалення зумовили явища судинної гіперемії?
=Гістамін
Інтерлейкін 1
Фактор активації тромбоцитів .
Тромбоксан
Лізосомальні ферменти

У жінки 57 років після тривалого больового нападу у правому підребір’ї з’явилась жовтяниця, після чого хвора звернулась до лікаря. Виникла підозра на появність у хворої гострого калькульозного холециститу. Дослідження якого показника крові свідчить про непрохідність жовчних протоків?
=Вільного та зв’язаного білірубіну
Білкових фракцій
Загальних ліпідів
Сечової кислоти
Залишкового азоту

Хлопчику 15 років встановлено діагноз гострого вірусного гепатиту. Дослідження якого показника крові необхідно провести для підтвердження гострого ураження печінкових клітин?
=Активність амінотрансфераз [ЛЛТ і АСТ]
Вміст вільного та зв’язаного білірубіну
Швидкість осідання еритроцитів [ШОЕ].
Рівень холестерину
Вміст білкових фракцій

У чоловіка 25 років діагностований гострий дифузний гломерулонефрит. З анамнезу хворого відомо, що за 18 днів до прояви цієї хвороби переніс ангіну. Який механізм ураження ниркових клубочків буде спостерігатися у цьому випадку?
=Імунний.
--
Нефротоксичний.
Ішемічний.
Медикаментозний.

У хворого внаслідок отруєння бертолетовою сіллю розвинулася гемічна гіпоксія. Утворення якої речовини грае роль в патогенезі цієї гіпоксиї?
=Метгемоглобіну
Оксиду азоту
Сульфгемоглобіну
Карбгемоглобіну
Карбоксігемоглобіну

Білому щуру ввели під шкіру сулему в дозі 5 мг/кг маси тіла. Через 24 години в плазмі крові концентрація креатиніну збільшилася в декілька разів. Який механизм ретенційної азотемії в даному випадку ?
=Зниження клубочкової фільтрації
Збільшення утворення креатиніну в м`язах
Зростання реабсорбції креатиніну
Зростання клубочкової фільтрації
Збільшення секреції креатиніну в канальцях нирок

Внаслідок травмування у хворого видалили прищитовидні залози, що супроводжувалося: млявістю, спрагою, різким підвищенням нервово-м’язової збудливості. З порушенням обміну якої речовини це пов’язано:
=Кальцію
Марганцю
Хлору
Молібдену
Цинку

У хворого з гломерулонефритом виявлено: анасарка, АТ 185/105 мм рт.ст. В крові: анемія, лейоцитоз, гіперазотемія, гіпопротеінемія. Який показник свідчить про ускладнення гломерулонефриту нефротичним синдромом ?
=Гіпопротеїнемія
Лейкоцитоз
Гіперазотемія
Артеріальна гіпертензія
Анемія

Чоловік 63 років, страждає раком стравоходу, метастази в лімфатичні вузли середостіння, ракова кахексія. Яка патогенетична стадія пухлинного процесу чоловіка?
=прогресії
промоції
трансформації
ініціації
---

Жінка 67 років, страждає раком шлунку з метастазами у печінку. Яка властивість пухлинних клітин обумовлює їх здатність до метастазування?
=інфільтративний ріст
біохімічний атипізм
швидкий ріст
автономність
антигенна анаплазія

Чоловік 27 років, скаржиться на загальну слабкість, задишку, підвищення температури (37-38 оС). Захворів тиждень тому, після переохолодження. Легкий ціаноз губ. ЧД 28 за хв., пульс 90 уд/хв., Права половина грудної клітин відстає в акті дихання, міжреберні проміжки згладжені. При перкусії: нижче вугла лопатки тупість, дихання не вислуховується. Діагноз: правосторонній ексудативний плеврит. Який механізм розвитку недостатності дихання має місце у чоловіка?
=порушення рухливості грудної клітини
рестриктивна недостатність дихання
обструктивна недостатність дихання
порушення дифузії газів в легенях
зменшення перфузії легень кров*ю

Жінку 44 років вжалила оса внаслідок чого розвинувся шок. В анамнезі вже була тяжка алергічна реакція на жалення оси. Об*єктивно: пульс 179 уд /хв, слабкий, АТ-80/40 мм рт. ст., ЧД-26 за хв. Яка провідна ланка патогенезу анафілактичного
=зниження периферійного опору судин
тахікардія
біль
зменшення ударного об*єму серця
зменшення об*єму циркулюючої крові

Хворий жаліється на підвищену дратливість, періодичний субфібрілітет. Частота пульсу - 120 за хв. В крові збільшена кількість гормонів Т3 і Т4. Яку ендокринну патологію найбільш логічно запідозрити?
=Гіпертіреоз.
Надниркову недостатність.
Гіпопаратіреоз.
Гіперпаратіреоз.
Гіпотіреоз.

В реанімаційне відділення доставили водія, який попав в автомобільну аварію. Він не зразу реагує на питання, байдужий до всього, блідий, дихання поверхневе, рідке, АТ дорівнює 75/50 мм рт. Ст. Назвіть головну ланку патогенезу вказаної
=Гальмування центральної нервової системи.
Збудження ЦНС.
Крововтрата.
Токсемія.
Перерозподіл крові.

До лікарні поступив юнак 17 років зі скаргами на затримання росту, посивіння, облисіння, зниження гостроти зору і слуху, гіперкератоз. В крові гіперглікемія, гіперхолестеринемія. Для якоі патології характерні ці явища?
=Синдром Вернера.
Синдром Гетчинсона-Гілфорда.
Синдром Шерешевського-Тернера.
Сімейна гіперхолестеринемія.
Синдром Клайнфельтера.

Хворий скаржиться на періодичне ослаблення стула, яке пов’язує з прийомом багатої на жири їжі. При цьому він відмічає зменшення забарвленості калу. При лабораторному обстеженні встановлено нормальний вміст ліпідів в сироватці крові. Порушення якого із станів ліпідного обміну має місце у даного
=Всмоктування.
Транспорту кров’ю.
Проміжного обміну.
Депонування в жировій тканині.
Мобілізація із жирової тканини.

При повному (з водою) аліментарному голодуванні розвинулись генералізовані набряки. Який із патогенетичних факторів у цьому випадку є ведучим?
=Зниження онкотичного тиску плазми крові.
Зниження гідростатичного тиску міжклітинної рідини.
Зниження осмотичного тиску плазми крові.
Підвищення онкотичного тиску тканинної рідини.
Підвищення осмотичного тиску міжклітинної рідини.

У пілота на висоті 14000 м трапилася аварійна розгереметизація кабіни. Який із видів емболій у нього розвинувся?
=Газова.
Емболія стороннім тілом.
Тромбоемболія.
Повітряна.
Жирова

Опікова хвороба, окрім іншого, характеризується розвитком анемії, однією з причин якої вважається дефект:
=Еритропоетину
Мієлопоетину
Вітаміну В12
Катехоламінів
Тромбопоетину

При работе с радиоактивными веществами сотрудник вследствие аварии получил дозу общего облучения 4 Гр. Жалуется на головную боль, тошноту, головокружение. Какие изменения в составе крови можно ожидать у больного через 10 часов после облучения?
=Нейтрофильный лейкоцитоз
Лимфоцитоз
Лейкопению
Агранулоцитоз
Нейтропению

У больной А., 18 лет, после перенесенной краснухи начала отмечаться потеря массы тела, постоянное ощущение сухости во рту, жажду, повышение аппетита, частое мочеотделение. Объективно: суточное количество мочи 6 л, глюкоза крови 17,8 ммоль/л, в моче выявлена глюкоза и ацетон. Каков наиболее вероятный патогенетический механизм вызвал повышение уровня глюкозы у больной?
=Уменьшение выработки инсулина
Увеличение глюконеогенеза
Повышенное разрушение инсулина
Повреждение инсулиновых рецепторов клеток
Увеличение выработки глюкокортикоидов

У больного С., выявлены такие изменения в периферической крови: Эр. 3,2x1012/л, Гем. 80 г/л, Лейк. 25x109/л. Лейкоцитарная формула: базофилы - 5\%, эозинофилы - 9\%, миелобласты - 3\%, промиелоциты - 8\%; миелоциты - 11\%, метамиелоциты - 22\%, палочкоядерные - 17\%, сегментоядерные - 19\%, лимфоциты - 3\%, моноциты - 3\%. Определите наиболее вероятную патологию соответствующую данному описанию
=хронический миелолейкоз
острый миелобластный лейкоз
эритромиелоз
лейкемоидная реакция
недеферинцируемый лейкоз

У больного П., выявлены такие изменения в периферической крови: Эр. 3,0x1012/л, Гем. 80 г/л, Лейк. 21x109/л. Лейкоцитарная формула: базофилы - 0\%, эозинофилы - 0\%, миелобласты - 54\%, промиелоциты - 1\%; миелоциты - 0\%, метамиелоциты - 0\%, палочкоядерные - 1\%, сегментоядерные - 28\%, лимфоциты - 13\%, моноциты - 3\%. Определите наиболее вероятную патологию соответствующую данному описанию картины крови:
=острый миелобластный лейкоз
хронический миелолейкоз
эритромиелоз
лейкемоидная реакция
недеферинцированный лейкоз

Больной Л., жалуется на отрыжку, изжогу, частые запоры. При титровании желудочного сока получили такие данные: общая кислотность - 88 млмоль/л., общая HCl - 83 млмоль/л, свободная HCl - 50 млмоль/л, связанная HCl - 33 млмоль/л, кислые фосфаты и органические кислоты - 5 млмоль/л. Оценить состояние кислотности желудка:
=Гиперацидное состояние
Гипоацидное состояние
Ахилия
Нормацидное состояние
Гипохлоргидрия

У больного Д., после длительного приступа сильных головных болей стали невозможны активные движения левой руки и ноги. На этих конечностях тонус мышц повышен, мышцы спазмированы, спинальные сухожильные рефлексы резко усилены, расширены зоны рефлексов. Какое расстройство нервной системы имеется у больного?
=Центральный паралич
Периферический паралич
Экстрацирапидный паралич
Вялый паралич
Рефлекторный паралич

Больной 38 лет доставлен в приемное отделение с признаками гипоксии, развившейся после отравления угарным газом. Объективно: состояние средней тяжести, тахикардия, одышка, АД 160/100. Какой механизм токсического действия окиси углерода на организм?
=Образование карбоксигемоглобина
Образование метгемоглобина
Нарушение диссоциации оксигемоглобина
Образование карбгемоглобина
Блокада кальциевых каналов эритроцитов

У больного П., после травмы возникла необходимость введения противостолбнячной сыворотки, однако проба на чувствительность к сыворотке оказалась положительной. Как провести специфическую гипосенсибилизацию у больного? Введением:
=малых дробных доз специфического аллергена
физиологических доз глюкокортикоидов
разрешающей дозы специфического аллергена
лечебных доз антигистаминных препаратов
наркотических веществ снижающих чувствительность

У больной Л., 40 лет, через месяц после сочетанной автомобильной травмы отсутствуют активные движения во всех суставах правой ноги. Объем мышц в области голени правой ноги на 2 см меньше, чем на левой. Ахилов и коленный рефлексы справа отсутствуют. Определяется термгипестезия и гипальгезия на наружной поверхности правой ноги, потеря проприоцептивной чувствительности в области стопы. Нарушением какого отдела двигательного анализатора обусловлены имеющиеся расстройства.
=Периферических нервов
Спиномозговых проводящих путей
Пирамидных нейронов
Нервно-мышечной синаптической передачи
Экстрапирамидных нейронов

Больному 25 лет установлен диагноз хронического гепатита. Больной жалуется на потерю массы тела на 10 кг на протяжении 2 месяцев. Объективно: кожа сухая, шелушащаяся, бледная с желтоватым оттенком, мелкоточечные кровоизлияния на коже, кровоточивость десен. Нарушение какой функции печени отражают мелкоточечные кровоизлияния и кровоточивость
=Белковосинтетической
Пигментообразующей
Гликогенсинтетической
Детоксицирующей
Депонирующей

У больного П., с сердечной недостаточностью возникли явные признаки гипоксии головного мозга и развилась одышка. В патогенезе какой одышки (из ниже перечисленных) основным звеном является понижение возбудимости дыхательного центра к углекислоте вследствие кислородного голодания этого
=периодическое дыхание
экспираторная одышка
инспираторная одышка
частое дыхание
глубокое дыхание

Больной Т., 19 лет, с диагнозом “острая пневмония” был направлен на лечение. При поступлении дыхание 32 в минуту, поверхностное. В дыхательных движениях участвуют межреберные мышцы. При аускультации выслушиваются мелкопузырчатые влажные и сухие хрипы. При рентгеноскопии легких изменения, характерные для двусторонней крупозной пневмонии. Каков механизм частого поверхностного дыхания при пневмонии?
=Перераздражение легочных рецепторов блуждающих нервов
Интоксикация продуктами воспаления дыхательного центра
Повышение возбудимости дыхательного центра
Торможение коры мозга вследствие интоксикации
Торможение блуждающих нервов

Больной И., 38 лет, поступил с жалобами на боль в боку, усиливающуюся при вдохе и кашле. Болевые ощущения уменьшаются в положении лежа на пораженном боку. При обследовании дыхание частое и поверхностное, заметно ограничение дыхательной подвижности соответствующей половины грудной клетки. Каков механизм изменения характера дыхания у больного?
=Ограничение корой мозга рефлекса Геринга-Брейера
Перераздражение легочных рецепторов блуждающих нервов
Интоксикация дыхательного центра продуктами воспаления
Повышение возбудимости дыхательного центра
Торможение коры мозга вследствие интоксикации

Больной Ж., 48 лет, находится в реанимации после отравлением четыреххлористым углеводородом. Состояние тяжелое, больной без сознания, пульс нитевидный, слабого наполнения, ЧСС 40 уд/мин АД 75/40 мм.рт.ст., дыхание периодическое типа Биота. В крови прямой билирубин - 155 мкмоль/л. В моче высокое содержание аммиака и билирубина. Что явилось основной причиной описанного состояния у
=Печеночная недостаточность,
Кардиогенный шок
Дыхательная недостаточность
Сердечная недостаточность
Почечная недостаточность

Больной А, 59 лет, директор частного предприятия. После проверки налоговой инспекцией вечером появились интенсивные жгучие боли, локализованные за грудиной, иррадиирующие в левую руку. Через 15 мин состояние больного нормализовалось. Какой из механизмов развития ишемии сердечной мышцы является ведущим у данного
=Повышение в крови уровня катехоламинов
Атеросклероз коронарных артерий
Внутрисосудистая агрегация форменных элементов
Сдавление коронарных артерий при дилятации полостей сердца
Функциональная перегрузка сердца

43-летний больной поступил в нефрологическое отделение с массивными отеками. Два года лечился амбулаторно и при этом постоянно отмечалось повышенное АД. Дважды лечился преднизолоном, с положительным эффектом. В моче: относительная плотность 1017, белок 4,0 г/л, Эр – 15-18 в поле зрения (выщелоченные), Лей - 5-7 в поле зрения. Какая преимущественно функция почек нарушена у больного?
=Фильтрационная
Реабсорбционная
Секреторная
Инкреторная
Концентрационная

У женщины на 7-м месяце беременности стала быстро нарастать анемия: Эритроциты - 2,7 х 10 /л, Нв -110 г/л, ЦП -1,2 анизоцитоз, пойкилоцитоз, единичные мегалоциты. Какой вид анемии развился у женщины?
=В12 - дефицитная анемия
Железодефицитная анемия
Гемолитическая анемия
Постгеморрагическая анемия
Таласемия

У больного тяжелая нефропатия с массивным отечным синдромом, осложнившая хроническую бронхоэктатическую болезнь. Лабораторные исследования выявляют обильную протеинурию, цилиндрурию, значительное снижение содержание белка в сыворотке крови, гиперлипемию, гипокалиемию и др. отклонения. Что является первичным и наиболее существеным звеном в патогенезе отеков у данного
=Снижение онкотического давления крови
Повышение гидростатического давления крови
Повышение давления внеклеточной жидкости
Блокада лимфооттока
Повышение проницаемости микрососудов

У больного через полторы недели после тяжелой стрептококковой ангины проявилась отечность, повысилось артериальное давление. В моче гематурия и умеренная протеинурия. В крови антистрептококковые антитела и снижение компонентов комплемента. В микрососудах каких структур наиболее вероятно локализация скоплений иммунных комплексов обусловивших развитие нефропатии?
=Клубочков
Пирамидах
Лоханок
Мочеточников
Мочевого пузыря

У беременной женщины развился токсикоз с тяжелыми повторными рвотами на протяжении суток. К концу суток начали проявляться тетанические судороги и обезвоживание организма. Какой сдвиг КЩР вызвал описанные изменения?
=Негазовый выделительный алкалоз
Газовый алкалоз
Газовый ацидоз
Негазовый метаболический ацидоз
Негазовый выделительный ацидоз

Больной сахарным диабетом поступил в клинику ввиду ухудшения состояния: общая слабость, полиурия, жажда, заторможенность и сонливость. Отмечается дыхание Куссмауля, аритмия сердца, запах ацетона в выдыхаемом воздухе. Какой сдвиг КЩР обеспечил описанную симптоматику?
=Негазовый метаболический ацидоз
Газовый ацидоз
Газовый алкалоз
Негазовый алкалоз
Негазовый выделительный ацидоз

Во время обеда ребенок поперхнулся и аспирировал пищу. Начался сильный кашель, кожа и слизистые цианотичны, пульс учащен, дыхание редкое, Выдох удлинен. Какое нарушение внешнего дыхания развилось у ребенка?
=Стадия экспираторной одышки при асфиксии
Стадия инспираторной одышки при асфиксии
Дыхание Биота
Дыхание Куссмауля
Дыхание Чейна-Стокса

При підйомі в гори у альпініста розвинулась ейфорія, головний біль, запаморочення, серцебиття, задишка, яка чергувалася з апное. Яке порушення кислотно-основного стану розвинулося у альпініста?
=Газовий алкалоз
Метаболічний алкалоз
Негазовий алкалоз
Газовий ацидоз
Негазовий ацидоз

Хворому поставили діагноз – синдром подразненої кишки. Виділення якого інкрету найбільш вірогідно збільшується при цьому?
=Мотиліну
Інтестинального пептиду
Глюкагону
Урогастрону
Секретину

Внаслідок поранення хворий втратив 25\% об’єму циркулюючої крові. Назвіть терміновий механізм компенсації крововтрати.
=Находження міжтканинної рідини в судини
Відновлення білкового складу крові
Збільшення числа ретикулоцитів
Відновлення числа еритроцитів
Активація еритропоезу

У робітника, який працював літом у щільному костюмі, різко підвищилась температура тіла, з’явились задишка, тахікардія, нудота, судоми, втрата свідомості. Що явилось причиною тяжкого стану робітника?
=Зниження тепловіддачі
Підвищення теплопродукції
Підвищення тепловіддачі
Зниження теплопродукції
Тепловіддача дорівнює теплопродукції

В приймальне відділення поступила дитина 1,5 р. з ознаками отруєння нітратами: стійкий ціаноз, задишка, судоми. Утворення якої форми гемоглобину лежить в основі цих симптомів?
=метгемоглобіну.
карбгемоглобіну
карбоксигемоглобіну
редукованого гемоглобіну
оксигемоглобіну

Хвора 3 р., поступила в дитячу клініку у важкому стані з гемоглобінопатією (серпоподібноклітинна анемія). Заміна якою аминокислотою глутамінової кислоти в бета–ланцюгу глобіну лежить в основї утворення патологічного гемоглобіну в даному випадку?
=валіном.
серином
тирозином
фенілаланіном
аргініном

Хвора поступила в клініку на обстеження. З дитинства відмічалось зниження гемоглобіну до 90-95 г/л. Лікування препаратами заліза було неефективне. Аналіз крові при поступленні: Е–3,2(1012/л, Hb–85 г/л, к.п.–0,78. В мазку анізоцитоз, пойкілоцитоз, мішеневидні еритроцити, ретикулоцити –16\%. Поставлений діагноз – таласемія. До якого виду гемолітичних анемій можна віднести дане захворювання?
=Спадкова гемоглобінопатія.
Спадкова мембранопатія
Набута мембранопатія
Спадкова ферментопатія
Набута ферментопатія

Хворий на протязі останнього року став відмічати під_вищену втомлюваність, загальну слабість. Аналіз крові: Е–4.1(1012/л, Hb–119г/л, к.п.–0.87, лейкоцити – 57(109/л, лейкоформула: Ю–0, П–0, С–9\%, Е–0, Б–0, лімфобласти–2\%, пролімфоцити–5\%, лімфоцити–81\%, М–3\%, тромбо_цити–160(109/л. В мазку: нормохромія, велика кількість тіней Боткіна–Гумпрехта. Про яку патологію системи крові свідчить дана гемограма?
=Хронічний лімфолейкоз.
Хронічний мієлолейкоз
Гострий лімфобластний лейкоз
Гострий мієлобластний лейкоз
Хронічний монолейкоз

У хворого діагностована пелагра. Порушення обміну якої амінокислоти лежить в основі цього захворювання?
=Триптофану
Тирозину
Фенілаланіну
Цистеїну
Валіну

У хлопчика М., 2 років була діагностована хвороба Гірке, що супроводжується надмірним відкладанням глікогену в печінці і нирках гіпоглікемією. При біохімічному дослідженні крові
=Дефіцит глюкозо-6-фосфатази
Зниження активності аміло-1,6-глюкозидази
Дефіцит фруктозо-дифосфат альдолази
Зниження активності фосфорилази
Дефіцит кетогексокінази

Масугі викликав розвиток гломерулонефриту у щурів таким чином: гомогенат нирок щура вводив кролю. Через декілька тижнів сироватку сенсибілізованого кроля вводив щурам. Який тип алергічної реакциї за Джеллом та Кумбсом лежить в основе розвитку гломерулонефрита у щурів?
=Цитотоксичний
Анафілактичний
Імунокомплексний
Гіперчутливість сповільненого типу
Стимулюючий

У хворої П., 45 років при електрокардіографічному обстеженні на ЕКГ виявлено такі зміни: інтервал Р–Q подовжений, при цьому випадає кожен другий або третій комплекс QRST. Яке саме порушення провідності серця тут спостерігається?
=Атріовентрикулярна блокада Ш–го ступеня
Атріовентрикулярна блокада повна
Синоаурікулярна блокада
Внутрішлуночкова блокада
Атріовентрикулярна блокада І–го ступеня

З метою пригнічення аутоімунних реакцій після пересадки органів обов*язковим є проведення курсу гормонотерапії. Які гормони застосовують з цієї мети ?
=Глюкокортикоїди.
Мінералокортикоїди.
Статеві гормони.
Адреналін.
Соматотропний гормон.

У хворого, що прибув із Тунісу, виявлена альфа-таласемія з гемолізом еритроцитів і жовтяницею. Хвороба була діагностована на основі наявності в крові
=Мішенеподібних еритроцитів
Зернистих еритроцитів
Поліхроматофільних еритроцитів
Нормоцитів
Ретикулоцитів

Після тотальної резекції шлунка у хворого розвинулася тяжка В12-дефіцитна анемія. Свідченням її була наявність в крові
=Мегалобластів
Мікроцитів
Овалоцитів
Нормоцитів
Анулоцитів

На шостому місяці вагітності в жінки з’явилася виражена залізодефіцитна анемія. Діагностичною ознакою її була поява в крові
=Анулоцитів
Макроцитів
Пойкілоцитів
Ретикулоцитів
Нормоцитів

У хворого 38 років, який переніс гепатит і продовжував вживати алкоголь, розвинулися ознаки цирозу печінки з асцитом і набряками на нижніх кінцівках. Які зміни складу крові стали вирішальними в розвитку набряків?
=Гіпоальбумінемія
Гіпоглобулінемія
Гіпохолестеринемія
Гіпокаліємія
Гіпоглікемія

Після вимушеного швидкого підняття водолаза з глибини на поверхню у нього з’явилися ознаки кесонної хвороби – біль у суглобах, свербіння шкіри, мерехтіння в очах, затьмарення свідомості. Яким видом ємболії вони були зумовлені?
=Газовою
Повітряною
Жировою
Тканинною
Тромбоємболією

У хворого Н. приступи гарячки виникають через день. Під час приступу температура різко підвищується і утримується на високому рівні до 2 год, а потім знижується до вихідного рівня. Цей тип гарячки характерний для
=Малярії
Поворотного тифу
Сепсису
Бруцельозу
Висипного тифу

В лікарню доставлений непритомний чоловік після отруєння чадним газом. Гіпоксія у нього зумовлена появою у крові
=Карбоксигемоглобіну
Метгемоглобіну
Карбгемоглобіну
Оксигемоглобіну
Дезоксигемоглобіну

Після занурення водолаза на глибину 60 м у нього з’явилися симптоми порушення функцій центральної нервової системи – збудження, ейфорія, ослаблення уваги, професійні помилки. Ці симптоми пов’язані з токсичною дією на нейрони
=Азоту
Кисню
Вуглекислого газу
Аміаку
Лактату

Електрик, працюючи з порушенням правил техніки безпеки, випадково торкнувся оголеного електропровода обома руками і загинув. Смерть настала внаслідок
=Фібриляції передсердь і шлуночків
Повної атріо-вентрикулярної блокади
Пригнічення автоматизму сино-атріального вузла
Зменшення скоротливої здатності міокарда
Порушення вагусної регуляції серця

До дерматолога звернулася пацієнтка із скаргами на екзематозне ураження шкіри рук, що з’являється після контакту з миючим засобом “Лотос”. Використання гумових рукавичок запобігає цьому. Патологічна реакція шкіри зумовлена
=Т-лімфоцитів
В-лімфоцитів
Моноцитів
Нейтрофілів
Базофілів

У хворого з синдромом Zollinger-Ellison [ пухлина підшлункової залози] відзначається збільшення секреції, перистальтики шлунково-кишкового тракту, а також діарея та пептичні виразки. Яка з перерахованих речовин, що секретується вказаною пухлиною, викликає цей комплекс симптомів?
=Гастрин
Вазоактивний інтестинальний пептид
Пепсин.
Трипсин
Секретин

У дитини двох років встановлено діагноз гіпоплазії тимуса. Який показник стану імунної системи є найбільш характерним для цього імунодефіцита?
=Зниження кількості Т-лімфоцитів
Зниження кількості В-лімфоцитів
Дефіцит Т и В-лімфоцитів
Відсутність плазматичних клітин
Зниження іммуноглобулінів М

Клінічне обстеження хворого дозволило встановити попередній діагноз рак печінки. Наявність якого білка в сироватці крові дозволить підтвердити діагноз?
=альфа-фетопротеїн
Пропердин
Парапротеїни
С-реактивний протеїн
гамма-глобуліни

У хворого на пневмонію виникла гарячка. Що безпосередньо спричинює зміну установочної точки температури в нейронах гіпоталамуса цього хворого?
=Простагландини $Е_1$, $Е_2$
Ендотоксин
Екзотоксин
Інтерлейкін-2
Тромбоцитарний фактор росту

При моделюванні запалення на брижі жаби під мікроскопом спостерігали розширення артеріальних судин, прискорення кровотоку, осьовий ток крові. Який вид артеріальної гіперемії виник при цьому?
=Метаболічна
Постішемічна
Вакатна
Реактивна
Робоча

Через 1 годину після накладання кільця, що звужує аорту, в собаки різко зросла сила та частота серцевих скорочень, а об’єм циркулюючої крові та товщина стінки лівого шлуночка не відрізнялися від вихідних показників. Яка стадія гіпертрофії міокарда спостерігається у тварини?
=Аварійна
Декомпенсації
Прогресуючого кардіосклерозу
Відносно стійкої гіперфункції
Завершеної гіпертрофії

У хворого на артеріальну гіпертензію наслідком гіпертонічного кризу стала гостра серцева недостатність. Який механізм серцевої недостатності є головним в даному випадку?
=Перевантаження серця опором
Перевантаження серця припливом крові
Ушкодження міокарда
Абсолютна коронарна недостатність
Відносна коронарна недостатність

Чоловік 20 років, що приймав участь у ліквідації наслідків Чорнобильської катастрофи захворів на пародонтит. Який етіологічний фактор є найбільш важливим у розвитку цієї патології?
=Ємоційне перенапруження
Дефіцит заліза
Неповноцінне харчування
Підвищення фізичного навантаження на зубощелепний аппарат
Стрептококки порожнини рота

У тварини викликали карієс. Який компенсаторний механізм є найважливішим при розвитку цього захворювання?
=Утворення одонтобластами вторинного дентину
Новоутворення емалі
Гіпотрофія слинних залоз
Пригнічення фагоцитозу
Гіперфункція прищитовидних залоз

У хворого на запалення трійчастого нерва останніми роками прогресує пародонтит. Який фактор є головним у розвитку пародонтиту?
=Нейродистрофічні порушення в пародонті
Зменшення активності лейкоцитарної еластази
Послаблення утворення імуноглобулінів
Підвищення тонуса вагуса
Зниження активності калікреїн-кінінової системи

Хворий з Прикарпаття, що страждає на єндемічниї зоб, звернувся до лікаря із скаргами на гноєтечі з ясенних закутків та розхитування зубів. Що в даному випадку є основним фактором розвитку пародонтиту?
=Ендокринні порушення
Стресорні впливи
Гіперсалівація
Порушення ковтання
Неповноцінне харчування

Мужчина 60 лет вследствие длительного пребывания в мокрой одежде при низкой температуре окружающей среды заболел крупозной пневмонией. Какова причина возникновения такой формы воспаления лёгких?
=Пневмококк
Возраст
Снижение реактивности организма
Воздействие на организм низкой температуры
Воздействие на организм высокой влажности

У больного, страдающего желчно - каменной болезнью, вследствие обтурации жёлчевыводящих путей обнаруживается обесцвеченный жирный кал. Отсутствие какого компонента желчи обусловливает явление стеаторреи?
=Жёлчных кислот
Холестерина
Жёлчных пигментов
Жирных кислот
Щелочной фосфатазы

У водолаза, проводившего работы на большой глубине, при быстром возвращении его в условия нормального атмосферного давления появилась боль в суставах, зуд кожи, нарушение зрения, потеря сознания. Как называется описанное явление?
=Болезнь декомпрессии
Состояние невесомости
Синдром взрывной декомпрессии
Баротравма
Гипероксия

У новорожденных крысят в эксперименте была удалена вилочковая железа. При этом развилась болезнь, которая характеризуется резким снижением в крови лимфоцитов, развитием инфекций, спленамегалией, остановкой роста и летальным исходом. Какое нарушение функции иммунной системы при этом наблюдается?
=Недостаточность системы Т - лимфоцитов
Недостаточность системы В - лимфоцитов
Гиперфункция системы Т - лимфоцитов
Гиперфункция системы В - лимфоцитов
Комбинированный дефект Т - и В - лимфоцитов

Предварительно сенсибилизированной морской свинке внутривенно ввели 10 мл лошадиной сыворотки и через полминуты отмечено: шерсть взъерошена, животное чихает, кашляет, почёсывает мордочку, дыхание затруднённое, судорожное, непроизвольная дефекация и мочеиспускание. Какая стадия анафилактического шока наблюдается у
=Патофизиологическая
Сенсибилизации
Патохимическая
Иммунологическая


У больного с переломом голеностопного сустава после снятия гипсовой повязки наблюдается отёк стопы, цианоз, местное понижение температуры, увеличение органа в объёме. Какой вид нарушения кровообращения наблюдается при этом?
=Венозная гиперемия
Рабочая гиперемия
Метаболическая артериальная гиперемия
Реактивная гиперемия
Ишемия

При микроскопии препарата брыжейки лягушки обнаружено, что в некоторых капиллярах отмечается маятникообразное движение крови, форменные элементы при этом (в частности, лейкоциты) из осевого слоя выходят в пристеночный, а некоторые даже выпускают псевдоподии в стенку капиляра. Какой стадии сосудистой реакции при воспалении соответствует описанное явление?
=Престаза
Стаза
Кратковременному спазму сосудов
Артериальной гиперемии
Венозной гиперемии

У больного наблюдаются приступы артериальной гипертензии, сопровождающиеся тахикардией, обильным потоотделением, резкой болью в надчревной области. Для какого из перечисленніх опухолевих заболеваний наиболее характерны данные симптомы?
=Феохромоцитома
Базофильная аденома гипофиза
Аденома клубочковой зоны надпочечников
Аденома щитовидной железы
Опухоль яичников

Установлено, что при развитии гепатомы в ней часто прекращается синтез жёлчных кислот. О каком виде анаплазии это свидетельствует?
=Функциональной
Энергетической
Морфологической
Биохимической
Физико- химической

У подопытного животного (крысы) путём внутривенного введения аллоксона был вызван экспериментальный сахарный диабет. Каков механизм действия данного вещества?
=Повреждение beta - клеток панкреотических островков
Связывание цинка
Образование антител к инсулину
Активация инсулиназы
Активация выработки контринсулярных гормонов

У больного сахарным диабетом отмечается высокий уровень гипергликемии, кетонурия, глюкозурия, гиперстенурия и полиурия. Какая форма нарушения кислотно-основного равновесия имеет место в данной ситуации?
=Метаболический ацидоз
Газовый ацидоз
Метаболическия алкалоз
Газовый алкалоз
Выделительный алкалоз

У больного 40 лет после перенесенной травмы головного мозга стало наблюдаться повышенное выделение мочи (до 8 - 10 л/сутки) и жажда. Анализ мочи показал отсутствие глюкозурии и низкую относительную плотность. Какая форма нарушения водно-солевого обмена возможна в некомпенсируемый период описанного заболевания?
=Гиперосмолярная гипогидрадация
Изоосмолярная гипогидрадация
Гипоосмолярная гипогидратация
Изоосмолярная гипергидратация
Гипоосмолярная гипергидратация

У больного с гемолитической желтухой в мазке крови присутствуют эритроциты в виде микросфероцитов 1 - 6 в поле зрения. Какова возможная причина гемолиза эритроцитов, вызывающая возникновение такой формы желтухи?
=Наследственный дефект развития их мембран
Наследственный дефект структуры гемоглобина
Нарушение ферментных систем эритроцитов
Воздействие на мембрану эритроцитов жёлчных кислот
Воздействие на мембрану эритроцитов билирубина

У больного в третьем периоде лихорадки наступило критическое падение температуры тела. При этом наблюдалась тахикардия и снижение артериального давления до 80/60 мм рт. ст. Какой вид коллапса развился при этом?
=Инфекционно- токсический
Ортостатический
Геморрагический
Кардиогенный
Панкреатический

У ребёнка, получившего в результате неосторожного обращения с огнём термические ожоги до 40\% поверхности тела, показатель гематокрита выявляет нарушение соотношения плазмы и форменных элементов. Какая форма нарушения общего объёма крови наблюдается при этом?
=Полицитемическая гиповолемия
Полицитемическая гиперволемия
Олигоцитемическая гиповолемия
Нормоцитемическая гиповолемия
Олигоцитемическая гиперволемия

Животное находится в состоянии полного голодания. Основной обмен повышен. Дыхательный коэффициент равен 1, азотистый баланс отрицательный. В каком периоде голодания находится животное?
=Возбуждения
Угнетения
Безразличия
Параличей
Терминального

У больного при гематологическом исследовании получена следующая картина: Эр.-$2,8*10^{12}/л$, Нв.- 80г/л, Ц.п.- 0,85, Ретикулоц.- 0,1\%, Тромбоц.- 160 тыс. в мкл, Лейкоц.- $60*10^9/л$. Б-2, Э-8, Промиелоц.-5, Миелоц.-5, Юн.-16, Пя-20, Ся-34, Л-5, М-5. О какой форме патологии крови
=Хроническом миелоидном лейкозе
Остром миелоидном лейкозе
Гипопластической анемии
Недифференцируемом лейкозе
Гемолитической анемии

У больного, прооперированного по поводу осложненного аппендицита, в анализе крови отмечаются следующие изменения: Эр.- $4,0*10^{12}/л$, Нв - 120г/л, Ц.п.-0,9, Лейкоц. - $18*10^9/л$, Б - 0, Э - 0, Мц - 0, Юн - 0, Пя - 20. Ся - 53, Л -21, М - 5. Как называются такой ядерный сдвиг лейкоцитарной
=Дегенеративный сдвиг влево
Сдвиг вправо
Регенеративный сдвиг влево
Гиперрегенеративный
Регенеративно-дегенеративный

У чоловіка 35 років через 30 хвилин після автомобільної аварії виявлена масивна травма нижніх кінцівок без значної зовнішньої крововтрати. Постраждалий знаходиться у збудженному стані . Який компонент патогенезу травматичного шоку є у пацієнта ведучим і потребує негайного корегування?
=Біль.
Внутрішня крововтрата.
Внутрішня плазмовтрата.
Інтоксікація.
Порушення функції органів.

Жінка 53 років, ріст 163 см., вага тіла 92 кг, рівномірне відткладання жиру, лице одутле, малорухома, апатична. При натискуванні шкіри ноги залишається ямка. Порушенням функції якої залози обумовлений стан хвороби?
=Щитовидної.
Гіпофізу.
Надниркових.
Статевих.
Прищитовидних.

У хворої 36 роківї, яка лікувалась сульфаніламідами з приводу респіраторної вірусної інфекції, в крові гіпорегенераторна нормохромна анемія, лейкопенія, тромбоцитопенія. В кістковому мозку – зменьшення кількості мієлокаріоцитів. Яка це анемія?
=Гіпопластична.
Гемолітична.
Постгеморагічна.
В12 – фолієводефіцитна.
Залізодефіцитна.

В аналізі крові 35-річного хворого: Нв – 58 г/л, еритроцити – 1,3х1012/л, колірний показник – 1,3, лейкоцити – 2,8х109/л, тромбоцити – 1,1х109/л, ретикулоцити – 2\, ШОЕ – 35 мм/час. Визначаються полісегментіровані нейтрофіли, а також тільця Жоллі, кільця Кебота, мегалоцити. Яка це анемія?
=В12 – фолієводефіцитна.
Гіпопластична.
Постгеморагічна.
Гемолітична.
Залізодефіцитна.

При обстеженні хворого встановлено, що кліренс ендогенного креатініну після збору 24-х годинного зразка сечі у нього становить 50 мл/хв (при нормі – 110-150 мл/хв]. Про зниження якої функції свідчить наявність такої ознаки?
=Клубочкової фільтрації
Канальцевої реабсорбції
Інкреторної функції нирок
Виведення з організму іонів.
Виведення з організму сечової кислоти.

Чоловік 30 років скаржиться на задуху, важкість в правій половині грудної клітки, загальну слабкість. Температура тіла 38,9оС. Об’єктивно: права половина грудної клітки відстає від лівої. Плевральна пункція дала ексудат. Що являється провідним чинником ексудації у хворого?
=Підвищення проникливості стінки судин.
Підвищення кров’яного тиску.
Гіпопротеїнемія.
Агрегація еритроцитів.
Зменшення резорбції плевральної рідини .

У чоловіка 48 років, хворого на хронічний бронхіт, діагностована емфізема легень. Що являється провідним чинником даного ускладнення?
=Зниження еластичних властивостей легень.
Зменшення розтяжності легень .
Зменшення загальної течії крові в легенях.
Порушення вентиляційно-перфузійного співвідношення
Зменшення альвеолярної вентиляції.

Хворий чоловік на протязі 17 років страждає хронічним гломерулонефритом. Пульс 82 за хвилину. АТ 190/120 мм рт.ст. Що являється первинним механізмом підвищення артеріального тиску у хворого?
=Підвищення загального периферичного опору.
Збільшення об’єму циркулюючої крові;
Підвищення тонусу венозних судин;
Збільшення ударного об’єму крові;
Збільшення хвилинного об’єму крові;

Піддослідній тварині ввели блокатор цитохромоксидази, що призвело до її миттєвої загибелі. Яка з перелічених речовин може викликати вказані зміни:
=Цианід калію
Нітрит калію
Сульфат калію
Фосфат калію
Оксалат калію

При синдромі реперфузії активуються процеси вільнорадикального окиснення, що призводить до пошкодження клітинних мембран та порушення специфічних функцій клітин. Ці зміни пов’язані з надмірним накопиченням в цитоплазмі іонів:
=Кальцію
Магнію
Хлору
Натрію
Калію

У хворого в мазку крові виявлено: мікроанізоцитоз, пойкілоцитоз, анулоцитоз. Для якої анемії характерні ці зміни?
=Залізодефіцитної
В-12 дефіцитної
Гіпопластичної
Серповидноклітинної
Мікросфероцитарної

У хворого після резекції шлунка розвинулася В-12 фолієводефіцитна анемія. Який із перерахованих кольорових показниівк характерний для цієї патології?
=1,4
1,0
0,8
0,5
0,2

У хворого на ранній стадії цукрового діабету спостерігається поліурія. Чим вона виклікана?
=Гіперглікемією
Кетонемією
Гіпохолестеринемією
Гіперхолестеринемією
Гіперкалійемією

У повної жінки 52 років встановлено цироз печінки. Лабораторно: гіпоальбумінемія, гіперглобулінемія. Візуально: набряк рук, повік, ніг. Найбільш імовірною причиною набряків є
=Онкотичного тиску крові
Буферної емності крові
Кислотно-лужної рівноваги
Дезінтоксикаційної функції печінки
Глікогенсинтезуючої функції печінки

У хворого 35 років розвинулася імунна гемолітична анемія. Який показник сироватки крові зросте в найбільшій мірі?
=Непрямий білірубін
Прямий білірубін
Стеркобіліноген
Мезобіліноген
Протопорфірин

У хворого 48 років спостерігається артеріальна гіпертензія, головний біль, м’язoва слабкість, судоми. В крові знижена концентрація К+ і підвищена концентрація Na+, що є наслідком гіперсекреції:
=Альдостерону
Адреналину
Паратгормону
Кортизолу
Дигідрохолестеролу

У хлопчика 15 років, хворого на алкаптонурію сеча набуває чорного кольору після відстоювання. Спадкове порушення обміну якої речовини має місце:
=Тирозину
Цистеїну
Аланіну
Сечовини
Сечової кислоти

Больная 23 лет жалуется на выраженную слабость, сонливость, потемнение в глазах, головокружение, извращение вкуса. В анализе меноррагии. Объективно: бледность кожных покровов, трещины в углах рта, слоящиеся ногти, увеличение ЧД и ЧСС. Анализ крови: Эр 2,8 х 1012/л , Hb 70 г/л, ЦП 0,75. Какая гипоксия вероятнее всего привела к развитию выявленных симптомов у больной?
=Гемическая
Циркуляторная
Тканевая
Респираторная
Субстратная

У больной 43-х лет, на фоне септического шока отмечается тромбоцитопения, уменьшение фибриногена, появление в крови продуктов деградации фибрина, появление петехиальных кровоизлияний. Укажите причину возникновения данных изменений.
=ДВС-синдром
Аутоиммунная тромбоцитопения
Геморрагический диатез
Нарушение выработки тромбоцитов
Экзогенная интоксикация

У ребенка, родившегося в позднем браке, - малый рост, отставание в умственном развитии, толстый ”географический” язык, узкие глазные щели, плоское лицо с широкими скулами. Что вероятнее всего обусловило развитие описанного
=Наследственная хромосомная патология
Родовая травма
Внутриутробная интоксикация
Внутриутробная инфекция
Внутриутробный иммунный конфликт

При аналізі ЕКГ встановлено: ритм синусовий, правильний, інтервал RR 0,58 сек, розташування і тривалість інших інтервалів, зубців і сегментів не змінені. Назвіть вид аритмії.
=Синусова тахекардія.
Синусова брадикардія
Ідіовентрикулярний ритм
Синусова аритмія
Миготлива аритмія. Миготтлива – мерцательная

В медико-генетичній консультації при обстеженні хворого хлопчика в крові були виявлені нейтрофільні лейкоцити з 1 “барабанною паличкою”. Наявність якого синдрому можлива у
=Синдром Клайнфельтера.
Синдром Дауна
Синдром Шерешевського-Тернера
Синдром Едвардса
Синдром трисомії – Х

У хворого виявлено порушення прохідності дихальних шляхів на рівні дрібних і середніх бронхів. Які порушення кислотно–лужної рівноваги можна виявити у крові в даному
=Респіраторний ацидоз.
Метаболічний ацидоз
Респіраторний алкалоз
Метаболічний алкалоз
--

У неврологічне відділення з приводу мозкового крововиливу поступив хворий, 62 р. Стан важкий. Спостерігається наростання глибини і частоти дихання, а потім його зменшення до апное, після чого цикл дихальних рухів відновлюється. Який тип дихання виник у хворого?
=Чейна–Стокса.
Кусмауля
Біота
Гаспінг–дихання
Апнеїстичне

У чоловіка 52 років через 3 роки після операції видалення шлунку вміст еритроцитів в крові складає 2,0х1012/л, Hb–85г/л, к.п.–1,27. Порушення засвоєння якого вітаміну викликало такї
=B12.
B6
C
P
A

Хворому з закритим переломом плечевої кістки накладена гіпсова пов’язка. Наступного дня з’явилася припухлість, синюшність і похолодання кисті травмованої руки. Про який розлад периферичного кровообігу свідчать ці ознаки?
=Венозна гіперемія
Артеріальна гіперемія
Ішемія
Тромбоз
Емболія

Хворий помер від інфаркту міокарда. Проведене патогістологічне дослідження міокарда виявило значні контрактурні зміни в кардіоміоцитах. Це зумовлено нагромадженням в кардіоміоцитах іонів
=Кальцію
Водню
Натрію
Магнію
Хлору

У дитини із розумовою відсталістю встановлено зелене забарвлення сечі після додавання 5\% розчину $FeCl_3$. Про порушення обміну якої амінокислоти свідчить позитивний результат цієї діагностичної проби?
=Фенілаланіну
Аргініну
Глутаміну
Тирозину
Триптофану

При моделюванні запалення нижньої кінцівки у тварини підвищилася температура тіла, збільшився вміст антитіл та лейкоцитів у крові. Які речовини обумовили розвиток цих загальних реакцій організму при запаленні?
=Інтерлейкіни
Глюкокортикоїди
Мінералокортикоїди
Лейкотриєни
Соматомедіни

При відтворенні артеріальної гіпертензії у собаки через 1 місяць товщина стінки лівого шлуночка зросла в 1.7 рази, а об’єм циркулюючої крові не змінився порівняно з вихідними даними. Яка стадія гіпертрофії міокарда спостерігається у тварини?
=Завершеної гіпертрофії
Аварійна
Початкова
Прогресуючого кардіосклерозу
Декомпенсації

У хворого діагностовано тиротоксикоз. У крові знайдено антитиреоїдні антитіла. Який тип алергічної реакції за Кумбсом і Джелом спостерігається при розвитку цього захворювання?
=Стимулюючий
Анафілактичний
Цитотоксичний
Імунокомплексний
Гіперчутливість сповільненого типу

У жінки 68 років після інсульту відсутні рухи в верхній та нижній правій кінцівках. Тонус м’язів цих кінцівок і рефлекси в них підвищені. Є патологічні рефлекси. Яка це форма параліча?
=Геміплегія
Параплегія
Тетраплегія
Моноплегія
Дисоціація

У жінки з первинним гіперпаратиреоідизмом періодично повторюються напади ниркової коліки. Ультразвукове обстеження показало наявність дрібних каменів в нирках, найбільш імовірною причиною утворення яких є:
=Гіперкальциемія
Гіперфосфатемія
Гіперхолестеринемія
Гіперурікемія
Гіперкаліемія

Через кілька тижнів після народження у дитини почали відмічатися прояви ураження ЦНС, шкіра та волосся посвітлішали. При доливанні до свіжої сечі 5% розчину трихлороцтового заліза з*являється оливково-зелене забарвлення. Який найбільш вірогідний діагноз?
=Фенілкетонурія
Алкаптонурія
Фруктозурія
Тирозиноз
Альбінізм

Підліток 15 років, скаржиться на недостачу повітря, загальну слабкість, серцебиття. ЧСС 130 уд/хв, АТ-100/60 мм рт. ст.на ЕКГ, комплекс QRS нормальної форми та тривалості. Число зубців Р та шлункових комплексів однакове, зубець Т злитий з зубцем Р.Яка аритмія серця спостерігається у підлітка ?
=синусова тахікардія
синусова екстрасистолія
мерехтіння передсердь
тремтіння передсердь
передсердна пароксизмальна тахікардія

В медико-генетичну консультацію звернулася жінка по рекомендації гінеколога з приводу відхилень фізичного і статевого розвитку. При мікроскопії клітин слизової ротової порожнини не знайдено статевого хроматину. Який буде найбільш вірогідний діагноз?
=Синдром Шерешевського-Тернера
Хвороба Дауна
Синдром Клаинфельтера
Хвороба Реклінгаузера
Трисомія по Х-хромосомі

У хворого через 9 діб після введення лікувальної сироватки зўявилася кропивўянка, зуд шкіри, набряк її та слизових оболонок, припухання лімфатичних вузлів. Яке захворювання
=Сироваткова хвороба.
Феномен Швартцмана.
Набряк Квінке.
Феномен Овері.
Поліноз.

У чоловіка 45 років після значного психоемоційного навантаження раптово зўявився стискаючий біль в ділянці серця з ірадіацією в ліву руку, шию, під ліву лопатку. Обличчя стало блідим, вкрилос холодним потом. Нітрогліцерин зняв приступ болю. Який процес розвинувся у хворого?
=Стенокардія
Інфаркт-міокарда
Інсульт
Психогенний шок
Перфорація виразки шлунку

У клініку доставили пацієнта 32 років з масивною крововтратою внаслідок автодорожнї травми. Пульс 110 уд/хв., частота дихання-22 за 2 хв., АТ -100/60 мм рт.ст. Яка зміна крові із перелічених буде найбільш характерною чере 1 годину після
=гіповолемія
еритропенія
гіпохромія еритроцитів
лейкопенія
гіпопротеінемія

У хворого 42 років при дослідженні периферичної крові виявлено: гемоглобін 80 г/л, еритроцитів 3,2 Т/л, лейкоцитів 25 Г/л; лейкоцитарна формула: базофілів - 5%, еозинофілів - 9%, міелобластів - 3%, проміелоцитів - 8%; нейтрофілів: міелоцитів - 11%, метаміелоцитів - 22%, паличкоядерних - 17%, сегментоядерних - 19%, лімфоцитів - 3%, моноцитів - 3%. Яка патологія крові найбільш вірогідна у хворого:
=Хронічний міелолейкоз
Міелобластний лейкоз
Еритроміелоз
Проміелоцитарний лейкоз
Панміелофтіз

У хворого М, 45 років, при аналізі ЕКГ встановлено: ритм синусовий, число передсердних комплексів більше числа шлуночкових комплексів; прогресуюче подовження інтервала P-Q від комплекса до комплекса; випадення окремих шлуночкових комплексів; зубці Р та комплекси QRST без змін. Назвіть тип порушення серцевого ритму.
=Атріовентрикулярна блокада II ступеня
Сіноаурікулярна блокада
Атріовентрикулярна блокада I ступеня
Внутрішньопередсердна блокада
Повна атріовентрикулярна блокада

Людина отримала електротравму. При цьому струм перейшов через серцевий м*яз. Які небезпечні порушення в роботі серця можуть виникнути у цііі ситуаціі, що вимагають термінового втручання?
=Фібриляція шлуночків
Фібриляція передсердь
Екстрасистолія
Тахікардія
Брадікардія

До гастроентерологічного відділення потрапив хворий 57 років з підозрою на синдром Золінгера-Еллісона, про що свідчило різке збільшення рівню гастрину у сироватці крові. Яке порушення секреторної функції шлунку найбільш вірогідне?
=Гіперсекреція гіперацидна
Гіпосекреція гіперацидна
Ахілія
Гіпосекреція гіпоацидна
Гіперсекреція гіпоацидна

У породілі через 24 години після пологів діагностована емболія легеневої артерії. Виникла задишка з частим, глибоким диханням. Артеріальний тиск – 90/40 мм рт.ст., частота серцевих скорочень –99/хв. Газовий аналіз: рН = 7,48; РСО2 = 32 мм рт.ст.; [HCO3] = 20 ммоль/л; [BE] = +1ммоль/л Який з перерахованих нижче розладів кислотно-лужного стану має
=Декомпенсований респіраторний алкалоз
Компенсований респіраторний алкоз
Декомпенсований метаболічний ацидоз
Компенсований метаболічний ацидоз
Респіраторний алкалоз + метаболічний ацидоз

Який із показників вказує на наявність компенсування ацидозу?
=Зниження показника резервної лужності.
Зниження рН.
Підвищення рН.
Нормальна величина рН.
Підвищення показника резервної лужності.

Ступінь патогенності іонізуючого випромінювання визначається насамперед
=енергетичною його характеристикою.
Його видом.
Йонізуючою його здатністю.
Проникливістю.
Експозицією (тривалістю дії).

Який із механізмів патогенної дії підвищеного барометричного тиску лімітує в принципі глибину занурення під воду ?
=Токсична дія газів дихальних сумішей.
Зміни теплових характеристик дихальних сумішей
Механічна дія на стінки повітроносних порожнин
Інтенсивна десатурація газів
Високий гідростатичний тиск води.

Який із перерахованих патологічних процесів відноситься до реакцій негайного типу?
=Поліноз
Аутоалергія.
Контактна алергія
Реакція відторгнення трансплантату.
Бактеріальна алергія.

Що знаходиться в основі механізму формування патологічної детермінанти?
=Формування генератора патологічно посиленого збудження.
Феномен випадіння.
Позамежове гальмування.
Перезбудження.
Парабіоз.

При емфіземі легенів яка із функційних змін за інших рівних умов, тобто сама по собі, має адаптивне значення?
=Зростання дихального об*єму.
Зростання частоти дихання.
Зростання частоти серцевих скорочень.
Зниження частоти серцевих скорочень.
Зростання артеріального тиску.

Який із показників прямо вказує на кардіальну форму недостатності кровообігу?
=Зменшення швидкості зростання тиску в лівому шлуночку в фазу ізометричного скорочення.
Підвищення мінімального артеріального тиску.
Зниження максимального артеріального тиску.
Тахікардія.
Брадікардія.

Який із механізмів забезпечує тонку адаптацію сили скорочення міокарду до коливань артеріального тиску?
=Гомеометричний.
Зміна симпатичного впливу на серце.
Гормонально обумовлена зміна концентрації Na+ і K+ в крові.
Зміна парасимпатичного впливу на серце.
Зміна інтенсивності виділення в кров гормонів Т3 і Т4 (три- і тетрайодтироніну.

Жінка 38 років скаржиться на загальну слабкість, біль в області серця, підвищення апетиту, відсутність менструацій. Об*єктивно: зріст 166 см, вага тіла 108 кг, лице місяцеподібне, відкладення підшкірної клітковини переважно області верхнього плечового поясу, тулубу; на шкірі стегон, живота криваво-червоні смуги, пульс 62 уд. за хв., АТ-160/105 мм. рт. ст. Для якого із перелічених нижче станів найбільш характерний
=хвороба Іценко-Кушинга
аліментарне ожиріння
мікседема
інсулінома
синдром Бабінського-Фреліха

Жінка 26 років, скарги на наявну загальну слабкість, втрату маси тіла на 18 кг, відсутність менструацій, хворіє вже 1 рік, після пологів; пологи були важкі, супроводжувались кровотечею. Об*єктивно: зріст 168 см, вага тіла 53 кг гіпоплазія молочних залоз. Діагностовано синдром Шихена. Що являється основним механізмом втрати ваги у жінки?
=зниження продукції гормонів аденогіпофізу
зниження функції статевих залоз
зниження функції кіркового шару наднирників
гіпотиреоз
гіпопаратиреоз

Чоловік 48 років, страждає на цукровий діабет з 6 років, госпіталізований у непритомному стані, який наступив після підвищеного фізичного навантаження. Об*єктивно: шкіра волога, тонус м*язів кінцівок підвищений сухожильні рефлекси підвищені, дихання поверхневе, пульс 78 уд/хв., АТ-95/60 мм. рт. ст., рівень глюкози у крові 1.88 ммоль/л. Для якого із перелічених станів найбільш характерні симптоми у чоловіка?
=гіпоглікемічної коми
гіперкетонемічної
лактацидемічної коми
гіперосмолярної коми
гіпотонічного стану

При обследовании больного с гемофилией обнаружено изменение некоторых показателей крови. Какой из перечисленных признаков соответствует этому заболеванию?
=Время свертывания крови замедлено
Тромбоцитопения
Эритроцитоз
Эозинофилия
Афибриногенемия

[ФАКТ Яким клітинам належить провідна роль в очищені вогнища запалення від продуктів альтерації ?
=Моноцитам-макрофагам
Нейтрофільним гранулоцитам
Тучним клітинам
Лімфоцитам
Фібробластам

[ФАКТ При дії на організм низької температури підвищується активність механізмів терморегуляції. Які з нижчевказаних механізмів найбільш ефективно обмежують тепловіддачу?
=Спазм судин шкіри
Брадіпноє
Брадікардія
Розширення судин шкіри
Посилення потовідділення

Чоловік 53 роки, доставлений у стаціонар у непритомному стані. Об*єктивно: шкіра суха , дихання часте поверхневе, запах ацетону відсутній, пульс 126 уд/хв., АТ-70/40 мм рт. ст., вміст глюкози у крові 48 ммоль/л, реакція сечі на ацетон негативна. Для якого із перелічених станів найбільш характерні
=гіперосмолярної коми
гіперкетонемічної коми
лактацидемічної коми
Токсичної коми
колапсу

Жінка 22 років, поступила у стаціонар в коматозном стані. Об*єктивно: шкіра суха, бліда, запах ацетону з рота, диханняКуссмауля, пульс 130 уд/хв., АТ-95/60 мм.рт.ст., вміст глюкози у крові - 37.7ммоль/л. Для якого із перелічених станів найбільш характерні симптоми, що спостерігаються ?
=гіперкетонемічної коми
гіперосмолярної коми
лактацидемічної коми
тривалого голодування
алкогольної інтоксикації

Жінка 27 років звернулась зі скаргами на свербіння та печію в очах, сльозотечію, чхання, виділення з носу. Симптоми з*явились після поїздки за місто влітку. Діагностовано поліноз.Якого типу алергічна реакція розвилась при наявному
=анафілактична
цитотоксична
реакція утворення імунних комплексів
сповільненої чутливості
стимулююча

Чоловік 28 років, поступив з скаргами на нудоту, блювоту, біль у правім підребер*ї. Об*єктивно: жовтушність шкіри, склер, температура тіла підвищена , печінка збільшена, сеча темна, кал гіпохолічний, гіпербілірубінемі (білірубін прямий та непрямий), білірубінурія, уробілінурія, гіпопротеінемія, зниження зсідання крові. Для якого з нижче перелічених станів найбільш характерні ці зміни?
=клітинно-паренхіматозної жовтяниці
підпечінкової жовтяниці
надпечінкової гемолітичної жовтяниці
гострого холециститу
гострого панкреатиту

Жінка 55 років, скаржиться на швидку втомлюваність, роздратованість, безсоння та сонливий стан в день, свербіння шкіри. Пульс 58 уд/хв., АТ-110/65 мм рт.ст. Кал безколірний, містить багато жиру. Підозрюється жовчнокам*яна хвороба з закупорюванням каменем жовчної протоки.Що з перелікованого нижче найбільш вірогідно обумовлює стан хворої?
=холемія
гіпербілірубінемія
гіперхолестеринемія
порушення всмоктування жиророзчинних вітамінів
порушення всмоктування жирів

Чоловік 32 років , звернувся з приводу диспептичних розладів, при обстеженні знайдено виразковий дефект слизової оболонки шлунка та діагностується синдром Золінгера-Еллісона. Що являється основним патогенетични механізмом виникнення виразки у даному випадку ?
=підвищення продукції гастрину
підвищення продукції соляної кислоти
зниження захисних властивостей слизової шлунка
підвищення продукції інсуліну
рефлюкс дуоденального вмісту у шлунок

Чоловік 38 років поступив у терапевтичне відділення з діагнозом: правосторонній ексудативний плевріт. Відкачана з плевральної порожнини грудної клітки рідина прозора, має відносну щільність 1.020; містить 55 г\л, білка альбуміно-глобуліновий коефіцієнт-1.6; загальна кількість клітин-2.8. в 1 мкл;pH-6.5.Який тип ексудату має місце у
=серозний
фібринозний
гнійний
гнилісний
геморагічний

Чоловік 65 років страждає хронічною серцевою недостатністю по лівошлуночковому типу. Об*єктивно: ціаноз, задишка, кашель з харкотинням, періодичні напади задухи. Який тип гіпоксії первинно виник у хворого ?
=циркуляторна застійна
циркуляторна ішемічна
тканинна
дихальна
кров*яна

Чоловік 36 років скаржиться на кашель з виділенням харкотиння, задишку, головний біль, загальну слабкість. Захворів після сильного переохолодження. При огляді: шкіра бліда, температура тіла 38 С. Пульс-91 в 1 хв.,АТ-125/6 мм рт.ст. В аналізі крові - нейтрофільний лейкоцитоз. Поставлений -діагноз: вогнищева пневмонія. Який тип гіпоксії має місце у хворого?
=дихальна
гемічна
тканинна
циркуляторна застійна
циркуляторна ішемічна

Жінку 50 років турбують часті головні болі, запаморочення, задишка при фізичному навантаженні. Останні три роки відзначались тривалі та значні менструації. При огляді- нормальної вгодованості, шкіра бліда суха. Аналіз крові: Hb-90г/л, еритроцити-3.7Т/л, кольоровий показник-0.7,ШОЕ-20 мм/год, значна гіпохромія еритроцитів, анізоцітоз, пойкілоцитоз. Яка анемія найбільш вірогідно має місце у хворого?
=геморагічна анемія
гемолітична анемія
метапластична анемія
В12 -фолієводефіцитна анемія
гіпопластична анемія

Чоловік 47 років скаржиться на слабкість , запаморочення . Півроку назад переніс операцію з приводу резекції шлунку Аналіз крові:Hb-80г/л, еритроцити-3.5Т/л, кольоровий показник-0.69, ШОЕ-15 мм/год. Сироваткове залізо-5.4 мкмоль/л. Гіпохромія еритроцитів. Яка анемія найбільш вірогідно має місце
=залізодефіцитна анемія
сидероахрестична анемія
геморагічна анемія
гемолітична анемія
В12 -фолієводефіцитна анемія

Жінка 48 років доставлена у лікарню в тяжкому стан: температура тіла 39.9 С, кашель з гнійно-кров*янистою мокротою, ціаноз, пульс 116 уд.за хв., ЧД-38 за хв., в легенях з обох боків- ділянки дрібнопузирчатих вологих хрипів Діагностована гостра двобічна нижньодольова стафілококова пневмонія. Яка форма дихальної недостатності має місце у
=бронхолегенева рестриктивна
обструктивна
нервово-м*язова
торакодіафрагмальна
центральна

Жінка 22років . Діагностований ревматичний міокардит. Чим обумовлене порушення кровообігу?
=пошкодженням міокарду
зниженням об*єму циркуючої крові
перевантаженням міокарду збільшеним об*ємом крові
перевантаженням міокарду підвищеним опором відтоку крові
збільшенням об*єму судинного русла

Діагностований стеноз аортального отвору.Чим обумовлене порушення кровообігу?
=перевантаженням міокарду підвищеним опором відтоку крові
Пошкодженням міокарду
зниженням об*єму циркуючої крові
перевантаженням міокарду збільшеним об*ємом крові
збільшенням об*єму судинного русла

Чоловік 60 років страждає атеросклерозом судин.Яка з приведених нижче сполук грає провідну роль в патогенезі даного захворювання?
=ліпопротеїди низької щільності
ліпопротеїди високої щільності
хіломікрони
тканинний фермент ліпопротеїнкіназа
комплекс жирних кислот з альбумінами

Вказати розвиток яких хвороб може бути, патогенетично, безпосередньо пов`язаний з неврозом:
=Виразка шлунка
Дифузний гломерулонефрит
Хвороба Іценко-Кушинга
Гепатит
Анемія

Вказати коли має місце гетерометричний механізм компенсації в лівому шлуночку:
=Недостаності аортального клапана
Стенозі аортального клапана
Недостатності клапана легеневої артерії
Стенозі правого атріовентрикулярного отвору
Стенозі мітрального клапана

Після перенесеного важкого інфекційного процесу у хворого розвинувся міокардит з ушкодженням провідної системи серця, що супроводжувалось періодичною втратою свідомості внаслідок розвитку синдрому Моргані Едемса-Стокса. Яке порушення виникло у хворого?
=Перехід неповної атріовентрикулярної блокади в повну
Блокада лівої ножки пучка Гіса
Блокада правої ножки пучка Гіса
Пароксизмальна тахікардія
Передчасне збудження шлуночків

Вкажіть “черговість” еміграції різних видів лейкоцитів в зону гострого гнійного запалення:
=Нейтрофіли, моноцити, лімфоцити
Лімфоцити, нейтрофіли, моноцити
Моноцити, нейтрофіли, лімфоцити
Моноцити, лімфоцити, нейтрофіли
Нейтрофіли, лімфоцити, моноцити

Відповідно до фізико-хімічної теорії Шаде в зоні запалення має місце: гіперосмія, гіперонкія, ацидоз, набряк. Розвиток гіперосмії, в деякій мірі, пов*язаний із зростанням концентрації К+ в зоні запалення. Вказати причини гіперкалійіонії в запальному
=Інтенсивна деструкція пошкоджених клітин
Збільшення проникненості судинної стінки
Активація проліферативних процесів
Пригнічення глікогеноліза в зоні запалення
Надлишок іонів Са++

У жінки віком 55 років відмічалась виражена хронічна крововтрата менструальної крові. При даній анемії встановлен мікроцитоз та гіпохромія еритроцитів. Як вона називається:
=Залізодефіцитна
В-12-фолієводефіцитна
Гостра постгеморагічна
Серповидноклітинна
Таласемія

При загальному огляді хворого встановлена гіперемія всіх шкірних покровів з цианотичним відтінком. Звертає на себе увагу заторможеність хворого та сповільнення його рухів. Аналіз крові показав: еритроцити 9 х 10в12/л гематокріт 60%. При якому патологічному стані має місце абсолютний
=Хвороба Вакеза
Мегалобластична анемія
Лімфома
Гемодилюція
Гемоконцентрація

У хворого на гострий мієлобластний лейкоз виявлено: збільшення печінки, селезінки, анемія, мієлобласти в периферичній крові. Вказати яка принципова ознака відрізняє гострий мієлобластний від хронічного:
=Лейкемічний провал
Бластні клітини в периферичній крові
Анемія
Панцитопенія
Тромбоцитопенія

У ребенка, страдающего пилоростенозом, сопровождающимся частыми рвотами, развились признаки обезвоживания организма. Какая форма нарушения кислотно-основного состояния может развиться в данном случае?
=Негазовый алкалоз
Негазовый ацидоз
Газовый ацидоз
Газовый алкалоз
Метаболический ацидоз

В результате производственной травмы у больного обнаружен перелом нескольких ребер. Какой характер дыхания обычно наблюдается в таких случаях?
=Полипноэ
Эупноэ
Гиперпноэ
Брадипноэ
Апноэ

У больного диабетом в периоде развития коматозного состояния наблюдается шумное и глубокое дыхание К какому типу нарушения легочной вентиляции относится данное
=Агональное дыхание
Гиперпноэ
Тахипноэ
Периодическое дыхание
Полипноэ

У больного после автомобильной травмы артериальное давление 70/40 мм рт.ст. Больной в бессознательном состоянии. В сутки выделяет около 550 мл мочи. Периодически возникают судороги, дыхание по типу Куссмауля. Как называется такое нарушение функции почек? Больной в бессознательном состоянии. В сутки выделяет около 550 мл мочи. Периодически возникают судороги, дыхание по типу Куссмауля. Как называется такое нарушение функции почек?
=Острая почечная недостаточность
Острый диффузный гломерулонефрит
Тубулопатия
Хроническая почечная недостаточность
Пиелонефрит

Ребенок 10-ти лет перенес несколько атак ревматизма.При его клиническом обследовании было установлено, что имели место воспалительные явления в суставах и обнаружились признаки недостаточности митральног клапана.Какое из патологических явлений у данного больного можно отнести к
=Ревматизм
Артрит
Порок митрального клапана
Воспаление суставов
Недостаточность митрального клапана

У больного с признаками недостаточности митрального клапана в намнезе отмечались атаки ревматизма, сопровождавшиеся воспалительными явлениями в суставах. Какое из патологических явлений у данного больног относится к категории *патологического состояния*?
=Недостаточность митрального клапана
Ревматизм
Артрит
Воспаление суставов
Ревмокардит

У больного наблюдается синдром ДиДжорджи, в основе которого лежит гипоплазия вилочковой железы К какой форме иммунной патологии относится это заболевание?
=Врожденному дефициту Т-лимфоцитов
Врожденному дефициту В- лимфоцитов
Приобретенному дефициту В- лимфоцитов
Приобретенному дефициту Т-лимфоцитов
Иммунодепрессии в системе Т- лимфоцитов

При голодании (во втором его периоде) у больного может повышаться количество липидов в крови и наблюдаться гипопротеинемия. Какая форма гиперлипемии имеет место в данном случае?
=Транспортная
Ретенционная
Алиментарная
Пищевая
Перераспределительная

У больного при обследовании обнаружен гирсутизм, *лунообразное лицо*, полосы растяжения на коже живота. Артериальное давление 190/100 мм рт.ст., глюкоза крови - 17,6 ммоль/л. При каком из указанных видов патологии встречается подобная картина?
=Гиперфункции коры надпочечников
Гипертиреозе
Гипотиреозе
Гипофункции половых желез
Гиперфункции инсулярного аппарата

У больного определяется наличие гипергликемии, полиурии, гиперстенурии и глюкозурии Для какой нозологической формы патологии обмена характерно такое сочетание этих
=Сахарного диабета
Почечного диабета
Несахарного диабета
Гликогеноза
Диэнцефального ожирения

В периоде разгара острой лучевой болезни у больного наблюдались лейкопения, тромбоцитопения, аутоинфекция, аутоинтоксикация, кровоточивость, повышение температуры тела Для какой формы лучевой болезни характерна данная
=Костно-мозговой
Кишечной
Токсемической
Церебральной
Геморрагической

У больного через сутки после травмы распух коленный сустав. При его пункции получено 30 мл жидкости розового цвета с удельной плотностью 1020. Общее содержание белка в ней - 3%, альбуминов - 0,3%, глобулинов 2%, фибриногена - 0,7%. Лейкоцитов - 1-3, эритроцитов - 15-20, местами до 50 в поле зрения. Какого характера эксудат получен при пункции коленного сустава у больного?
=Геморрагический
Серозный
Гнойный
Гнилостный
Фибринозний

У больного днем внезапно поднялась температура до 39, 5 0С и через 6 часов вернулась к норме. На вторые сутки приступ повторился и температура достигла 41, 5 0С, период апирексии наступил через 8 часов. Какой тип температурной
=Перемежающий
Послабляющий
Септический
Изнуряющий
Постоянный

У больного во время приступа бронхиальной астмы при определении рСО2 в крови выявлено наличие гиперкапнии, при определении РО2 гипоксемии Какой вид гипоксии наблюдается в данном случае?
=Гипоксическая
Гемическая
Циркуляторная
Дыхательная
Гистотоксическая

Анализ прозрачной жидкости лимонно-желтого цвета, полученной у больного при пункции брюшной полости, дал следующий результат:относительная плотность - 1012, альбумина - 1%, глобулины - 0,2%, фибриногена нет эритроциты единичные, лейкоциты 1-3 в поле зрения. Жидкость стерильна, при стоянии в течение часа не свернулась. Какое из перечисленных явлений сопровождается такими
=Асцит вследствие застоя крови в портальной системе
Воспаление брюшины
Отечная форма гемолитической болезни новорожденных
Перитонит
Эмпиема брюшины

У больного желтухой при лабораторном исследовании получены следующие данные, характеризующие нарушение пигментного обмена: общее содержание в сыворотке крови билирубина - 80 мкмоль/л; содержание прямого билирубина в сыворотке крови -63 мкмоль/л; в моче присутствует билирубин; уробилиноген и стеркобилин в моче отсутствует. Какова причина отсутствия в моче уробилиногена при данном
=Отсутствие поступления в кишечник билирубина
Нарушение всасывания уробилина в кишечнике
Нарушение выделения уробилина почками
Нарушение связывания уробилина транспортным белком
Дефицит ферментов,превращающих глюкоронидбилирубин в уробилиноген

У лихорадящего больного наблюдается побледнение кожных покровов, *гусиная кожа*, озноб, тахикардия Какой стадии лихорадки соответствует данное состояние?
=Стадии подъема температуры
Стадии стояния температуры
Стадии падения температуры



У длительно лихорадящего больного утренняя температура тела была в пределах 36,4-36,9 0С. К вечеру она поднималась до 37,0-38,0 0С, в некоторые дни до 38,8 0С. Больной температурит более 2-х месяцев. Какой тип лихорадки
=Постоянный
Послаблящий
Изнуряющий
Волнообразный
Гектический

В анамнезі: з дитинства відмічався знижений рівень гемоглобіна. Лікування препаратами заліза без эффекту При вступі в аналізі крові: кількість еритроцитів 3,1*1012/л, ретикулоцитів 16%, гемоглобіну 85 г/л, КП 0,75; в мазку крові анізоцити, пойкілоцити мішеневидні еритроцити, еритроцити з базофільною зернистістю; рівень заліза у сировотки 30 мкмоль/л. Для якої патології системи крові характерні дані?
=Талассемії
Залізодефіцитні анемії
В12-дефіцитні анемії
Фоліеводефіцитні анемії
Гіпопластична анемія

Хворий (28 років) прибув у стаціонар зі скаргами на біль у животі, нестійкий стул, слабкість,втому,віддишку. В анамнезі: 2 роки тому операція з приводу гострої кишкової непрохідності з резекцією 60 см тонкої кишки. При вступі в аналізі крові: кількість еритроцитів 2,4*10в12/л, ретикулоцитів 0,4%, гемоглобіну 80 г/л, КП 1,25; в мазку крові макроанізоцити, пойкілоцити, шизоцити поодинокі мегалоцити, мегалобласти. Для якої патології системи крові характерні ці дані?
=В12-дефіцитна анемія
Залізодефіцитна анемія
Гіпопластична анемія
Гемолітична анемія
Хронична постгеморагична анемія

Рабітника АЕС доставили у клініку після одноразового опромінення зі скаргами на слабкість, головний біль, підвищення температури, діарею. В аналізі крові – лейкоцитоз з лімфопеніею. Яка стадія променевої хвороб найбільш ймрвірна у
=Період первинних реакцій
Період вдаваного мнимого благополуччя
Період разгорнутої кліничної картини
Латентний період
Продромальний період

Хворий Н. поступив зі скаргами на диспептичні розлади, мелену, гемороїдальні кровотечі. При дослідженні виявлено розширення сітки венозних судин на переднеій стінці животу в комплексі зі збільшенням його размірів. Як патологія ШКТ може виявитись такими симптомами?
=Портальная гіпертензія
Кишкова аутоінтоксикація
Виразкова хвороба
Коліт
Энтерит

У хворого А. після травматичного шоку развинулись ознаки ниркової недостатності. Якими патогенетичними механізмами зумовлено цей стан?
=Зниженням обўему клубочкової фільтрації
Обтурацією канальців нирок
Пригніченням екскреції сечі в канальцях
Блокування відтоку сечі
Пошкодження клубочкового апарату нирок

При обстеженні дитини з олігофренією виявлено в крові і спинномозковій рідині підвищений вміст фенілаланіну і фенілпіровиноградної кислоти. Реакція сечі з трихлороцтовим залізом позитивна. Виберіть – із порушення синтезу якого ферменту пов*язаний розвиток захворювання.
=Фенілаланінгідроксилаза.
Оксидаза п-гідроксифенілпіровиноградної кислоти.
Тирозиназа.
Дофамінгідроксилаза.
Оксидаза гомогентизинової кислоти.

У хворого 30 років діагностована ішемічна хвороба серця. Приступи стенокардії турбують вже 3 роки. Виявлений спадковий характер захворювання. Який вид гіперліпопротеїдемії найбільш імовірно буде виявлений у
=ІІ А (гіпербеталіпопротеїдемія).
І (гіперхіломікронемія).
ІІІ (дисбеталіпопротеїдемія).
IV (гіперпребеталіпопротеїдемія).
V (гіперпребеталіпопротеїдемія і гіперхіломікронемія).

У альпініста під час сходження на висоті 6 тис. м над рівнем моря виникла ейфорія, неадекватна оцінка обстановки, спостерігались галюцинації. Яка головна причина у розвитку цих ознак гірської хвороби?
=Пониження парціального тиску кисню у повітрі.
Фізичне навантаження.
Пониження атмосферного тиску
Снігова офтальмія.
Розширення повітря в лобних пазухах.

У жінки на 6-му місяці вагітності при обстеженні виявлено знижену кількість еритроцитів і гемоглобіну, кольоровий показник – 1,4. В мазку з*явилися мегалоцити, поодинокі оксифільні мегалобласти. Який вид анемії за патогенезом найбільш імовірний в даному випадку?
=В12 і фолієводефіцитна анемія.
Залізодефіцитна анемія.
Мієлотоксична анемія.
Апластична анемія.
Метапластична анемія.

У жінки, що хворіє на міастенію, виникли розлади дихання, що вимагали застосування штучної вентиляції легень. Який вид недостатності розвинувся у даної хворої?
=Нервово-м*язовий.
Центрогенний.
Торакодіафрагмальний.
Обструктивний.
Рестриктивний.

У хворого висока механічна кишкова непрохідність (заворот). Скарги на різкий біль в животі, блювоту, під час якої втрачає багато рідини. Об*єктивно: ЧСС 110 уд/хв, АТ 90/50, гематокрит 0,52, гіпонатріемія. Який найбільш імовірний провідник механізм патогенезу?
=Дегідратація, гіповолемія.
Негазовий ацидоз.
Розтягнення кишки, біль.
Кишкова аутоінтоксикація.
Утворення великої кількості кінінів.

После двухстороннего удаления надпочечников у собаки появилась мышечная слабость, адинамия, снижение температуры тела, гипогликемия.Какие еще могут быть отмечены проявления надпочечниковой недостаточности?
=Артериальная гипотония
Лимфопения
Усиление синтеза гликогена
Увеличение концентрации натрия и хлоридов в сыворотке крови
Повышение резистентности к действию бактерий и токсинов

Во время обеда ребенок поперхнулся и аспирировал пищу. Начался сильный кашель, кожа и слизистые цианотичны, пульс учащен, дыхание редкое, Выдох удлинен Какое нарушение внешнего дыхания развилось у ребенка?
=Стадия экспираторной одышки при асфиксии
Стадия инспираторной одышки при асфиксии
Стенотическое дыхание
Альтернирующее дыхание
Дыхание Биота

У беременной женщины развился токсикоз с тяжелыми повторными рвотами на протяжении суток. К концу суток начали проявляться тетанические судороги и обезвоживание организма. Какой сдвиг КЩ состояния вызвал описанные
=Выделительный алкалоз
Газовый алкалоз
Газовый ацидоз
Метаболический ацидоз
Выделительный ацидоз

У больного отмечаются боли в эпигастральной области, больше слева, с иррадиацией в левую лопатку. Возникновение болей нередко связано с приемом жирной обильной пищи. Боли нередко сопровождаются диспептическим расстройствами: плохим аппетитом, отвращением к пище, тошнотой, рвотой, метеоризмом. Стул обильный /полифекалии/, жирный /стеаторея/, нередко поносы. Какое заболевание можно
=Хронический панкреатит
Язвенная болезнь 12-ти перстной кишки
Энтероколит
Хроническое пищевое отравление
Хронический гепатит

При обследовании больного установленно, что артериальное давление составляет 190/100 мм.рт.ст. Какой механизм ведет к повышению артериального давления ?
=Спазм резистивных сосудов
Увеличение венозного давления
Тоногенная дилятация сердечной мышцы
Миогенная дилятация сердечной мыщцы
Снижение объема циркулирующей крови

У ребенка 1,5 лет переодические приступы слабости натощак, периодические потери сознания, клонические судороги. Ребенок маленького роста с худыми конечностями и большим животом, резко увеличена печень Пункционная биопсия выявляет значительное накопление гликогена в гепатоцитах, то же и в лейкоцитах. Что лежит в основе заболевания?
=Дефект печеночной глюкозо-6 фосфатазы
Дефект мальтазы
Дефект амило-1,6-глюкозидазы
Дефект фосфолипазы
Дефект гексогеназы

[факт] У хворого діагностована бронхіальна астма алергійної природи. Як провести специфічну гіпосенсибілізуючу терапію ?
=малими дозами специфічного антигену
антигістамінними препаратами
малими дозами адреналіну
глюкокортикоїдами
Мінералокортикої

У хворого з гіперфункцією щитовидної залози підвищена температура тіла. Яке порушення енергетичного обміну є головним у підвищенні температури при цьому?
=Роз*єднання окиснення та окисного фосфорування.
Збільшення розпаду глікогену.
Посилення ліполізу.
Активація ферментів в циклі Кребса.
Активація ферментів дихального ланцюга.

Яка ознака найбільш вірогідно характерізує наявність у хворого швейцарського типу імунодефіциту?
=Дефіцит Т- і В- лімфоцитів.
Аутосомно-рецисивне успадкування.
Лімфопенія.
Зменшення плазматичних клітин.
Гіпоплазія вилочкової залози.

Який метод генетичного обстеження найбільш вірогідно встановить діагноз синдрому Шерешевського - Тернера / Тернера / у хворої людини?
=Виявлення статевого хроматину.
Підрахунок *барабанних паличок*.
Генеалогічний.
Близнюковий.
Дерматогліфіка.

[факт] Який гематологичний показ найбільш ймовірний при діагностиці В12 - фолієводефіцитна анемія?
=Поява мегалобластів.
Різке зниження кількості еритроцитів.
Зниження вмісту гемоглобіну.
Пойкілоцитоз.
Зменшення кількості ретикулоцитів.

У дитини 3 років, яка страждає на квашіоркор, спостерігається порушення ороговіння епідермісу та збільшення його злущення, є жирова інфільтрація печінки. Який тип голодування спостерігається пр цьому?
=Білкове.
Вуглеводне.
Мінеральне
Енергетичне.
Жирове.

Яка експериментальна модель гіпертензії найбільш відповідає сучасним уявленням про патогенез есенціальної гіпертензії?
=Тварини із спадковою гіпертензією.
Ренопривна.
Рефлексогенна.
Реноваскулярна.
Кортикоїдна.

У хворого з недостатністю мітрального клапану виникла гіпертрофія лівого шлуночка серця. Який механізм є пусковим у розвитку гіпертрофії?
=Активація генетичного апарату.
Збільшення споживання жирних кислот.
Збільшення інтенсивності клітинного дихання.
Активація гліколізу.
Збільшення надходження Ca2+ в клітину.

[Факт] Хворому на туберкульоз зроблений аналіз крові. Які зміни лейкоцитів найбільш характерні саме для туберкульозної
=Лімфоцитоз.
Еозінофілія.
Нейтрофільоз.
Ядерний зсув лейкоцитарної формули вправо.
Ядерний зсув лейкоцитарної формули вліво.

Хворому на лейкоз зроблений аналіз крові. Що найвірогідніше вказує на наявність гострого мієлобластного лейкозу?
=Наявність лейкемічного провалу.
Лейкоцитоз.
Поява в крові мієлобластів.
Анемія.
Дегенерація лейкоцитів.

У кроля після опромінення спостерігається III період кістково-мозкової форми гострої променевої хвороби. Ураження якої тканини є провідним у патогенезі розладів при цьому?
=Кровотворної.
Кісткової.
Нервової.
Епітелію статевих залоз.
Залозистого епітелію.

У хворого спостерігається експіраторний тип задишки. Яка патологія найбільш ймовірно спричинила таке порушення
=Емфізема
Пневмоторакс.
Недостатність кровообігу.
Загальне збудження центральної нервової системи.
Перша стадія асфіксії.

[Факт] У хворого виникла передсердно-шлуночкова блокада I ступеня. Які зміни ЕКГ можуть при цьому спостерігатися?
=Продовження інтервалу PQ.
Зниження висоти зубця R.
Розширення комплексу QRS.
Випадіння кожного 8-10 комплексу QRS.
Розвиток ідіовентрикулярного ритму.

[факт У дорослої людини найбільшу резистентність до іонізуючого опромінення виявляє
=Головний мозок
Епітелій шкіри
Серезінка
Кістковий мозок
Епітелій травного тракту

[факт] Головними проліферуючими клітинами, які формують грануляційну тканину в зоні запалення, є
=Фібробласти і ендотеліоцити капілярів
Лімфоцити і фібробласти
Ендотеліоцити капілярів і лаброцити
Лейкоцити і ендотеліоцити капілярів
Гістіоцити і фібробласти

[факт] Клітинний склад ексудату у вогнищі гострого запалення залежить переважно від
=Природи флогогенного фактора
Ступеня проникливості капілярної сітки
Концентрації гістаміну в крові
Рівня гарячки
Рівня мікроциркуляції

[факт] Синдрому набутого імунодефіциту вважається пошкодження ВІЛ
=Т-хелперів
Т-супресорів
В-лімфоцитів
Макрофагів
Нейронів головного мозку

[Факт] Мішенеподібні еритроцити властиві для
=Таласемії
Анемії Мінковського-Шофара
В12-дефіцитні анемії
Залізодефіцитні анемії
Серповидноклітинні анемії

[факт] У хворого 55 років діагностоване гіпертонічне хвороба. Який механізм є провідним в патогенезі гіпертонічної хвороби 7
=Підвищення периферичного опору судин
Почащення серцевих скорочень
Збільшення хвилинного об’єму крові
Збільшення маси циркулюючої крові
Веноконстрикція

[факт] Алкаптонурія виникає внаслідок блокади метаболізму тирозину на етапі
=Перетворення гомогентизинової кислоти на малеїлацетооцтову
Перетворення гідроксифенілпіровиноградної кислоти на гомогентизинову
Перетворення малеїлацетооцтової кислоти на фумарилацетооцтову
Перетворення тирозину в меланін
Йодування тирозину вільним йодом

[факт] Первинний патогенетичний механізм пошкодження клітин іонізуючими променями пов’язаний з
=Радіолізом води
Активацією фосфоліпази клітинних мембран
Нагромадженням Са++ в клітині
Посиленою дифузією Na+ в клітину
Внутрішньоклітинним ацидозом

[факт] У хворих з тиреотоксикозом підвищення температури тіла виникає у зв’язку з
=Підвищенням активності біологічного окіслення, роз’єднанням процесів окислення і фосфорилювання
Збільшенням поглинання кисню організмом
Посиленням теплоутворення в печінці
Звуженням периферичних судин
Активацією катаболізму білків

З метою відтворення серцевої недостатності серце жаби перфузували розчином бромістого кадмію - блокатору сульфгідрильних груп. Який варіант серцевої недостатності при цьому виникає?
=При пошкодженні міокарду внаслідок дефіциту енергії
Змішана форма
Від перевантаження
При пошкодженні міокарду, що спричинене порушенням вінцевого кровообігу
При пошкодженні міокарду, що спричинена інтоксікацією

У хворого відмічені такі зміни: порушення зору в сутінках, підсихання кон*юнктиви та рогової оболонки. Такі порушення можуть бути при недостачі вітаміну :

В
С
D
В12

У 40-річної вагітної проведено амніоцентез. При дослідженні картіотипу плоду одержано результат: 47,ХУ+21. Яку патологію плоду виявлено?
=Синдром Дауна
Синдром Клайнфельтера
Хвороба Шерешевського-Тернера
Фенілкетонурія
Хвороба Патау

Завдяки піддослідна миша не загине, якщо їй ввести смертельну дозу стовбнячного токсину в порожнину скипидарного
=Бар*єру з фібробластів, колагену, глікозаміногліканів.
Активації синтезу антитіл.
Стимуляції лейкопоезу
Посиленій васкулярізації
Бар*єру з фібробластів, еозинофілів та тканинних базофілів.

У хворого плевритом в плевральній порожнині виявлена смердюча рідина, яка містить в собі біогенні аміни, гази. Який різновид запалення в даному випадку?
=Гниляве
Альтеративне.
Катаральне.
Гнійне.
Фібрінозне.

Під час гри у волейбол спротсмен після стрибка приземлився на зовнішній край стопи. Виник гострий біль в гомілкостопному суглобі, активні рухи в ньому обмежені, пасивні - в повному обсязі, але болісні. Потім розвинулась припухлість в ділянці зовнішньої кісточки, шкіра почервоніла, стала теплішою на дотик. Який вид розладу периферичного кровообігу розвинувся
=Артеріальна гіперемія.
Стаз
Емболія
Венозна гіперемія.
Тромбоз.

При роботі по ліквідації наслідків аварії на АЕС робітник одержав дозу опромінення 500 рентген. Скаржиться на головний біль, нудоту, запаморочення. Які зміни в кількості лейкоцитів можна чекати у хворого через 10 годин після опромінення?
=Нейтрофільний лейкоцитоз.
Лімфоцитоз.
Лейкопенія.
Агранулоцитоз.
Лейкемія.

Хворий А., 18 років, після перенесеної краснухи почав худнути, постійно відчував сухість в роті, спрагу, у нього підвищився апетит, почалось часте сечовиділення. Обўєктивно: добова кількість сечі 6 л, глюкоза крові 17, ммоль/л, в сечі виявлена глюкоза та ацетон. Яке захворювання виникло у хворого?
=ІЗЦД (інсулінзалежний цукровий діабет).
Симптоматичний цукровий діабет.
Вторинний цукровий діабет.
ІНЦД (інсуліннезалежний цукровий діабет).
Стероїдний діабет.

Хворого доставили в клініку в коматозному стані. Дихання шумне, глибоке. Чути запах ацетону. Вміст глюкози в крові 15,2 ммоль/л, кетонові тіла - 100 мкмоль/л. Для якого виду коми характерні такі розлади?
=Кетоацидотичної
Печінкової
Гіперглікемічної
Гіпоглікемічної
Гіперосмолярної

У хворого 42 років виявлені такі зміни в периферичній крові: гемоглобін 80 г/л, еритроцитів 3,2*10в12/л, лейкоцитів 250*10 в 9/л; лейкоцитарна формула: базофілів - 5%, еозинофілів - 9%, мієлобластів - 3%, промієлоцитов - 8%, нейтрофілів: мієлоцитів - 11%, метамієлоцитів - 22%, паличкоядерних - 17%, сегментоядерних - 19%, лімфоцитів - 3%, моноцитів - 3%. Назвіть найбільш вірогідний діагноз.
=Хронічний мієлолейкоз.
Гострий мієлобластний лейкоз.
Еритромієлоз.
Нейтрофільний лейкоцитоз.
Еозинофільний лейкоцитоз.

При аналізі ЕКГ встановлено: ритм синусовий, число передсердних комплексів більше числа шлуночкових комплексів; прогресуюче подовження інтервалу P-Q від комплексу до комплексу; випадання окремих шлуночкових комплексів, тому після зубця Р йде довга пауза; зубці Р та комплекси QRST без змін. Назвіть тип порушення серцевого ритму.
=Неповна атріо-вентрикулярна блокада П ступеня
Сино-атріальна блокада.
Неповна атріо-вентрикулярна блокада Iступеня
Неповна атріо-вентрикулярна блокада IIIступеня
Повна атріо-вентрикулярна блокада.

Хворий скаржиться на відрижку, печію, часті запори. При титруванні шлункового соку одержали такі дані: загальна кислотність - 88 т.о., загальна HCl - 83 т.о., вільна HCl - 50 т.о., звўязана HCl - 33 т.о., кислі фосфати та органічн кислоти - 5 т.о. Оцініть стан кислотності шлункового соку.
=Гіперацидний.
Гіпоацидний.
Ахілія.
Нормацидний.
Гіперсекреція.

Хворий скаржиться на біль в правому підребірўї, зуд шкіри, головний біль, дратливість, швидку стомлюваність. При обстеженні встановлено: жовтяничний колір шкіри та слизових оболонок, печінка збільшена, болісна пр пальпації. АТ - 80/40 мм рт.ст., ЧСС - 46 за хви-лину. В крові виявлено: вільний біліру-бін - 34,15 мкмоль/л, звўязаний - 35,2 мкмоль/л, жовчні кислоти; в сечі - жовч-ні кислоти, звўязаний білірубін, уробіліноген; вміст стеркобіліногену в калі зменшений. Який вид
=Печінково-клітинна.
Гемолітична.
Підпечінкова.
Обтураційна.
Транспортне

Після крововиливу в мозок у хворого стали неможливими активні рухи лівої руки і ноги. Тонус м*язів цих кінцівок збільшений, їх спинальні рефлекси різко посилені, розширені зони рефлексів. Позитивний рефлекс Бабінського Назвіть вид розладу нервової системи у хворого.
=Центральний параліч.
Периферичний параліч.
Спинальний шок.
Децеребраційна ригідність
Рефлекторний параліч.

До лікаря звернувся чоловік 65 років з скаргами на гострий біль в великих пальцях ніг. Він любить та часто вживає пиво. Виникло підозріння на подагру. Для підтвердження діагнозу вміст якої із перелічених речовин необхідно визначити у крові?
=сечової кислоти
сечовини
лактату
білірубіну
кетонових тіл

У клініку доставлений чоловік 30 років, з профузним поносом тривалістю 12 годин. Блювоти не було. Які зміни водно-електролітного балансу та кислотно-основної рівноваги спостерігаються у чоловіка?
=негазовий ацидоз з дегідратацією
газовий ацидоз з дегідратацією
газовий алкалоз з дегідратацією
без змін pH крові
негазовий алкалоз з дегідратацією

Чоловік 64 років скаржиться на задишку, часте серцебиття, швидку втомлюваність. Ввечері з*являються набряки на нижчих кінцівках. Що із нижче переліченого являється патогенетичним фактором цих набряків?
=підвищення гідростатичного тиску крові у венозних частинах капілярів
зниження онкотичного тиску крові
підвищення онкотичного тиску тканинної рідини
підвищення проникливості капілярів
порушення лімфовідтоку

Чоловік 26 років скаржиться на бесплідність.Об*єктивно: зріст 186 см, довгі кінцівки, гінекомастія, гіпоплазія яєчок, в соскобі слизової оболонки щоки знайдені тільця Барра. Діагностований синдром Клайнфельтера. Яки механізм хромосомної аномалії має місце при даному захворюванні.
=нерозходження гетосом у мейозі
нерозходження хроматид у мітозі
транслокація
інверсія хромосоми
ділеція хромосоми

Чоловік 58 років, страждає раком сечового міхура. У процесі трудової діяльності мав контакт з канцерогенними речовинами. Дія якого з нижче перелічених канцерогенів найбільш вірогідна у данному випадку?
=b - нафтіламін
20 - метілхолантрен
бензапірен
діметіламіноазобензол
ортоаміноазотолуол

Чоловік 25 років скаржиться на загальну слабкість, озноб, біль у горлі. Об*єктивно: почервоніння в області мигдаликів. Температура тіла 38.6 оС. Які з перелічених клітин є головним джерелом ендогенних пірогенів, щ викликають гарячку у
=нейтрофіли
еозинофіли
в - лімфоцити
базофіли
тучні клітини

Жінка 23 років поступила в стаціонар з діагнозом гостра пневмонія. Захворіла гостро, 2 дні назад, коли з*явився озноб з підвищенням температури тіла до 39 , слабкість, сухий кашель. Який з перерахованих медіаторів запалення має властивості ендогенного пірогену?
=інтерлейкін 1
тромбоксан А2
гістамін
серотонін
брадикінін

Чоловік приблизно 50 років, винесений в непритомному стані з закритого помешкання, повного диму від пожежі.Який вид гіпоксії виник у постраждавшого?
=гемічна
дихальна
гіпоксична
тканинна
циркуляторна

Жінка 55 років, яка проживає в гірській місцевості, діагностований ендемічний зоб. Об*єктивно: трохи підвищеної вгодованості, загальмована, апатична, збільшення щитовидної залози. Дефіцит якого з перелічених нижче елементів викликає
=йод
фтор
марганець
молібден
натрій

Проводиться медичний огляд робітників цеху по виробництву анілінових барвників. Наявність пухлини якої локалізації може бути оцінене як професійне захворювання, внаслідок контакту з бета-нафтіламіном?
=сечового міхура
стравоходу
печінки
нирок
легень

Чоловік 25 років, потупив у лікарню через одну годину після автомобільної катастрофи. Картина гострого стресу без розповсюджених пошкоджень. Яка із перелічених змін формених елементів крові буде найбільш характерною для цього стану?
=еозинопенія
нейтропенія
лейкопенія
еритропенія
моноцитопенія

У дівчинки 14 років, експіраторна задишка. Стан розвився після гри з собакою. В анамнезі хворіє бронхіальною астмою. Якою із перелічених нижче біологічно активною речовиною найбільш вірогідно викликаний спаз гладких м*язів бронхів дівчинки?
=лейкотрієн Д4
тромбоксан А2
серотонін
брадикінін
ацетілхолін

У жінки 45 років, в період цвітіння трав з*явилось гостре запальне захворювання верхніх дихальних шляхів та очей: гіперемія, набряк, слизове виділення. Який вид лейкоцитозу буде найбільш характерним при цьому?
=Еозінофілія.
Базофілія.
Нейтрофілія.
Лімфоцитоз.
Моноцитоз.

Жінка 68 років скаржиться на відсутність рухів в правих руці і нозі. Чотири місяці тому у неї був інсульт. Об*єктивно: рухи в правих руці і нозі відсутні, тонус м*язів цих кінцівок підвищений. Який з перелічених нижче стані спостерігається у
=геміплегія
моноплегія
параплегія
тетраплегія


У підлітка 12 років, який хворіє на бронхіальну астму, розвився тяжкий напад астми: виражена експіраторна задишка, блідість шкірних покровів. Який вид порушення альвеолярної вентиляції має місце ?
=обструктивний
рестриктивний
торако-діафрагмальний
центральний
нервово-м*язовий

Чоловік 30 років, скаржиться на слабкість, жагу, головний біль та біль у попереку. Місяцьтому перехворів ангіною. На обличчі набряки. Пульс - 84 уд/хв, АТ - 175/100 мм рт. ст. В сечі - еритроцити 40-52 в полі зору, лейкоцити - 1- 2 в полі зору, білок - 4 г/л. Встановлено діагноз гострого дифузного гломерулонефриту. Який основний механізм пураження нирок у
=імунне пошкодження клубочків
пошкодження канальців
порушення гемодинаміки в нирках
порушення уродинаміки
пряме пошкодження клубочків мікроорганізмами

Чоловік 32 років, чотири роки страждає хронічним гломерулонефритом. Відмічаються явні набряки на обличчі, в останній час з*явилися набряки на ногах та тулубі, тобто гломерулонефрит перебігає з нефротични синдромом. Який із перелічених механізмів найбільш характерний для розвитку набряку у чоловіка?
=зниження онкотичного тиску крові
підвищення гідростатичного тиску крові у капілярах
підвищення онкотичного тиску тканинної рідини
утруднення лімфовідтоку
підвищення проникливості капілярів

У хлопчиеа діагностовано ендемічний зоб. Який основний механізм розвитку гіпотиреозу у хлопця?
=зниження продукції тироксину та трийодтиранину
зниження продукції тиреотропіну
зниження чутливості рецепторів тканин до тироксину та трийодтиранину
підвищення метаболізму тироксину та трийодтиранину
порушення транспорту тироксину та трийодтиранину

Чоловіку 46 років, страждаючому дифузним токсичним зобом, була проведена операція резекції щитовидяної залози. Після операції відмічаються відсутність апетиту, диспепсія, підвищена нервно-м*язова збудливість. Маса тіла не збільшилась. Температура тіла нормальна.Чим із нижче переліченого обумовлений стан чоловіка?
=зниженням продукції паратгормону
зниженням продукції тироксину
підвищенням продукції кальцітонину
підвищенням продукції тіреоліберіну
порушення продукції тироксину

Чоловік 42 років, що страждає ожирінням за верхнім типом (верхній плечовий пояс, лице місяцеподібне), АТ - 160/95 мм рт.ст., глюкоза крові 8.0 ммоль/л. Вміст кортизолу в крові підвищений, а адренокортикотропін знижений. Яка найбільш вірогідна причина розвитку гіперкортицизму?
=гормонопродуктуюча пухлина кори наднирників
гормонопродуктуюча пухлина передньої ділянки гіпофізу
зменшення продукції статинів
збільшення продукції кортиколіберину
зменшення вироблення статевих гормонів

Чоловік 30 років травмовав праве яєчко. Через 6 тижнів, коли запальний процес яєчка закінчився, з*явився біль у лівому яєчку. Об*єктивно: шкіра над яєчком червоного кольору, припухла, яєчко болісне на дотик, щільне. Який процес лежить в основі цього пошкодження?
=Аутоалергія.
Параалергія.
Гетероалергія.
Феномен Артюса.
Феномен Оверї

Хворий 57 років скаржиться на слабкість, серцебиття, задишку при виконанні нетяжкої роботи. В крові: еритроцити - 0,79 Т/л, гемоглобін - 40 г(л, КП - 1,45, лейкоцити - 3,4 Г/л. В мазку: анізоцитоз і пойкілоцитоз еритроцитів, мегалобласти і мегалоцити. Яка анемія найбільш ймовірна у цього хворого?
=В12- фолієводефіцитна
Постгеморагічна
Залізодефіцитна
Гемолітична
Апластична

У хворого виявлені такі зміни в крові:еритроцити - 2,8 Т/л, гемоглобін - 60 г(л, КП - 0,64, ретикулоцити - 0,1%, лейкоцити - 8,7 Г/л; в мазку: мікроцитоз і пойкілоцитоз еритроцитів, залізо сироватки крові 4,5 мкмоль/л. Яка анемія у хворого?
=Залізодефіцитна
Залізорефрактерна
Метапластична
Гіпопластична
Гемолітична

У хворого на цукровий діабет знижені процеси регенерації, довго не загоюються рани. З якими змінами в обміні речовин це пов язане?
=Пригніченням протеосинтезу.
Накопиченням кетонових тіл.
Ацидозом.
Зменшенням надходження глюкози в клітини.
Порушенням жирового обміну.

Під час ліквідації аварії на ЧАЕС робітник отримав дозу опромінення. При його обстеженні виявлені такі дані: еритроцитів - 2*1012/л, ретикулоцитів - немає, гемоглобін - 50 г/л, кількість лейкоцитів - 3*109/л, в лейкоцитарній формулі виявлена лімофпенія, число тромбоцитів 85*109/л. Для якої стадії променевої хвороби характерні такі зміни?
=Період розпалу.
Період первинних реакцій.
Прихований період.
Латентний період.
Кінець хвороби.

У больного на ЭКГ обнаружено смещение сегмента S-T выше изоэлектрической линии на 1 мм и увеличение продолжительности зубца T до 0,25 с. С нарушением какого процесса связаны указанные изменения на ЭКГ?
=реполяризации желудочков
деполяризации желудочков
атриовентрикулярного проведения
реполяризации предсердий
деполяризации предсердий

Відомо, що фенілкетонурія виникає внаслідок мутації гена, що відповідає за перетворення амінокислоти фенілаланіну і розпаду її до кінцевих продуктів обміну – СО2 і Н2О. Вкажіть, який шлях обміну фенілаланіну приведе до розвитку фенілкетонурії.
=Фенілаланін - фенілпіруват - кетокислоти
фенілаланін - тирозин - тироксин
фенілаланін - тирозин - меланін
фенілаланін - тирозин - норадреналін
фенілаланін - тирозин - алкаптон

У студентки, 20 років, третю весну підряд з початком цвітіння тополі починається свербіння і гіперемія очей, носа, ринорея, покашлювання, дрібне уртикарне висипання відкритих ділянок тіла. При обстеженні виявлено різке збільшення рівня Ig
Алергологом призначена специфічна гіпосенсибілізуюча терапія. Вкажіть, яким чином проводять дане лікування?
=Багаторазовим введенням малих доз алергену
Однократним введенням великої дози алергену
Введенням антигістамінних препаратів
Введенням гормонів наднирнииків
Введенням хлористого кальцію.

З метою попередження відторгнення трансплантата після пересадки органів обов*язковим є проведення курсу гормонотерапії з метою імуносупресії. Які гормони застосовують з цією метою ?
=Глюкокортикоїди.
Мінералокортикоїди.
Статеві гормони.
Катехоламіни.
Щитовидної залози

Методом непрямої калориметрії встановлено, що основний обмін досліджуваного на 40 % нижче повинного. Порушення діяльності якої ендокринної залози можна припустити ?
=Щитовидної залози.
Тимуса.
Підшлункової залози.
Епіфіза
Прищитовидних залоз

У хворого після перенесеного простудного захворювання виникло порушення міміки. Він не може закривати очі, наморщувати брови, вискалювати зуби. Який нерв
=Лицевий
Блукаючий
Трійчастий
Язикоглотковий
Інфраорбітальний

У 50-річної хворої після перенесеного інфекційного захворювання головного мозку значно збільшився діурез до 12 л за добу. При аналізі крові кількість глюкози становила 4,1 ммоль/л. Якого гормону найвірогідніше недостатньо?
=Антидіуретичного
Глюкагону
Інсуліну
Кортизону
Альдостерону

У больного с анацидным гастритом при исследовании крови получены следующие данные. Эр.- 3,0.Т/л; Нв-62 г/л; Ц.П-0,6; ретикулоц.-0,1%; тромбоц.-260 Г/л. Лейкоц.-5,6 Г/л. Б-0, Э-З, Мц-О, Ю-О, П-З, С-66, Л-25, Мо-З. Анизоцитоз-микроцитоз, выраженный пойкилоцитоз, СОЭ- 10 мм/час. О какой форме патологии крови можно думать на основании данных этого анализа?
=Железодефицитной анемии
В-12 дефицитной анемии
Микросфероцитозе
Острой постгеморрагической анемии
Апластической анемии

При обследовании температурящего больного обнаружены следующие объективные данные: кожные покровы гиперемированы, влажные на ощупь, температура тела- 37,2 0С, наблюдается полиурия, полидипсия. Какой стадии лихорадки соответствует данное состояние?
=Падения температуры
Подъема температуры
Стояния температуры



У ребенка 6 лет, часто болеющего респираторными заболеваниями, отмечаются экзематозные явления после приема цитрусовых, склонность к затяжному течению воспалительных процессов. Какой вид диатеза можно предположить в данном случае?
=Экссудативно-катаральный
Геморрагический
Лимфатико-гипопластический
Нервно-артритический
Астенический

У больного сахарным диабетом возникла диабетическая кома, которая характеризовалась полной утратой сознания, арефлексией, гипотензией, падением температуры тела, большим шумным дыханием. Как называется такой тип дыхания?
=Куссмауля
Биота
Чейн-Стокса
Периодическое
Апнейстическое

У больного с дыхательной недостаточностью рН крови 7,35. Определение рСО2 показало наличие гиперкапнии. При исследовании рН мочи отмечается повышение ее кислотности. Какая форма нарушения кислотно-основного состояния в данном случае?
=Ацидоз газовый компенсированный
Ацидоз метаболический компенсированный
Ацидоз метаболический декомпенсированный
Алкалоз газовый компенсированный
Алкалоз газовый некомпенсированный

У больного с опухолевой обтурацией желчевыводящих путей в составе кала содержится большое количество жира (стеаторрея). Недостаток какого компонента желчи
=Желчных кислот
Жирных кислот
Холестерина
Желчных пигментов
Щелочной фосфатазы

Эр.-3,1 Т/л Нв-90 г/л; Ц.П.-0,88; тромбоц. -110 Г/л; лейкоц. -51 Г/л. Б-О,Э-I, Мц-О,Ю-О, П-I, С-24, Л-70, Мо-2, лимфобласты-2%.Тени Боткина-Гумпрехта. СОЭ- 27 мм/час. Какой вид патологии сопровождается такими изменениями в
=Хронический лимфолейкоз
Острый лимфолейкоз
Туберкулез
Бруцеллез
Сифилис

Эр.--3,5 Т/л Нв-110 г/л; Ц.П.-0,9; тромбоц. -100 Г/л; лейкоц. -80 Г/л. Б-О, Э-О, Мц-О, Ю-0, П-0, С-30, Л-2, Мо-0, миелобласты-68%. СОЭ- 25 мм/час. Для какого вида лейкоза характерен представленный анализ?
=Миелобластный
Хронический миелоидный
Недифференцируемый
Хронический лимфоидный
Моноцитарный

У больного наблюдаются желтушность кожных покровов, увеличение содержания в крови непрямого билирубина, увеличение выделения стеркобилина с калом и мочой, снижение осмотической резистентности эритроцитов. Какой вид желтухи отмечается в данном случае?
=Гемолитическая
Механическая
Перенхиматозная



У длительно лихорадящего больного температура утром была в пределах 36,4-36,9 0С, к вечеру она поднималась до 37,0-38,0. Какой тип лихорадки по степени подъема температуры
=Субфебрильный
Умеренный
Гиперпиретический
Высокий


В больницу был доставлен водитель, который после работы заснул в машине с работающим двигателем. Проснувшись он почувствовал голов ную боль, началась рвота. Образование какого соединения явилось причиной наблюдающихся
=карбоксигемоглобина
метгемоглобина
карбгемоглобина
дезоксигемоглобина
сульфгемоглобина

В клинике установлено, что при беременности тяжесть симптомов ревматоидного артрита резко снижается. Ускорение секреции каких гормонов, обладающих противовоспалительным действием, наблюдается при этом?
=Глюкокортикоидов
Эстрогенов
Иодированных щитовидной железы
Катехоламинов
Гонадотропных

У больного с гипертонической болезнью появилась головная боль, шум в ушах, рвота, АД повысилось до 220/160 мм рт. ст. При обследовании выявлена ассиметрия лица справа, отсутствие произвольных движений, повышение сухожильных рефлексов и тонуса мышц правой руки и ноги. Какая форма расстройств двигательной функции нервной системы имеет место в данном случае?
=Гемиплегия
Параплегия
Тетраплегия
Гиперкинез
Моноплегия

При подъеме на *высоту* в барокамере у крысы появилось частое дыхание, тахикардия, снижение напряжения рО2 в крови. Какая форма гипоксии имеет место в данном случае?
=Гипоксическая
Гемическая
Циркуляторная
Тканевая
Дыхательная

У чоловіка 63 років зі слів родичів тричі відзначалась втрата свідомості. ЧД 18 за 1 хв., ЧСС 45 за 1 хв., АТ 100/70 мм рт. ст. На ЕКГ: частота Р 80 за хвилину, частота R 42 за хвилину, ритм правильний. Яка найбільш вірогідна аритмія?
=Повна AV блокада
AV блокада II ступеню.
AV блокада I ступеню.
Синусова брадікардія.
Синоаурикулярна блокада.

Жінка 37 років скаржиться на загальну слабкість, часті запаморочення, утруднення ковтання їжі, бажання їсти крейду. Шкіра та видимі слизові оболонки бліді. В крові: Ер. 3,4 Т/л, Hb 70 г/л, КП 0,7, ретикулоцити – 0,1%, лейк. 4,7 Г/л, еоз. – 2%, пал.-3%, сегм.-64%, лімф. – 26%, мон. – 5%. ШЗЕ – 15 мм/г. Сироваткове залізо – 7,3 мкмоль/л. Дефіцит якої речовини обумовив виникнення захворювання?
=Заліза.
Білка.
Вітаміну В6.
Вітаміну В12.
Фолієвої кислоти.

Жінка 29 років скаржиться на загальну слабкість, втрату маси тіла на 22 кг, аменорею. Хворіє після пологів. Об’єктивно: ріст 162 см, маса 46 кг, гіпоплазія молочних залоз. Діагностована гіпофізарна кахексія. Зменшення продукції якого гормону явилось найбільш суттєвим у схудненні жінки?
=Соматотропіну.
Адренокортикотропіну.
Тиреотропіну.
Меланотропіну.
Пролактотропіну.

Чоловіку 44 років з гострою пневмонією призначили пеніцилін внутрішньом’язово. Після проведення ін’єкції стан хворого різко погіршився: з’явилась задишка, хворий покрився холодним потом. Пульс 140 за хв., слабкого наповнення. АТ 90/40 мм.рт.ст. Яке ускладнення найбільш ймовірно виникло у
=Анафілактичний шок.
Тромбоемболія легеневої артерії.
Кардіогенний шок
Інфекційно-токсичний шок.


У хворого А. в серпні після праці на дачі развився стан, що характеризувався лікарем, як стан підвищенної і якісно зміненої реакції на надходження до організму сполук антигенної або гаптенної природи. Який з перерахованих станів найбільш підходить під описану лікарем характеристику?
=алергія
анафілаксія
параалергія
тахіфілаксія
імунологічна толерантність

Хворому М. з метою введення лікувальної дози протиправцевої сироватки було зроблено пробу на чутливість, яка виявилась позитивною. Як найбільш правильно провести специфічну гіпосенсибілізацію хворому? Введенням:
=малих доз протиправцевої сироватки
глюкокортикоїдів
дозволеної дози протиправцевої сироватки
антигістамінних препаратів
імунодепресантів

При дослідженні запалення піддослідній тварині ввели смертельну дозу правцевого токсину в порожнину абсцесу, індукованого скипидаром. Але піддослідна тварина не загинула. Вкажіть найбільш ймовірну причину такого результату досліду?
=Формування бар*єру навколо запалення
Активація синтезу антитіл при запаленні
Стимуляція лейкопоезу при запаленні
Посилення васкуляризації місця запалення
Активація дезінтоксикаційнної функції фагоцитів

Під час гри у волейбол спортсмен посля стрибка приземлився на зовнішній край стопи. Виник гострий біль у гомілковостопному суставі. Потім развинулась припухлість, шкіра почервоніла, стала більш теплою на дотик. Який вид розладу периферичного кровообігу розвинувся в даному випадку?
=Артеріальна гіперемія
Ішемія
Стаз
Венозна гіперемія
Тромбоз

При роботі по ліквідації наслідків аварії на АЕС робітник отримав дозу опромінення 500 рентген. Скаржиться на головний біль, нудоту, частї запаморочення. Які зміни лейкоцитів можливо спостерігати у хворого через 10 годин після опромінення?
=Нейтрофільний лейкоцитоз
Лімфоцитоз
Лейкопенія
Агранулоцитоз
Еозинофілія

У хворого А., 18 років після перенесеної краснухи виникла втрата маси тіла, постійне відчуття спраги, підвищення апетиту. Об*єктивно: добова кількість сечі 6 л, глюкоза крові 17,8 ммоль/л, в сечі виявлено глюкозу та ацетон. Яку патологію можливо запідозрити у хворого?
=Інсулінзалежний цукровий діабет
Нецукровий діабет
Стероїдний діабет
Інсуліннезалежний цукровий діабет
Діабетична нефропатія

Хворий скаржиться на відрижку, печію, часті запори. При титруванні шлункового соку отримали такі дані: загальна кислотність - 88 млмоль/л, загальна HCl - 83 млмоль/л, вільна HCl - 50 млмоль/л, зв(язана HCl - 33 млмоль/л, залишкова кислотність - 5 млмоль/л. Який стан кислотоутворюючої функції шлунку?
=Гіперацидний
Гіпоацидний
Анацидний
Нормацидний


У хворого С. з вираженим жовтушним синдромом виявлено: в крові: рівень непрямого білірубіну - 34,5 мкмоль/л, прямого - 35,2 мкмоль/л; в сечі: жовчні кислоти, уробіліноген; в калі – кількість стеркобіліногену зменшено. Який наибільш ймовірний вид жовтяниці розвинувся у хворого?
=Печінкова
Надпечінкова
Підпечінкова



У хворої після пологів развинулись такі ознаки: атрофія скелетних м(язів, дістрофія шкіри, випадання волосся, гіпотрофія внутрішніх органів, зниження температури тіла, артеріального тиску, рівня глюкози в крові, атрофія щитовидної, надниркових та статевих залоз. Для якої патології характерні ці ознаки?
=Атрофії гіпофіза
Пошкодження статевих залоз
Гіпофункції щитовидної залози
Гіперфункції аденогіпофіза
Пошкодження надниркових залоз

Після геморагічного крововиливу в мозок у хворого стали неможливими активні рухи лівої руки і ноги. Тонус м(язів ціх кінцівок підвищено, їх спинні рефлекси різко підсилені, розширено зони рефлексів. Позитивний рефлекс Бабінського. Назвіть вид розладу центральної нервової системи у хворого:
=Центральний параліч
Периферичний параліч
Спінальний шок
В(ялий параліч
Рефлекторний параліч

Альпініст на протязі кількох діб підіймався в гору. На висоті 5 000 метрів його стали непокоїти тахіпное, тахікардія, головний біль розпираючого характеру. Вкажіть можливі причини вказаних симптомів?
=Зниження парціального тиску кисню у повітрі
Зниження барометричного тиску повітря
Недостатня вентиляції легень
Газова емболія
Зниження температури повітря

Після 4-го підшкірного введення кінської сироватки у кролика на стегні розвиннулось різке запалення за типом феномена Артюса. До якого виду зміненої реактивності відноситься така
=Гіперергія
Позитивна гіпоергія
Негативна гіпоергія
Дізергія
Анергія

У вiддiлення реанiмацiї доставлено непритомного пацiєнта iз запахом ацетону з ротової порожнини. Методи експрес-аналiзу виявили у кровi 17,3 ммоль/л глюкози. Пiдвищення вмiсту яких речовин призвело до втрати свiдомостi?
=Кетоновi тiла
Молочна кислота
Глюкоза
Жирнi кислоти
Сечовина

Внаслідок захворювання нирок у паціінта відмічаються набряки. В аналізах сечі масивна протеїнурія. Який механізм є основним у виникненні набряків у такого
=Зниження онкотичного тиску плазми крові
Підвищення осмотичного тиску плазми крові
Зниження онкотичного тиску лімфи
Зниження онкотичного тиску тканин
Зниження фільтраційного тиску в нирках

В отоларингологічне відділення госпіталізована жінка зі скаргами на попадання стороннього предмету [вишневої кістки] у дихальні шляхи. Які зміни зовнішнього дихання слід очікувати?
=Глибоке рідке
Глибоке часте
Часте поверхневе
Куссмауля
Періодичне

Хворому М, 63 років, в хірургічному відділенні ампутована нога. Після ампутації виник сильний біль в ампутованій кінцівці. Який біль виник?
=Фантомний
Вторинний
Каузалгія
*Перший
Рефлекторний

У хворого встановлено стабільне підвищення температури тіла, тахікардія, емоційна лабільність, тремор. Зі зміною продукціі якого гормона пов’язане виникнення цього стану?
=Тироксину.
Вазопресину.
Тестостерону.
Альдостерону.
Інсуліну.

После аварии на химическом производстве произошло загрязнение окружающей среды нитроcоединениями. У части людей, проживающих в этой местности, появилась резкая слабость, головная боль, одышка, головокружение. Каков механизм развития данной формы гипоксии?
=Увеличение образования метгемоглобина
Снижение функции флавиновых ферментов
Образование карбоксигемоглобина
Инактивация цитохромоксидазы
Угнетение дегидрогеназ

У больной длительно болеющей сахарным диабетом 2-го типа, после нарушения диеты /употребление в пищу легкоусваеваемых углеводов/ постепенно нарастала общая слабость, снизилось артериальное давление, появились галлюцинации, судороги/. Кожа сухая, четкие проявления дегидратации организма. Уровень глюкозы крови 40 ммоль/л. Какой вид комы является причиной ухудшения состояния
=Гиперосмолярная
Ацидотическая
Лактатацидемическая
Кетонемическая
Гипогликемическая

Через 5-8 дней после применения значительных количеств лечебной сыворотки у больного выявились кожные высыпания, зуд, припухлость, боли в суставах, повысилась температура тела, в моче появился белок. Был поставлен диагноз сывороточная болезнь Что является важным фактором в патогенезе этого синдрома?
=накопление в крови циркулирующих иммунных комплексов.
дегрануляция тканевых базофилов.
активация Т-киллеров.
активация макрофагов
цитолиз форменных элементов крови

У человека с хроническим гломерулонефритом нарастает общая слабость, резкая тахикардия с периодической аритмией, заторможенность и сонливость. Какой сдвиг КОС сопровождает приближение уремической комы?
=Негазовый выделительный ацидоз
Негазовый метаболический ацидоз
Газовый ацидоз
Газовый алкалоз
Негазовый алкалоз

У группы альпинистов на высоте 3000 метров был сделан анализ крови. При этом было выявлено снижение НСО3-- до 15 ммоль/л ( норма 22-26 ммоль/л ). Каков механизм снижения
=Гипервентиляция
Усиление ацидогенеза
Гиповентиляция
Снижение аммониогенеза
Снижение реабсорбции бикарбонатов в почках

У больного через 7 лет после резекции желудка обнаружена гиперхромная В12 дефицитная анемия и фуникулерный миелоз. Какой патогенез поражений спинного мозга?
=Накопление метилмалоновой кислоты.
Гипоксическое повреждение тканей мозга
Нарушение синтеза ДНК
Дефицит фолиевой кислоты
Гипотрансферринемия

У ликвидатора аварии на Чернобыльской АЭС через некоторое время появились жалобы на повышенную возбудимость, нервозность, сердцебиение, снижение массы тела, постоянную слабость, дрожание тела, ощущение жара, плохую переносимость тепла. Гиперфункция какой железы может быть причиной указанных изменений:
=Щитовидной железы
Аденогипофиза
Коркового вещества надпочечников
Мозгового вещества надпочечников
Паращитовидной железы

У молодої людини чоловічої статі віком 20 років високого росту та астенічної будови тіла з ознаками гіпогонадизму, гінекомастією та зменшеною продукцією сперми [азооспермія] виявлено каріотип 47 ХХY. Який спадкоий синдром супроводжується такою хромосомною аномалією?
=Клайнфельтера
Віскотта-Олдрича
Тернера
Луї-Барра
Дауна

У хворого на цукровий діабет виникла значна спрага, дисфагія та порушення психічної діяльності. Який тип розладів водно-електролітного балансу характеризує поява вказаних
=Дегідратація гіперосмотична.
Дегідратація гіпоосмотична.
Дегідратація ізоосмотична.
Гідратація гіпоосмотична.
Гідратація ізоосмотична.

Пациент 64 лет с острой сердечной недостаточностью, артериальным давлением 80/60 мм рт ст, суточным диурезом 530 мл, существенно увеличена концентрация мочевины и креатинина в крови. Назовите патогенетический механизм развития азотемии и олигурии:
=Уменьшение фильтрационного давления.
Спазм приносящих артериол клубочка
Увеличение выработки вазопрессина
Уменьшение объема циркулирующей крови
Гипернатриемия

Хворому 45-ти років при оперативному втручанні на щитовидній залозі випадково видалили прищитовидні залози. Це призвело
=Тетанії
Підвищення рівня кальцію в крові і резорбції кісток
Підвищення рівня кальцію, натрію і калію в крові
Зниження артеріального тиску
Підвищення артеріального тиску

У пациента 65 лет с длительными жалобами, харктерными для хронического гастрита, в периферической крови обнаружены мегалоциты, в костном мозге мегалобластический эритропоэз. Без дальнейшего обследования наиболее вероятен диагноз:
=В12 фолиево дефицитная анемия
Апластическая анемия
Гипопластическая анемия
Гемолитическая анемия
Железодефицитная анемия

Жінка 25 років скаржиться на постійний біль в області серця, задишку при русі, загальну слабість. Об*єктивно: шкіра бліда та холодна, акроціаноз. Пульс 96 за 1 хв., АТ - 105/70 мм рт.ст. Межа серця зміщена на 2 см вліво. Перший тон над верхівкою серця послаблений, систолічний шум над верхівкою. Діагностована недостатність мітрального клапана серця. Чим обумовлене порушення кровообігу?
=Перевантаженням міокарда збільшеним об*ємом крові
Перевантаженням міокарда підвищеним опором відтоку крові
Пошкодженням міокарда
Зниженням об*єму циркулюючої крові
Збільшенням об*єму судинного русла

Одним із самих небезпечних моментів в патогенезі некрозу міокарда є подальше наростання зон некрозу, дистрофії та ішемії. Важлива роль в цьому належить підвищенню споживання міокардом кисню. Які речовини сприяють даному процесу?
=Катехоламіни
Ацетилхолін
Аденозин
Холестерин
Iони хлору

У больной с жалобами на боли в эпигастральной области опоясывающего характера при лабораторном обследовании обнаружено повышенное содержание диастазы в моче, а также содержание в кале большого количества непереваренного жира. Для какой формы патологии ЖКТ наиболее характерны описанные явления?
=Острый панкреатит
Язвенная болезнь желудка
Острый аппендицит
Воспаление толстого кишечника
Инфекционный гепатит

В эксперименте у животного в результате произведенной перерезки депрессорного нерва и разрушения каротидных клубочков развилась стойкая гипертензия. С нарушением какой функции нервной системы связано это явление?
=Вегетативной
Высшей нервной деятельности
Двигательной
Сенсорной
Трофической

У пациента, носителя наследственной серповидной аномалии эритроцитов, заболевание пневмонией сопровождалось гемолитическим кризом и развитием анемии. Что является непосредственно причиной гемолитического криза в данном случае?
=Гипоксия, вызванная пневмонией
Изменение осмолярности крови
Гетерозиготность по Нb S
Мутация структурного гена
Гипероксия

При обследовании буккального эпителия мужчины был обнаружен половой хроматин. Для какой хромосомной болезни это характерно?
=Синдром Клайнфельтера
Болезнь Дауна
Синдром Шерешевского-Тернера
Трисомия по Х-хромосоме
Гипофосфатемический рахит

При обследовании больного определяется наличие гипергликемии, кетонурии, полиурии, гиперстенурии и глюкозурии. Какая форма нарушения кислотно-основного равновесия имеет место в данной ситуации?
=Метаболический ацидоз
Газовый ацидоз
Газовый алкалоз
Метаболический алкалоз
Негазовый алкалоз

У ребенка пяти лет, часто болеющего респираторными заболеваниями, отмечаются экзематозные явления после приема некоторых пищевых продуктов, склонность к затяжному течению воспалительных процессов. Какой вид диатеза можно предположить в данном случае?
=Экссудативно-катаральный
Геморрагический
Нервно-артритический
Лимфатико-гипопластический
Астенический

Хворий 59 років госпіталізований у кардіологічне відділення в тяжкому стані з діагнозом: гострий інфаркт міокарда в області задньої стінки лівого шлуночка та перегородки, початковий набряк легень. Який первинний механізм, що викликає розвиток набряку легень у пацієнта?
=Лівошлуночкова недостатність
Легенева венозна гіпертензія
Легенева артеріальна гіпертензія
Гіпоксемія
Зниження альвеоло-капілярної дифузії кисню

При обстеженні юнака з розумовою відсталістю виявлено євнухоїдну будову тіла, недорозвиненість статевих органів. В клітинах порожнини рота - статевий хроматин. Який метод генетичного дослідження слід застосувати для уточнення
=Цитологічний
Біохімічний
Клініко-генеалогічний
Популяційно-статистичний
Дерматогліфіка

У ребенка, находящегося на искусственном вскармливании коровьим молоком, развилась тяжелая анемия: эритроциты -$4*10^{12}$/л, Нb - 68 г/л, ретикулоциты - 0\%. Какая анемия развилась у ребенка?
=Железодефицитная
$B_{12}$-дефицитная
Врожденная гемолитическая
Гипопластическая
Серповидно-клеточная

У женщины на 7-м месяце беременности стала быстро нарастать анемия: эритроциты - $2,7*10^{12}$/л, Нb -90 г/л, анизоцитоз, пойкилоцитоз, единичные мегалобластные мегалоциты, ретикулоциты - 0\%. Какой вид анемии развился в данном
=$B_{12}$-дефицитная
Железодефицитная
Гемолитическая
Постгеморрагическая
Талассемия

У больного 24 лет через полторы недели после тяжелой стрептококковой ангины проявилась отечность лица, повысилось АД. Гематурия и протеинурия 1,2 г/л. В крови выявлены антистрептококковые антитела и снижение компонентов комплемента. В микрососудах каких структур наиболее вероятно локализация скоплений иммунных комплексов, обусловивших развитие нефропатии?
=Клубочки
Пирамиды
Проксимальный отдел канальцев
Петля Генле
Нисходящий отдел канальцев

В детскую поликлинику поступил 8-месячный ребенок с диареей, вздутием живота, гипотрофией, обильным зловонным стулом. Симптомы начали проявляться и нарастать после введения в рацион питания мучных изделий. Стул обильный, пенистый, белесоватый с гнилостным запахом, рН - 6. Какую патологию можно предположить?
=Целиакия
Гиполактазия
Панкреатит
Энтероколит
Гипоацидный гастрит

У больного отмечается цианоз, увеличение печени, отеки на нижних конечностях как следствие правожелудочковой недостаточности. Какова причина развития правожелудочковой недостаточности?
=Гипертония малого круга кровообращения
Функциональное шунтирование в легких
Гиперкатехолемия
Повышение венозного давления
Кардиогенный цирроз печени

В отделение неотложной помощи из литейного цеха доставлен больной 23 лет с массой тела 60 кг. Исследования водного обмена показали: общая вода 33 л (55\% массы тела), внеклеточный сектор 17,2 л (28,6\%), внутрисосудистая жидкость 2,4 л (4\%), внутриклеточный сектор 15,8 л (26,3\%). Р осм. - 340 мосм/л, Na+ - 160 ммоль/л. Диурез 0,4 л/сут. Определите вид дисгидрии.
=Гиперосмолярная гипогидратация
Гиперосмолярная гипергидратация
Изоосмолярная гипергидратация
Гипоосмолярная гипергидратация
Дисгидрии нет

Больной сонлив, сознание помрачено, реакции на сильные раздражители заторможены. Кожа бледная, сухая, выраженные отеки. Мышечные фибриллярные подергивания. Мидриаз. Дыхание типа Чейна-Стокса с аммиачным запахом. При аускультации сердца - шум трения перикарда. Какой вид комы развился у больного?
=Почечная
Кетоацидотическая
Гиперосмолярная
Печеночная
Апоплексическая

Дитина, хвора на фенілкетонурію, страждає на розумову відсталість. Який механізм буде головним у розвитку пошкодження центральної нервової системи?
=Накопичення в крові фенілаланіну і фенілкетонів
Підвищення синтезу тирозину
Зниження синтезу меланіну
Зниження синтезу тиреоїдних гормонів
Збільшення екскреції з сечею фенілкетонових тіл

Після прийому амідопірину у хворого виникла лейкопенія. В крові знайдені антилейкоцитарні антитіла. Який тип алергічної реакції за Кумбсом і Джеллом виник у даному випадку?
=Цитотоксичний.
Стимулюючий.
Анафілактичний.
Гіперчутливість сповільненого типу.
Імунокомплексний.

Тромбоз коронарної артерії спричинив розвиток інфаркту міокарда. Які механізми ушкодження клітин є домінуючими при цьому захворюванні?
=Кальцієві
Ліпідні
Ацидотичні
Електролітно-осмотичні
Протеїнові

Через 3 тижні після гострого інфаркту міокарда у хворого з*явилися болі в серці та суглобах, запалення легень. Який механізм є основним у розвитку постінфарктного синдрому Дресслера у цього хворого?
=Аутоімунне запалення
Вторинна інфекція
Iшемія міокарда
Тромбоз судин
Резорбція білків з некротизованої ділянки міокарда

Після ампутації лівої верхньої кінцівки хворий тривалий час відчував сильний біль у ній. Який механізм формування болісних відчуттів найбільш імовірний у цьому випадку?
=Фантомний
Каузалгічний
Рефлекторний
Гіпопродукція b-ендорфіну
Гіпопродукція енкефаліну

У новонародженого відмічається збліднілість шкірних покривів, часте поверхневе дихання. При рентгенобстеженні відмічаються численні дифузно розташовані ателектази. Яка найбільш вірогідна причина цього стану ?
=Дефіцит сурфактанта
Бронхіальна астма
Пневмоторакс
Гідроторакс
Туберкульоз

Через рік після субтотальної резекції шлунка з приводу виразки малої кривизни виявлені зміни в лабораторному аналізі крові: анемія, лейко- і тромбоцитопенія, кольоровий показник - 1,3, наявність мегалобластів та мегалоцитів. Дефіцит якого фактора призводить до цих змін?
=Гастромукопротеїну
Хлороводневої кислоти
Муцину
Пепсину
Гастрину

Хвора звернулась в клініку зі скаргами на слабкість, задишку, швидку стомлюваність, запаморочення. В крові: ер. - 1,8 $10^{12}$/л; Hb - 80 г/л; к.п. - 1,5; лейк. - 3,2 $10^9$/л. У мазку: анізоцитоз, пойкілоцитоз, мегалобласти, мегалоцити. Який найбільш вірогідний діагноз?
=$В_{12}$-дефіцитна анемія
Залізодефіцитна анемія
Постгеморагічна анемія
Iмуногемолітична анемія
Гострий лейкоз

У хворого після оперативного втручання на підшлунковій залозі розвинувся геморагічний синдром із порушенням третьої фази зсідання крові. Що буде найбільш вірогідним механізмом порушення гемостазу?
=Активація фібринолізу
Зниження синтезу протромбіну
Зниження синтезу фібриногену
Якісні аномалії фібриногенезу
Дефіцит фібриностабілізуючого фактора

У собаки з ендокринною патологією було виявлено: зменшення споживання кисню у стані спокою, зниження толерантності до глюкози, зниження температури тіла. Недостатністю якого гормону можна пояснити виявлені зміни?
=Тироксину
Інсуліну
Соматотропного
Адренокортикотропного
Гонадотропного

У мишей з відсутнім волосяним покривом (тобто nude - голі) не було клітинних реакцій уповільненого типу. Для цієї патології найбільш вірогідним є:
=Відсутність вилочкової залози
Відсутність гамаглобулінів у крові
Порушення гемопоезу
Дефект фагоцитозу
Дефіцит компонентів системи комплементу

В медико-генетичну консультацію за рекомендацією андролога звернувся чоловік 35 років з приводу відхилень фізичного і психічного розвитку. Об*єктивно встановлено: високий зріст, астенічна будова тіла, гінекомастія, розумова відсталість. При мікроскопії клітин слизової оболонки ротової порожнини знайдено в 30\% статевий хроматин (одне тільце Барра). Який найбільш вірогідний діагноз?
=Синдром Клайнфельтера.
Синдром Ді Джорджі.
Хвороба Дауна
Хвороба Реклінгаузена.
Хвороба Іценка-Кушинга.

При підйомі в гори у альпініста розвинулась ейфорія, яка замінилася головним болем, запамороченням, серцебиттям, задишкою, яка змінилась на апное. Яке порушення кислотно-лужної рівноваги розвинулось в даному випадку?
=Газовий алкалоз.
Негазовий ацидоз.
Газовий ацидоз.
Негазовий алкалоз.
Видільний алкалоз.

Хворий переніс операцію з приводу резекції пілоричного відділу шлунку. Через рік скаржиться на слабкість, періодичну появу темних кіл під очима, задишку. В крові: Hb - 70 г/л, ер. - 3,0 $10^{12}$ /л. Які зміни еритроцитів в мазках периферичної крові характерні для даного захворювання?
=Гіпохромні еритроцити
Еритроцити з тільцями Кебо
Еритроцити з тільцями Жоллі
Гіперхромні еритроцити
Макроцити

У хворого виявлені множинні синяки на тілі, тривалість кровотечі за Дуке 25 хвилин, число тромбоцитів крові $25*10^{9}$/л. Для якого захворювання характерні такі ознаки?
=Спадковий дефект утворення тромбоцитів.
Гемофілія А.
Гемофілія В.
Хвороба Віллебранда.
Авітаміноз С.

Чоловік 67 років страждає на атеросклероз судин головного мозку. При обстеженні знайдена гіперліпідемія. Вміст якого класу ліпопротеїдів плазми крові найбільш вірогідно буде значно підвищений при біохімічному дослідженні?
=Ліпопротеїди низької щільності
Хіломікрони
Ліпопротеїди дуже низької щільності
Ліпопротеїди високої щільності
Комплекси жирних кислот з альбумінами

Чоловік 25 років скаржиться на часто виникаючі запальні захворювання різної локалізації. Встановлено, що він - ін*єкційний наркоман. Проба на ВІЛ-інфекцію виявилась позитивною. Який з перелічених типів клітин імунної системи найбільш суттєво вражається ВІЛ?
=хелпери
кілери
нейтрофільні гранулоцити
плазматичні клітини
--

У дівчинки 6 років пастозність повік, губ, шиї, слизової оболонки язика виникла після того, як вона з*їла апельсин. Раніше на апельсини виникали висипання на шкірі, свербіння. Який патогенетичний механізм являється провідним у розвитку набряка у дівчинки?
=підвищення проникливості капілярів
порушення лімфовідтоку
підвищення онкотичного тиску тканинної рідини
зниження онкотичного тиску крові
підвищення гідростатичного тиску крові у капілярах

Чоловік 32 років впродовж 4 років хворіє на хронічний гломерулонефрит. Госпіталізований з ознаками анасарки: АТ - 185/105 мм рт.ст. У крові: Hb - 110 г/л, ер. - 2,6 $10^{12}$/л, лейк. - 9,5 $10^9$/л, залишковий азот - 32 ммоль/л, загальний білок - 50 г/л. Яка зміна з найбільшою вірогідністю вказує на гломерулонефрит з нефротичним синдромом?
=Гіпопротеїнемія
Анемія
Лейкоцитоз
Артеріальна гіпертензія
Гіперазотемія

На ЕКГ пацієнта мають місце такі зміни: зубець Р – нормальний, інтервал Р-Q - вкорочений, шлуночковий комплекс QRST - розширений, зубець R - двогорбий або двофазний. Яка із форм аритмії має місце у даного пацієнта?
=Синдром WPW (Вольфа-Паркінсона-Уайта)
Синдром Фредеріка (тремтіння передсердь)
Атріо-вентрикулярна блокада
Миготіння шлуночків
Миготлива аритмія

У хворого в коматозному стані відчувається запах яблук із рота. Вміст глюкози в плазмі крові - 18 ммоль/л. Яку із ком найвірогідніше запідозрити в даному випадку?
=Кетоацидемічну
Токсичну
Гіперосмолярну
Лактатацидемічну
Гіпоглікемічну

Жінка 44 років скаржиться на загальну слабість, біль в ділянці серця, значне збільшення маси тіла. Об*єктивно: обличчя місяцеподібне, гірсутизм, АТ - 165/100 мм рт.ст., зріст - 164 см, вага - 103 кг, переважно накопичення жиру на шиї, верхньому плечовому поясі, животі. Що є основним патогенетичним механізмом ожиріння у жінки?
=Підвищення продукції глюкокортикоїдів
Зниження продукції тиреоїдних гормонів
Підвищення продукції інсуліну
Зниження продукції глюкагону
Підвищення продукції мінералокортикоїдів

При ревматизмі у хворих часто виявляється збільшення та деформація суглобів. Який вид запалення лежить в основі цих змін?
=Проліферативне
Ексудативне
Геморагічне
Альтеративне
Фібринозне

У хворого з тромбофлебітом нижніх кінцівок раптово після навантаження виникла задишка, різкий біль у грудях, ціаноз, набухання шийних вен. Яке найбільш імовірне порушення кровообігу виникло у хворого?
=Тромбоемболія легеневої артерії
Тромбоемболія вінцевих судин
Тромбоемболія судин головного мозку
Тромбоемболія мезентеріальних судин
Тромбоемболія ворітної вени

Після інсульту з ураженням ядер гіпоталамусу у хворого виник нецукровий діабет. Що стало причиною посиленого сечовиділення у цього хворого?
=Зменшення реабсорбції води
Зменшення реабсорбції натрію
Зниження артеріального тиску
Гіперглікемія
Прискорення клубочкової фільтрації

У хворого із запаленням легень спостерігається підвищення температури тіла. Яка біологічно активна речовина відіграє провідну роль у виникненні цього прояву?
=Iнтерлейкін-I
Гістамін
Брадикінін
Серотонін
Лейкотрієни

У неврологічне відділення у несвідомому стані доставлено молодого чоловіка з підозрою на отруєння наркотичною речовиною. Яке з порушень зовнішнього дихання слід
=Альвеолярну гіповентиляцію
Альвеолярну гіпервентиляцію
Асфіксію
Дихання Біота
Дихання Куссмауля

Жінку 32 років вжалила оса. На шкірі лівої щоки (на місці укусу) - набряк та гіперемія. Який механізм набряку є первинним у даному випадку?
=Підвищення проникливості капілярів
Підвищення гідростатичного тиску крові у капілярах
Зниження онкотичного тиску крові
Підвищення онкотичного тиску тканинної рідини
Утруднення лімфовідтоку

Чоловіку 27 років була проведена туберкулінова проба Манту. Через 24 години на місці ін*єкції відмічається інфільтрат $40*35$ мм, шкіра над яким гіперемійована. Яка із груп біологічно активних речовин визначила в основному розвиток алергічного запалення у пацієнта?
=Лімфокіни
Кініни
Простагландини
Лейкотрієни
Біогенні аміни

У жінки 34 років діагностована спадкова мікросфероцитарна гемолітична анемія (хвороба Мінковського-Шоффара). Який механізм викликав гемоліз еритроцитів у хворої?
=Мембранопатія
Ензимопатія
Гемоглобінопатія
Аутоімунне ураження
Гіпоплазія кісткового мозку

У групі дітей, які їли солодкий соковитий кавун, у двох з*явились ознаки отруєння: різка слабість, запаморочення, головний біль, блювання, задишка, тахікардія, синюшність губів, вух, кінчиків пальців. Лабораторний аналіз кавуна показав високий вміст нітратів. Який провідний механізм у патогенезі отруєння тільки у двох дітей?
=Недостатність мет-Hb-редуктази
Недостатність супероксиддисмутази
Блокада цитохромоксидази
Недостатність глутатіон-піроксидази
Недостатність каталази

У хворого з запальним процесом шкіри і підшкірної клітковини хронічного перебігу виявлено переважання процесів проліферації. Нестача якого гормону може призвести до цього?
=Кортизон
Альдостерон
Інсулін
СТГ
Тироксин

При обстеженні хворого виявлені наступні клінічні прояви: шкірні покриви рожеві, теплі на дотик, сухі, ЧСС - 92/хв.,ЧД - 22/хв., температура тіла - $39,2^0C$. Яке співвідношення процесів утворення і віддачі тепла в описаному періоді
=Теплопродукція дорівнює тепловіддачі
Теплопродукція перевищує тепловіддачу
Теплопродукція нижче за тепловіддачу
Зниження тепловіддачі на фоні незміненої теплопродукції
Посилення теплопродукції без зміни тепловіддачі

При гепатиті, інфаркті міокарда в плазмі крові хворих різко зростає активність аланін- і аспартамамінотрасфераз. Які причини зростання активності цих ферментів у крові?
=Пошкодження мембран клітин і вихід ферментів у кров
Підвищення активності ферментів гормонами
Нестача піридоксину
Зростання швидкості синтезу амінокислот у тканинах
Збільшення швидкості розпаду амінокислот у тканинах

Хворий 2 роки тому переніс операцію резекції пілоричного відділу шлунка. Спостерігається слабість, періодична поява темних кіл перед очима, задишка. В аналізі крові: Hb - 70 г/л, ер. - $3,0*10^{12}/л$, к. п. - 0,7. Які зміни еритроцитів у мазках крові найбільш характерні для даного стану?
=Мікроцити
Мегалоцити
Шизоцити
Овалоцити
Макроцити

Хворий 50 років скаржиться на спрагу, п*є багато води, виражена поліурія. Глюкоза крові 4,8 ммоль/л, в сечі глюкози і ацетону немає, сеча безбарвна, питома вага 1,002-1,004. Яка причина поліурії?
=Нестача АДГ
Гіпотиреоз
Інсулінова недостатність
Альдостеронізм
Тиреотоксикоз

Пацієнту з гострим інфарктом міокарда внутрішньовенно крапельно введено 1500 мл різних розчинів протягом 8 годин, кисень інтраназально. Смерть настала від набряку легень. Що спричинило набряк легень?
=Перевантаження лівого шлуночка об*ємом
Зменшення онкотичного тиску за рахунок гемодилюції
Алергічна реакція
Нейрогенна реакція
Інгаляція кисню

При диспансерному обстеженні хлопчику 7 років встановлено діагноз - дальтонізм. Батьки здорові, кольоровий зір нормальний. Але у дідуся по материнській лінії така ж аномалія. Який тип успадкування цієї аномалії?
=Рецесивний, зчеплений зі статтю
Домінантний, зчеплений зі статтю
Неповне домінування
Аутосомно-рецесивний
Аутосомно-домінантний

У жінки, яка протягом 15 років страждала вираженою гіпертензією, останній час з’явилась задишка, серцебиття, трохи знизився систолічний тиск. Який основний механізм виникнення у хворої серцевої недостатності?
=Перевантаження серця збільшеним опором викіду крові.
Перевантаження серця збільшеним об’ємом крові .
Ушкодження міокарду.
Порушення проведення імпульсу по міокарду.
Порушення регуляції серцевої діяльності.

У хворого з гіпохромною анемією січеться і випадає волосся, підвищена ламкість нігтів, псування смаку. Який механізм указаних симптомів?
=Дефіцит залізовмісних ферментів.
Дефіцит вітаміна В12.
Зниження продукції паратирину.
Дефіцит вітаміна А.
Зниження продукції тиреоідних гормонів.

Хвора 27 років закапала в очі краплі, до складу яких входить пеніцилін. Через декілька хвилин з’явились свербіння та печія тіла, набряк губ, повік, кашель з свистом, став падати АТ. Які імуноглобуліни приймають участь в розвитку даної алергічної реакції?
=IgE і IgG .
IgM і IgG .
IgA і IgM
IgM i IgD
IgG i IgD

Хворий 12 років поступив в клініку з гемартрозом колінного суглобу, з раннього дитинства страждає кровоточивістю. Яка хвороба у хлопчика?
=Гемофілія.
Геморрагічний васкуліт.
Гемолітична анемія.
Вітамін В12 (фолієво) дефіцитна анемія.
Тромбоцитопенічна пурпура.

У чоловіка віком 50 років, який лікувався на виразкову хворобу шлунку, нормалізувалося травлення, зникли болі, поліпшився настрій. Але через кілька тижнів знов з’явились болі в епігастрії, печія, відрижка кислим. Як можна характеризувати такий перебіг хвороби?
=Рецидив хвороби.
Період ремісії.
Термінальний стан.
Продромальний період
Латентній період.

До клініки поступив чоловік віком 40 років якого укусила гадюка. Де переважно буде проходити гемоліз еритроцитів у цьому випадку?
=У кровоносному руслі.
У клітинах печінки.
У клітинах селезінки.
У кістковому мозку.
У паренхімі нирок.

У жінки віком 45 років, яка тривалий час хворіє на бронхіальну астму, виник напад ядухи. Який патогенетичний механізм має це явище?
=Спазм дрібних бронхів.
Втрата еластичності легеневої тканини.
Зниження чутливості дихального центру.
Порушення рухомості грудної клітини.
Порушення перфузії легеневої тканини .

У жінки віком 67 років, яка тривалий час страждала на холецистит, після їжі раптово виникла різка біль у верхній частині живота, нудота, блювання. Встановлен діагноз – гострий панкреатит. Що є основною ланкою патогенеза цього
=Передчасна активація ферментів підшлункової залози.
Зниження рівня ферментів у панкреатичному соку.
Підвищення активації ферментів у дванадцятипалій кишці.
Зниження секреції панкреатичного поліпептиду.
Підвищення рівня холецистокініну.

Чоловіку 57 років, після обстеження був поставлений діагноз - В12 дефіцитна анемія, назначене лікування. Через 3 доби був зроблений контрольний аналіз крові. Що буде найбільш адекватним критерієм підвищення ерітропоезу?
=Підвищення кількості ретикулоцитів.
Підвищення рівня гемоглобіну.
Зниження кольорового показника.
Підвищення кількості тромбоцитів.
Підвищення кількості лейкоцитів.

Хворий 21 року поступив в стаціонар з загостренням хронічного тонзиліту. Скаржиться на слабкість, задуху при помірному фізичному навантаженні. Температура 37,50С. ЧСС 110 за хв. ЕКГ: ритм синусів, інтервал PQ подовжений. Яка аритмія у хворого?
=Передсердньо-шлуночкова блокада І ст.
Передсердньо-шлуночкова блокада ІІ ст.
Внутрішньопередсердня блокада.
Порушення внутрішньошлуночкової провідності.
Предсердньо-шлуночкова екстросістолія.

Хворий на механічну жовтяницю поступив в лікарню з ознаками холемічного синдрому. На ЄКГ виявлена аритмія. Яке порушення ритму серця найбільш імовірне у хворого?
=Синусова брадикардія.
Синусова тахікардія.
Предсердна екстрасистола.
Шлуночкова екстрасистола.
Атріовентрикулярна блокада.

В експерименті при моделюванні ниркової патології у тварини отримали ознаки: набряки, висока протеїнурія, гіпопротеінемія, диспротеінемія, гіперліпідемія. Для якої патології нирок характерна така сукупність ознак?
=Нефротичний синдром.
Гострий дифузний гломерулонефрит.
Пієлонефрит.
Гостра ниркова недостатність.
Хронічна ниркова недостатність.

У хворого має місце підвищення опору відтоку крові з лівого шлуночка, що призвело до включення гомеометричного механізму компенсації. При якому із перерахованих патологічних процесів може мати місце цей механізм компенсації у лівому шлуночку серця(
=Стеноз аортального клапана
Недостатність аортального клапана
Мітральний стеноз
Артеріальна гіпотензія
Емболія легеневої артерії

У хворого внаслідок отруєння сулемою розвинулася гостра ниркова недостатність, перебіг якої включав 4 стадії: перша-початкова, друга-оліго-, анурії, четверта-одуження. Як називається третя стадія гострої ниркової недостатності?
=Поліурична
Метаболічна
Гемодинамічна
Ішемічна
Патохімічна

У хворого має місце мутація гена, що відповідає за синтез гемоглобіну. Це призвело до розвитку захворювання -серповидно-клітинної анемії. Як називається патологічний гемоглобін, що виявляється при цьому захворюванні?
=HbS
HbA
HbF
HbA1
Bart-Hb

У хворого виявлено порушення реабсорбції глюкози в проксимальному відділі нефрону з розвитком глюкозурії, при цьому в плазмі крові має місце гіпоглікемія. Як називається це порушення?
=Ниркова глікозурія
Цукровий діабет
Фосфатний нирковий діабет
Позаниркова глікозурія
Галактоземія

У хворого має місце пошкодження нирок з розвитком масивної протеїнурії, гіпопротеїнемії, набряків та ретенційної гіперліпемії. Це порушення має назву:
=Нефротичний синдром
Гостра ниркова недостатність
Хронічна ниркова недостатність
Ниркова гіпертензія
Ізогіпостенурія

При запаленні ока у хворого відмічалося накопичення мутної рідини з високим вмістом білка на дні передньої камери, яке отримало назву - гіпопіон. Який процес лежить в основі спостерігаємих змін?
=Порушення мікроциркуляції
Первинна альтерація
Вторинна альтерація
Проліферація


У хворого з хронічним гіпоацидним гастритом має місце гіпохромна анемія. У мазку крові виявлені анулоцити, мікроанізоцитоз, пойкілоцитоз. Який вид анемії спостерігається у
=Залізодефіцитна
Гостра посгеморагічна
Таласемія
Серповидноклітинна
Перніціозна

У хворого на гострий апендецит у крові виявлено зростання числа лейкоцитів. Який вид лейко_цитозу може мати місце за
=Нейтрофільний
Базофільний
Еозинофільний
Лімфоцитоз
Моноцитоз

У хворого має місце передсердно-шлуночкова блокада І ступеня, що супроводжується подовженням інтервалу P-Q до 0,25 с. Порушення якої функції серця має місце за цих умов?
=Провідності
Автоматизму
Збудливості
Скоротливості
Засвоєння ритму

У хворого має місце зниження в крові кількості еритроцитів, гемоглобіну, кольорового показника, концентрації сироваткового заліза, мікроанізоцитоз, пойкілоцитоз. Ці зміни супроводжуються розвитком гіпоксії. Який вид гіпоксії спостерігається у даному випадку?
=Гемічна
Гіпоксична
Циркуляторна
Тканинна
Дихальна

У хворого має місце зміщення установочної точки терморегуляції на більш високий рівень регулювання температури тіла внаслідок пірогенного впливу інтерлейкіну 1. Як називається цей типовий патологічний процес?
=Гарячка
Перегрівання
Гіпотермія
Запалення
Гіпоксія

У хворого на крупозну пневмонію має місце гарячка з температурою тіла 39 С, при цьому добові коливання температури не перевищували 1 С впродовж 9 діб. До якого типу температурних кривих відноситься ця гарячка?
=Постійна
Гектична
Ремітуюча
Гіперпіретична
Поворотна

У хворого розвинулася гарячка, яка супроводжувалася зміщенням установчої точки термо_регуляційного центру на більш високий рівень, з послідовним чергуванням наступних стадій: Incrementi, fastigii, decrementi. При якому захворюванні можуть спостерігатися подібні зміни(
=Гостра пневмонія
Акромегалія
Цукровий діабет
Ренальний діабет
Гіпертрофія міокарда

Морській свинці ввели з метою сенсибілізації 0,1мл кінської сироватки. Якими зовнішніми ознаками проявляється стан сенсибілізації?
=Немає зовнішніх проявів.
Висипи на шкірі.
Припухлість суглобів.
Підвищення температури тіла.
Біль.

Хвора, 59 років, госпіталізована в хірургічне відділення з приводу загострення хронічного остеомієліта лівої гомілки. В аналізі крові: лейкоцитів - 15,0(109/л; лейкоцитарна формула: мієлоцити – 0\%, метамієлоцити – 8\%, паличкоядерні – 28\%, сегментоядерні нейтрофіли – 32\%, лімфоцити – 29\%, моноцити – 3\%. Яку назву має така картина крові?
=Регенеративний зсув формули вліво
Зсув формули вправо
Гіперрегенеративний зсув формули вліво
Дегенеративний зсув формули вліво
Регенеративно-дегенеративний зсув формули вліво

Дівчина, 15 років, хворіє на бронхіальну астму. Весною, в період цвітіння трав, у неї розвинувся тяжкий напад експіраторної задишки. Яка з біологічно активних речовин вірогідно викликала спазм непосмугованої м’язової тканини
=Повільно реагуюча субстанція.
Тромбоксан А2.
Простациклін.
Брадікінін.
Серотонін.

Хворий звернувся до лікаря із скаргами на те, що кожної весни, в період цвітіння рослин в нього відмічаються головний біль, нежить, слабість, підвищення температури. Який тип алергічної реакції за
=Анафілактичний
Цитотоксичний
Стимулюючий
Імунокомплексний
Клітинно-опосередкований

Хвора, 48 років, поступила в клініку із скаргами на слабість, дратівливість, порушення сну. Шкіра, склери жовтого кольору. В крові – прямий білірубін, холалемія; кал – ахолічний; сеча – темного кольору (білірубін). Яка жовтяниця має місце у
=Механічна.
Гемолітична.
Паренхіматозна.
Синдром Жільбера.
Синдром Кріглера-Найяра.

У хворого на дифтерію розвинувся міокардит з проявами недостатності кровообігу. Яке з порушень гемодінаміки є характерним для даного хворого?
=Зниження максимального артеріального тиску.
Зниження мінімального артеріального тиску.
Підвищення максимального артеріального тиску.
Зростання швидкості кровообігу.
Зниження венозного тиску.

Хворий поступив у клініку з поширеними опіками поверхні тулуба. Який із патогенетичних факторів опікової хвороби повинен бути ліквідований в першу чергу для попередження
=Біль.
Інтоксикація продуктами розпаду тканин.
Плазмовтрата.
Інфікування через пошкоджену шкіру.
Ауто алергія.

У хворого час від часу суттєво знижується вміст гемоглобіну і еритроцитів в крові. Із анамнезу з’ясовано, що ці напади завжди виникають після вживання кінських бобів. Яка із форм анемії має місце у даного хворого?
=Спадкова ферментопатія.
Спадкова мембранопатія.
Залізодефіцитна анемія.
Спадкова гемоглобінопатія
Набута гемолітична анемія.

У жінки після ускладнених пологів, що супроводжувались значною кровотечею, через місяць діагностована гіпохромна анемія. Які патологічні форми еритроцитів є характерними для
=Анулоцити.
Мішенеподібні еритроцити.
Серпоподібні еритроцити.
Сфероцити.
Мікроцити.

У хворої віком 39 років були видалені яєчники з приводу злоякісної пухлини. Через 2 роки у неї з’явились гірсутизм, огрубіння голосу, статура набула чоловічих ознак. Яке із гормональних зрушень лежить в основі даного
=Зниження естрогенів.
Відсутність прогестерону.
Посилення продукції пролактину.
Зниження продукції андрогенів
Гіпофункція наднирникових залоз.

У хворого на цироз печінки з’явилась артеріальна гіпертензія, м’язова слабість, періодичні судоми. В крові – збільшений вміст Nа і зменшений вміст К. Який із видів ендокринних порушень лежить в основі даного симптомокомплексу?
=Вторинний альдостеронізм.
Гіпопітуітаризм.
Первинний альдостеронізм.
Гіперпітуітаризм.
Гіпоальдостеронізм.

Хворий, 19 років, перніс ангіну. Через 2 тижні звернувся до лікаря із скаргами на олігурію, змінений колір сечі (“м’ясних помиїв”). Артеріальний тиск – 190/100мм.рт.ст. Що є пусковим механізмом розвитку артеріальної гіпертензії при
=Ішемія ниркових клубочків.
Гіперсекреція альдостерону.
Гіперсекреція реніну.
Збільшення продукції АДГ.
Підвищення нейрогенного компоненту судинного тонусу.

У хворого, що тривалий час лікувався з приводу однієї із форм колагенозу препаратами кортикостероїдних гормонів, з’явилися швидка фізична і психічна втомлюваність, схуднення, артеріальна гіпотензія, прогресуюча гіперпігментація шкіри. Що лежить в основі цього симптомокомплексу?
=Хронічна гіпофункція кори наднирників
Гостра гіпофункція кори наднирників
Хронічна гіперфункція кори наднирників
Гостра гіперфункція кори наднирників
Хронічна гіпофункція мозкової речовини наднирників

У больного отмечаются периодические приступы сердцебиений (пароксизмы), сильное потоотделение, приступы головной боли. При обследовании обнаружена гипертензия, гипергликемия, повышение основного обмена, тахикардия. При какой патологии надпочечников наблюдается подобная картина?
=Гиперфункции мозгового слоя
Гипофукции мозгового слоя
Гиперфункции коры надпочечников
Гипофункции коры надпочечников
Первичном альдостеронизме

Больному, трое суток назад перенесшему острую кровопотерю, исследовали кровь и получили следующие данные о ее лейкоцитарном составе: Лейкоцитов – 12*10 9/л, Б – 0, Э-3, мц-0, Ю-3, Пя-12, Ся-62, Л-16, М-4. Какая форма изменения лейкоцитарного состава крови имеет место в данном случае?
=Нейтрофилия с регенеративным сдвигом влево
Нейтрофилия с дегенеративным сдвигом влево
Нейтрофилия со сдвигом вправо
Абсолютная лимфоцитопения
Абсолютная моноцитопения

У больного желудочная секреция повышена как на механический, так и на химический раздражитель.Кислотность желудочного сока высокая. После пробного завтрака показатель рН низкий, и продолжает снижаться в течение последующих 2 часов. Какой тип желудочной секреции наблюдается у
=Возбудимый
Тормозной
Инертный
Астенический
Нормальный

У новорожденного ребенка на 5-6 сутки появился частый жидкий стул, признаки интоксикации, развилось обезвоживание. Переход на исскуственное кормление кисло-молочными смесями привел к исчезновению указанных явлений. Сделан взвод о врожденной недостаточности лактозы. Какой процесс при этом был нарушен у ребенка?
=Мембранного пищеварения
Полостного пищеварения
Экскреторной функции кишечника
Секреторной функции поджелудочной железы
Секреции желудочного сока

На фоне аллергической реакции у ребенка развился отек гортани. Какой вид дыхательной недостаточности развился в данном случае?
=Обструктивная
Рестриктивная
Дисрегуляторная
Диффузионная
Перфузионная

В параллельных експериментах на крысах, которые подвергались длительному прямому солнечному облучению, и в камерах, закрытых стеклом, было отмечено возникновение опухолей непокрытых шерстью частей кожи у животных, которые находились в открытых камерах. С влиянием какого из перечисленнях ниже факторов связано это явление?
=Ультрафиолетового облучения
Эндогенных химических канцерогенов
Биологических канцерогенов
Экзогенных химических канцерогенов
Инфракрасного облучения

78-летнему больному поставлен диагноз “паркинсонизм”. Недостаток какого медиатора в хвостатом ядре головного мезга при этом будет иметь место?
=Дофамина
Ацетилхолина
Адреналина
Гистамина
Норадреналина

После употребления меда у подростка появилась крапивница, сопровождающаяся лейкоцитозом. Какой вид лейкоцитоза возник в данном случае?
=Эозинофильный
Лимфоцитоз
Моноцитоз
Базофильный
Нейтрофильный

У больного, страдающего тяжелой формой нарушения водно-солевого обмена, наступила остановка сердца в диастоле. Какой наиболее вероятный механизм остановки
=Гиперкалиемия
Гипернатриемия
Дегидратация организма
Гипокалиемия
Гипонатриемия

Анализируя электрокардиограмму больного, врач выявил на ней при частоте сердечных сокращений 60 ударов в минуту, периодически наблюдается укорочение диастолического интервала Т- Р. Для какой аритмии характерно данное изменение на ЭКГ?
=Для синусовой экстрасистолы
Для предсердной экстрасистолы
Для предсердно-желудочковой экстрасистолы
Для желудочковой экстрасистолы
Для пароксизмальной тахикардии

В отделение реанимации доставлен больной, в крови которого обнаружено высокое содержание сульфгемоглобина. Какой тип гипоксии имеет место в данном случае?
=Гемический
Респираторный
Циркуляторный
Тканевой
Экзогенный

При исследовании первичной структуры моллекулы глобина обнаружена замена глутаминовой кислоты на валин. Для какой наследственной патологии это характерно?
=Серповидноклеточной анемии
Талассемии
Болезни Минковского-Шоффара
Фавизма
Гемоглобиноза

В отделении реанимации находится больной с двухсторонним отеком легких. Какой характер дыхания наблюдается в данном случае?
=Поверхностное частое
Глубокое частое
Поверхностное редкое
Глубокое редкое
Стенотическое

У больной температура тела высокая, бледность кожи сменилась гиперемией, возникло чувство жара, кожа горячая на ощупь. Каково соотношение процессов теплопродукции и теплоотдачи в описанной стадии лихорадки?
=Теплоотдача равна теплопродукции
Теплоотдача выше теплопродукции
Теплопродукция выше теплоотдачи
Теплоотдача ниже теплопродукции
Теплопродукция ниже теплоотдачи

Установлено, что при затрате одного и того же количества глюкозы опухолевая ткань получает энергии в 20-25 раз меньше, чем здоровая. О каком изменении в метаболизме глюкозы в опухоли говорит данное явление?
=Усилении анаеробного гликолиза
Усилении окислительных процессов
Нормальном соотношении этих процессов
Усилении тканевого дыхания
Ослаблении анаэробного дыхания

При воспалении в сосудах микроциркуляторного русла отмечается повышение их проницаемости, увеличение гидродинамического давления. В межтканевой жидкости имеет место повышение осмотической концентрации и дисперсности белковых структур. Какой вид отека будет наблюдаться в
=Смешанный
Гидродинамический
Коллоидно-осмотический
Лимфогенный
Мембранногенный

У женщины 38 лет, длительно болеющей сахарным диабетом, обнаружены признаки ожирения печени. Недостаток какого фактора является ведущим в патогенезе этого состояния?
=Липокаина
Лецитина
Инсулина
Гликогена
Ацетил – КоА

У больного, страдающего сердечной недостаточностью, наблюдается увеличение печени, отеки нижних конечностей, асцит. Какой механизм является ведущим в образовании данного отека?
=Гидродинамический
Колоидно-осмотический
Лимфогенный
Мембраногенный


У 20-летнего юноши спустя 2 недели после перенесенной лакунарной ангины появились жалобы на общую слабость, отеки под глазами. После обследования больному поставлен диагноз: острый гломерулонефрит. Какие потологические изменения в составе мочи наиболее вероятны?
=Протеинурия
Цилиндрурия
Наличие свежих эритроцитов
Пиурия
Натрийурия

При подъеме в горы на высоте 5000 метров у участников альпинистской группы появились жалобы на одышку, учащенное сердцебиение, головную боль, головокружение, звон в ушах. Какой патогенетический фактор определяет указанные явления?
=Гипоксемия
Гипокалиемия
Увеличение кислородной емкости крови
Лактацидемия
Гипернатриемия

Через несколько минут после проведения врачем-стоматологом местной анестезии зуба новокаином у пациента внезапно появилась слабость, зуд кожи. Объективно отмечались гиперемия кожних покровов, тахикардия, снижение АД до 70/40 мм рт.ст.. К какому типу аллергических реакций относится описанная патология?
=Анафилактических
Цитотоксических
Стимулирующих
Клеточноопосредованных
Иммунных комплексов

У медсестры манипуляционного кабинета со стажем работы 20 лет развился контактный дерматит верхних конечностей. К какому типу иммунных нарушений относится данное
=Аллергическая реакция замедленного типа
Первичный иммунодефицит
Аллергическая реакция немедленного типа
В-клеточный иммунодефицит
Т-клеточный иммунодефицит

Пациент 16 лет, страдающий болезнью Иценко-Кушинга, консультирован по поводу избыточного веса тела. При опросе выяснилось, что энергетическая ценность потребляемой пищи составляет 1700-1900 ккал/сут. Какова ведущая причина ожирения в данном случае?
=Избыток глюкокортикоидов
Недостаток инсулина
Избыток инсулина
Недостаток глюкокортикоидов
Гиподинамия

У дитини 6 років розвинулася гіперергічна форма запалення верхніх дихальних шляхів. З’явилася загроза серйозного порушення дихання, а тому виникла необхідність застосувати протизапальні гормони. Серед гормонів протизапальний ефект проявляє
=Глюкокортикоїди
Катехоламіни
Соматотропін
Естрогени
Мінералокортикоїди

Больной 57 лет, поступил в клинику с жалобами на слабость, головокружение, боли и чувство жжения в языке. Анализ крови: Эр. – 1,8 * 10 12/л, Hb – 59 г/л, цв.пок. –1,3, ретик. – 0,02\%, тромб. 120*10. 9 /л, лейкоц. – 2,2* 10 9/л. Б-0, Э-1, МЦ-0, Ю-0, ПЯ-0, СЯ-45, Л.49, М.-5. СОЭ – 30 мм/час. В мазке мегалобласты, мегалоциты. Какая патология крови отмечается у
=В12-дефицитная анемия
Острый миелолейкоз
Острый лимфолейкоз
Железодефицитная анемия
Апластическая анемия

У мужчини 32 років високий зріст, гінекомастія, жіночий тип обволосіння, високий голос, розумова відсталість, безпліддя. Попередній діагноз – синдром Кляйнфельтера. Для його уточнення необхідно дослідити
=Каріотип
Лейкоцитарну формулу
Сперматогенез
Групу крові
Родовід

Хворому з облітеруючим ендартеріїтом проведена гангліонарна симпатектомія. Позитивний лікувальний ефект цієї операції пов’язаний з розвитком артеріальної гіперемії нижніх кінцівок, яку можна віднести до
=Нейропалалітичної
Нейротонічної
Метаболічної
Реактивної
Робочої

Приступи гарячки у хворого виникають через добу. Під час приступу температура різко підвищується, утримується на високому рівні біля 2 год, а потім опускається до нормального рівня. Цей тип гарячки характерний для
=Триденної малярії
Сепсису
Бруцельозу
Поворотного тифу
Висипного тифу

У хворого з інфарктом міокарда з’явилися ознаки гіпоксії – задишка, тахікардія, ціаноз видимих слизових. Розвиток гіпоксії у нього пов’язаний з
=Зменшенням швидкості кровотоку
Зменшенням кількості еритроцитів
Зменшенням вмісту гемоглобіну
Інактивацією гемоглобіну
Утрудненою дисоціацією оксигемоглобіну

Машиною швидкої допомоги в лікарню доставлено непритомного чоловіка після отруєння чадним газом. Гіпоксія у нього зумовлена нагромадженням у крові
=Карбоксигемоглобіну
Оксигемоглобіну
Сульфгемоглобіну
Метгемоглобіну
Карбгемоглобіну

У мишей із спадковим ожирінням встановлено гіперглікемію та зниження кількості інсулінових рецепторів у ліпоцитах. Який патогенетичний механізм є первинним у посиленні ліпогенезу у цих тварин?
=Гіперінсулінемія
Гіпоінсулінізм
Гіпертрофія ліпоцитів
Зниження толерантності до глюкози
Підвищення депонування жирів

Чоловік 70 років страждає на подагричний артрит. В його родоводі також були хворі на подагру. Який фактор є безпосередньою причиною розвитку патології в даному
=Генетичний дефект обміну сечової кислоти
Генетичний дефект обміну сечовини
Похилий вік
Надмірне споживання м’яса
Чоловіча стать

Чоловік потрапив в лікарню через 3 доби після впливу іонізуючого випромінювання в дозі 3 грея. Зміни з боку якої фізіологічної системи в першу чергу слід очікувати у нього?
=Крові
Серцево-судинної
Імунної
Травлення
Ендокринної

Після перев’язки однієї з гілок вінцевих артерій у собаки розвинувся інфаркт міокарда, який супроводжувався проявами резорбційно-некротичного синдрому. Назвіть найбільш характерну ознаку розвитку цього синдрому.
=Збільшення в крові креатинкінази
Підвищення в крові катехоламінів
Біль за грудиною
Фібриляція шлуночків
Зниження хвилинного об’єму крові

Трансмуральний інфаркт міокарда у хворого ускладнився розвитком гострої лівошлуночкової недостатності серця. Що є найбільш типовим для цього стану?
=Набряк легенів
Набряк кінцівок
Ціаноз
Асцит
Артеріальна гіпертензія

Артеріальна гіпертензія у хворого обумовлена стенозом ниркових артерій. Активація якої системи є головною ланкою в патогенезі цієї форми гіпертензії?
=Ренін-ангіотензинової
Симпато-адреналової
Парасимпатичної
Калікреїн-кінінової
Гіпоталамо-гіпофізарної

У хворого на інфаркт міокарда артеріальний тиск знизився до 70/40 мм рт. ст. Що є первинним механізмом розвитку артеріальної гіпотензії при недостатності серця?
=Зменшення хвилинного об’єму крові
Втрата організмом натрію
Затримка калію в організмі
Генералізована вазодилятація
Вазоконстрикція периферичних судин

У хворого з хронічною серцевою недостатністю спостерігаються набряки нижніх кінцівок. Надмірна активація якої системи є головною ланкою їх патогенезу?
=Ренін-ангіотензин-альдостеронової
Гіпоталамо-гіпофізарної
Симпато-адреналової
Парасимпатичної
Калекреїн-кінінової

Підвищення внутрішньочерепного тиску у хворого з церебральною гематомою обумовило надмірну активність блукаючого нерва (ваготонію) та зміну частоти серцевих скорочень. Який вид аритмії серця виникає при цьому?
=Синусова брадікардія
Синусова тахікардія
Шлуночкова екстрасистолія
Пароксизмальна тахікардія
Передсердно-шлуночкова блокада

Після введення місцевого анестетика у пацієнта розвинувся анафілактичний шок. Який механізм порушення кровообігу є провідним при цьому?
=Зменшення тонусу судин
Гіперволемія
Біль
Активація симпато-адреналової системи
Зниження скоротливої функції серця

У людини, яка тривалий час голодувала, розвинулись набряки. Який основний механізм виникнення цих набряків?
=Зменшення онкотичного тиску крові
Збільшення онкотичного тиску тканин
Збільшення гідростатичного тиску венозної крові
Зменшення гідростатичного тиску тканин
Зменшення об’єму циркулюючої крові

У людини, яку покусали бджоли, розвинувся набряк верхніх кінцівок та обличчя. Який основний патогенетичний механізм розвитку цього набряку?
=Збільшення проникності стінки судин
Збільшення гідростатичного тиску в капілярах
Зменшення гідростатичного тиску тканин
Збільшення онкотичного тиску тканин
Зменшення онкотичного тиску крові

У хворого на пневмосклероз розвинулася легенева гіпертензія та правошлуночкова серцева недостатність з асцитом та набряками. Який основний патогенетичний механізм розвитку набряків у цього хворого?
=Збільшення гідростатичного тиску у венах
Збільшення онкотичного тиску тканин
Зменшення онкотичного тиску крові
Зменшення осмотичного тиску крові
Збільшення проникності судин

У дівчинки 10 років з вродженими вадами серця і щитоподібної залози часто виникають вірусні та грибкові захворювання. При імунологічному обстеженні виявлено відсутність Т-лімфоцитів. Яке порушення імунної системи має місце в даному випадку?
=Гіпоплазія тимуса
Гіпогаммаглобулінемія Брутона
Комбінований імунодефіцит
Синдром Шерешевського-Тернера
Спадковий дефіцит системи комплементу

У хлопчика 5 років, як і у його дідуся, часто виникають пневмонії та гнійні ураження шкіри. При імунологічному обстеженні виявлено відсутність В-лімфоцитів. Яке порушення імунної системи має місце в даному випадку?
=Гіпогаммаглобулінемія Брутона
Гіпоплазія тимуса
Комбінований імунодефіцит
Синдром Шерешевського-Тернера
Спадковий дефіцит системи комплементу

У 12-річного хлопця часто виникають вірусні та бактеріальні інфекції, екзематозні ураження шкіри. При обстеженні виявлено зменшення Т-лімфоцитів та IgМ при нормальному вмісті IgA і IgG. Якій вид патології імунної системи спостерігається у
=Комбінований імунодефіцит
Гіпоплазія тимуса
Гіпогаммаглобулінемія Брутона
Синдром Шерешевського-Тернера
Спадковий дефіцит системи комплементу

У 20-річної дівчини, яка страждає на поліпоз кишечника, в анамнезі часті грибкові та вірусні захворювання. Недостатність якої ланки імунної системи є найбільш вірогідною в даному
=Т-лімфоцитів
В-лімфоцитів
Натуральних кілерів
Комплементу
Фагоцитів

Внаслідок передозування вазодилятатора у пацієнта артеріальний тиск знизився до 60/40 мм рт. ст. та виник колапс. До якої патології це може призвести?
=Гострої ниркової недостатності
Інфаркту міокарда
Гострої печінкової недостатності
Інсульту
Гострої дихальної недостатності

Хворий на цукровий діабет вчасно не отримав ін’єкцію інсуліну, що призвело до розвитку гіперглікемічної коми (вміст глюкози в крові – 50 ммоль/л). Який механізм є головним у розвитку цієї
=Гіперосмія
Гіпокаліємія
Гіпоксія
Гіпонатріємія
Ацидоз

Тривале вживання мінералокортикоїдів призвело до появи у пацієнта м’язової слабкості. Що лежить в основі патогенезу цього явища?
=Гіпокаліємія
Гіперкаліємія
Гіпернатріємія
Гіпонатріємія
Гіперволемія

При аварійному підйомі з глибини у водолаза розвинулися судоми із втратою свідомості. Який патогенетичний механізм є основним у розвитку цих порушень?
=Газова емболія
Гіпоксія
Токсична дія кисню
Токсична дія азоту
Гіперкапнія

У шахтаря виявлено фіброз легень, що супроводжувався порушенням альвеолярної вентиляції. Який механізм виникнення цього порушення є провідним?
=Обмеження дихальної поверхні легень
Звуження верхніх дихальних шляхів
Порушення нервової регуляції дихання
Обмеження рухомості грудної клітки
Спазм бронхів

У хворого з крововиливом у ділянку задньої долі гіпофізу виникла поліурія та зменшення рівня вазопресину в крові. Що є головним механізмом розвитку поліурії в даному випадку?
=Зменшення реабсорбції води в ниркових канальцях
Збільшення фільтрації води в клубочках
Збільшення реабсорбції натрію в канальцях
Зменшення реабсорбції натрію в канальцях
Збільшення екскреції калію

Внутрішньовенне введення хлориду ртуті експериментальній тварині викликало утворення пристінкового тромбу. Який патогенетичний фактор є основним у розвитку цього патологічного процесу?
=Ушкодження судинної стінки
Активація системи зсідання крові
Активація адгезії тромбоцитів
Уповільнення течії крові
Зменшення активності антикоагулянтів

У пацієнта діагностовано цироз печінки, який супроводжується асцитом та загальними порушеннями гемодинаміки. Який патологічний синдром виникає в даному випадку?
=Портальна гіпертензія
Гепатолієнальний
Гепатокардіальний
Гепаторенальний
Гепатоцеребральний

Через декілька годин після опіку в ділянці гіперемії та набряку шкіри у хворого з’явилось вогнище некрозу. Який головний механізм забезпечує посилення руйнівних явищ в осередку запалення?
=Вторинна альтерація
Первинна альтерація
Еміграція лімфоцитів
Діапедез еритроцитів
Проліферація фібробластів

У хворого, який отримав тривалий курс лікування глюкокортикоїдами, виявлені виразки в шлунку. Який механізм є головним в їх розвитку?
=Збільшення секреції та кислотності шлункового соку
Зниження гістаміну в слизовій оболонці шлунку
Підвищення тонусу симпатичної нервової системи
Збільшення продукції простагландинів Е1, Е2
Зниження тонусу парасимпатичної нервової системи

У підлітка після перенесеного інфекційного захворювання з’явилася різко виражена аритмія з вкороченням інтервалу R-R під час вдиху і подовження його під час видиху. Що лежить в основі даного виду аритмії?
=Коливання тонусу блукаючого нерва під час акту дихання
Порушення фунції збудливості серця
Порушення скоротливої функції серця
Рефлекс Бейнбріджа
Порушення функції провідності серця

У тварини відтворили гостру недостатність мітрального клапана. Серце пристосувалось включенням гетерометричного механізму компенсації, в основі якого лежить:
=Закон Франка-Старлінга
Компенсаторна гіпертрофія міокарда
Зниження утворення кальцій-тропонінових комплексів
Посилення біосинтезу білка
Механізм Боудіча

У кролика відтворили хронічний гломерулонефрит шляхом введення великих доз протиниркової сироватки морської свинки. Що лежить в основі його розвитку?
=Алергічний процес
Некроз епітелію канальців
Гломерулосклероз
Амілоїдоз нирок
Азотемія

У хворого М., діагностовано хронічний гломерулонефрит. Внаслідок значних склеротичних змін маса функціонуючих нефронів зменшилася до 10\%. Яке з перерахованих нижче порушень лежить в основі наростаючого уремічного синдрому?
=Азотемія
Порушення водного гемостазу
Порушення осмотичного гемостазу
Ниркова остеодистрофія
Артеріальна гіпертензія

У пацієнта після переливання крові в кількості 200 мл піднялась температура до 37,9 oС. Яка з нижче названих речовин найбільш ймовірно призвела до підвищення температури тіла?
=Інтерлейкін 1
Інтерлейкін 2
Фактор некрозу пухлин
Інтерлейкін 3
Інтерлейкін 4

В шлунку людини із неканцерогенних речовин у присутності хлористоводневої кислоти можливе утворення сполук здатних викликати пухлинний ріст. До якої групи канцерогенів відносять ці сполуки?
=Нітрозаміни
Поліциклічні ароматичні вуглеводні
Аміни
Аміноазосполуки
Афлатоксин

В експерименті у кролика було видалено верхній шийний вузол симпатичного стовбура. На боці видалення спостерігається почервоніння і підвищення температури шкіри голови. Яка форма порушень периферичного кровообігу розвинулась у
=Нейропаралітична артеріальна гіперемія
Нейротонічна артеріальна гіперемія
Метаболічна артеріальна гіперемія
Венозна гіперемія
Стаз

У синтезі та виділенні медіаторів запалення приймають участь ряд клітин крові та сполучної тканини. В яких із цих клітин синтезується інтерлейкін-1?
=Макрофаги
Тканинні базофіли
Лімфоцити
Еозинофільні гранулоцити
Тромбоцити

З метою вивчення гіпоксії тварині було введено розчін ціаністого калію. Який тип кисневого голодування розвинувся у даному випадку?
=Тканинний
Респіраторний
Гіпоксичний
Гемічний
Циркуляторний

Виникнення нижчеперерахованих захворювань пов’язане із генетичними факторами.Назвіть патологію із спадковою схильністю.
=Цукровий діабет
Хорея Гентінгтона
Фенілкетонурія
Серповидноклітинна анемія
Дальтонізм

Жінка 49 років звернулася до лікаря зі скаргами на підвищену втомлюваність і появу задишки при фізичному навантаженні. Аналіз ЕКГ показав: частота серцевих скорочень 50 ударів за 1 хв; РQ – подовжений, QRS – не змінений, кількість зубців Р переважає кількість комплексів QRS. Який вид аритмії у
=Атріовентрикулярна блокада
Екстрасистолія
Синусова брадикардія
Миготлива аритмія
Синоатріальна блокада

Після перенесеної психічної травми у пацієнтки періодично відбувається підвищення артеріального тиску, що супроводжується головним болем, серцебиттям, загальною слабістю. Який механізм лежить в основі гіпертензії у хворої?
=Підвищення тонусу артеріол
Збільшення маси циркулюючої крові
Зниження хвилинного об’єму крові
Тахікардія
Веноконстрикція

Після нападу бронхіальної астми хворому проведено дослідження периферичної крові. Які зміни очікуються?
=Еозинофілія
Лейкопенія
Лімфоцитоз
Тромбоцитопенія
Еритроцитоз

Після тижневого застосування нового косметичного засобу у жінки розвинулося запалення повік з гіперемією, інфильт_рацією і болючістю. Алергічна реакція якого типу розвину_лася у пацієнтки?
=ІV
I
II
III
V

У хворої з переломом стегнової кістки виникла емболія малого кола кровообі_гу. Який вид емболії у хворої?
=Жирова
Тромбоемболія
Тканинна
Газова
Повітряна

У жінки 46 р. на фоні тривалої меноррагії розвинулась анемія : Е - 3,6 . 10 12/л , Нв - 60 г/л , К.п - 0,5 , ретикулоцити - 0,1\% . В маз_ку : гіпохромія , анулоцити , мікроцитоз , пойкілоцитоз . Який вид анемії розвинувся у хворої ? ^
=Залізодефіцитна
ВІ2- фолієводефіцитна
Апластична
Гемолітична
Гостра постгеморагічна

Хворий звернувся із скаргами на загальну слабість , носові кровотечі , значну кровоточивість ясен , множинні підшкірні крововиливи . В крові : Е – 2,2 . 1О 12/л, Нв -48г/л/, Lе - 2,1 109 /л , нейтропенія, тромбоцити - 35 109 /л . Який ймовірний патогенез тромбоцитопенії при даній патології?
=Зменшення продукції тромбоцитів
Посилене руйнування тромбоцитів
Підвищене споживання тромбоцитів
Перерозподіл тромбоцитів
Підвищена втрата тромбоцитів

У хворої на пневмонію у перший тиждень температура тіла утримувалась в межах 38,3 - 38,5оС . Така гарячка називається
=Помірною
Гіперпіретичною
Високою
Субфебрильною
Низькою

Для розвитку гарячкових станів характерним є зростання рівня білків в “гострої фази “ – церулоплазміну , фібриногену , С-реактивного протеїну . Вкажіть можливий механізм цього
=Стимулюючий вплив ІЛ-1 на гепатоцити
Руйнівна дія підвищеної температури на клітини організму
Проліферативна дія ІЛ-2 на Т-лімфоцити

Дегрануляція тканевих базофілів

У молодих здорових батьків народилась дівчинка , білява , з голубими очима .У перші ж місяці життя у дитини розвинулись дратівливість , неспокій , порушення сну і харчування , а обстеження невропатолога показало відставання в розвитку дитини . Який метод генетичного обстеження дитини слід застосувати для точного встановлення діагнозу ?
=Біохімічний
Цитологічний
Близнюковий
Генеалогічний
Популяційно-статистичний

У результаті пошкодження одного з реакторів АЕС пройшло витікання радіоактивних продуктів . Люди , які знаходилися в зоні підвищеної радіації , орієнтовно отримали по 250-300 Р. Їх негайно госпіталізовано . Який ведучий симптом буде характерний у потерпілих ?
=Лімфопенія
Лейкопенія
Анемія
Тромбоцитопенія
Нейтропенія

Внаслідок аварійної ситуації відбулася розгерметизація літака на висоті 5000 м . Яка основна причина розвитку порушень у людей , які знаходяться у літаку ?
=Гіпоксична гіпоксія
Газова емболія
Тканинна емфізема
Респіраторна гіпоксія
Циркуляторна гіпоксія

Хворий знаходиться на обліку в ендокринологічному диспансері з приводу гіпертиреозу . Крім похудіння , тахікардії , тремтіння пальців рук , приєдналися симптоми гіпоксії – головний біль, втомлюваність , мерехтіння перед очима. Який механізм дії тіреоїдних гормонів лежить в основі розвитку гіпоксії ?
=Роз’єднання, окиснення і фосфорування
Гальмувння синтезу дихальних ферментів
Конкурентне гальмування дихальних ферментів.
Посилення синтезу дихальних ферментів.
Специфічне зв’язування активних центрів дихальних ферментів

Піддослідним тваринам з їжею давали нітрит натрію . У 80\% тварин розвинулась пухлина . До якої групи канцерогенів відноситься дана сполука ?
=Нітрозаміни
Аміноазосполуки
Поліциклічні ароматичні вуглеводні
Прості хімічні речовини
Гормони

Дистрофічні зміни серцевого м’язу супроводжуються розширенням порожнин серця , зниженням сили серцевих скорочень , збільшується об’єм крові , що залишається під час систоли в порожнині серця , переповнюються вени . Все це
=Міогенної дилатації
Тоногенної дилатації
Аварійної стадії гіпертрофії міокарда
Стадії кардіосклерозу
Тампонади серця

До лікувального закладу звернувся хворий зі скаргами на по_яву свербіння в носі, постійного чхання, задухи, кон’юнктивіту, які з’являлися щовесни, у період появи топо_линого пуху. Про який тип алергічної реакції [за Джелом
=Анафілактичний (атопічний)
Цитотоксичний
Імунокомплексний
Клітинноопосередкований
Стимулюючий

В результаті гострої ниркової недостатності у хворого виникла олігурія. Яка добова кількість сечі відповідає даному симптому?
=100-500 мл
1500-2000 мл
1000-1500 мл
500-1000 мл
50-100 мл

Серед богатьох факторів, які визначають тяжність електротравми, першорядне значення маї ураження внутрішніх органів, розташованих на шляху проходження струму. Ураження якого з них ї найнебезпечнішим?
=Серця
Наднирників
Легенів
Печінки
Нирок

Хвора К., 24 роки, потрапила до лікарні зі скаргами на головні болі, болі в поперековій області, набряки на обличчі, загальну слабкість.Місяць тому перенесла ангіну. При поступленні: АТ-180/110 мм рт. ст. У сечі: виражена протеінурія, мікрогематурія, лейкоцитурія. Якою формою
=Нирковою
Гіпертонічною хворобою
Есенціальною
Первинною
Ендокринною

Хворий 60 років, який страждає на цукровий діабет, у стані коми доставлений до лікарні. Наявне шумне прискорене дихання, при якому глибокі вдохи змінялися посиленими видохами за участю експіраторних м*язів. Яка форма порушення дихання спостерігається у хворого?
=Дихання Куссмауля
Стенотичне дихання
Тахіпное
Дихання Чейн-Стокса
Дихання Біота

Хвора 65 років тривалий час страждаї стенозом аортального клапану. Після перенесеної вірусної інфекції поступила у терапевтичне відділення з ознаками хронічної серцево-судинної недостатності: задишкою, цианозом, набряками. Який тип гіпоксії спостерігаїться у хворої?
=Циркуляторний
Гемічний
Гіпоксичний
Дихальний
Тканинний

Через рік після субтотальної резекції шлунку з приводу виразки малої кривизни виявлені зміни в аналізі крові - анемія, лейко- і тромбоцитопенія, КП-1,3, наявність мегалобластів та мегалоцитів. Дефіціт якого фактору обумовив розвиток цієї
=Фактора Касла
Хлороводневої кислоти
Муцину
Пепсину
Гастрину

Хворий звернувся до лікаря зі скаргами на болі у голові, зміни у кінцівках, збільшення рук та ніг. Зовні: масивні надбровні дуги, губи. При надлишку якого гормону виявляються подібні
=Соматотропного
Адренокортикотропного
Тироксину
Глюкокортикоїдів
Адреналіну

У дитини 3 років пастозний вигляд, часті запалення з вираженним ексудатом, схильністю до тривалого перебігу та алергічних реакцій. Для якого діатезу характерні ці явища?
=Ексудативного
Астенічного
Лімфатико-гіпопластичного
Нервово-артритичного
Геморагічного

Відомо, що медіатори запалення мають клітинне або гуморальне походження. Який з медіаторів, наведенних нижче,
=комплемент
гістамін
серотонін
інтерлейкін-2
інтерлейкін-1

Больной Г., 40 лет предъявляет жалобы на сильное сердцебиение, потливость, тошноту, нарушение зрения, тремор рук, повышение артериального давления. Из анамнеза: 2 года назад был поставлен диагноз феохромоцитома. Гиперпродукция каких гормонов обуславливает данную патологию?
=катехоламинов
альдостерона
глюкокортикоидов
АКТГ
тиреоидных гормонов

Больной К., 45 лет поступил в эндокринологическое отделение с жалобами на головную боль, жажду, никтурию, периодические приступы судорог, повышение артериального давления. 6 мес. назад был поставлен диагноз первичный гиперальдостеронизм (болезнь Кона). Гиперпродукция каких гормонов обусловливает данную патологию?
=альдостерона
катехоламинов
глюкокортикоидов
АКТГ
тиреоидных гормонов

Больная Л., 27 лет с диагнозом болезнь Иценка-Кушинга предъявляет жалобы на мышечную слабость, диспептические расстройства. Объективно: кожа сухая, истонченная, мраморного оттенка, стрии, АД 180/140 мм рт.ст. Гиперпродукция каких гормонов обуславливает данную
=АКТГ
альдостерона
катехоламинов
инсулина
тироксина

У больного К., 60 лет, развился крупноочаговый инфаркт миокарда осложнившийся отеком легких. Какие кардиогемодинамические нарушения способствовали развитию отека легкого.
=острая левожелудочковая недостаточность
острая правожелудочковая недостаточность

кардиогенный шок
коллапс

У больного К, длительно страдающего бронхиальной астмой развилась острая дыхательная недостаточность. Какой основной механизм развития данной патологии?
=обструктивные нарушения вентиляции легких
рестриктивные нарушения вентиляции
нарушение кровоснабжения легких
нарушение ферментных систем в легких
снижение эластичности легочной ткани

У больного с черепно-мозговой травмой отмечается дыхание, которое характеризуется дыхательными движениями, нарастающими по глубине, а затем убывающими, после чего наступает непродолжительная пауза. Для какого патологического дыхания характерен данный тип?
=Чейн-Стокса
Биота
Куссмауля
Гастпинг-дыхания
Апнейстического

Больной В., 70 лет находится на стационарном лечении в кардиологическом отделении с диагнозом: Ишемическая болезнь сердца, сердечная недостаточность. Какой тип гипоксии имеется у данного больного?
=циркуляторная
гемическая
дыхательная
тканевая
смешанная

Больная Н., 15 лет на протяжении 5 лет страдает бронхиальной астмой. Какой тип гипоксии имеется у данной больной?
=дыхательная
циркуляторная
гемическая
тканевая
смешанная

Больная Л., 13 лет находится на стационарном лечении в гематологическом отделении областной детской больницы с диагнозом железодефицитная анемия. Какой тип гипоксии имеется у данной больной?
=гемическая
циркуляторная
тканевая
дыхательная
смешанная

Больная О., 56 лет длительное время болеет тиреотоксикозом. Какой тип гипоксии может развиться у данной больной?
=тканевая
гемическая
циркуляторная
дыхательная
смешанная

Больной Н. 3 года назад был поставлен диагноз хронический гломерулонефрит. Последние 6 месяцев появились отеки. Что лежит в основе их развития?
=протеинурия
гиперпродукция вазопрессина
нарушение белковообразующей функции печени
гиперосмолярность плазмы
гиперальдостеронизм

Больная Л., 55 лет с острым приступом печеночной колики поступила в гастроэнтерологическое отделение. Объективно: температура тела 38 С, склеры, слизистые и кожа иктеричны, моча темная, кал светлый. Жалобы на кожный зуд. Какова причина желтухи у данной больной?
=обтурация желчных ходов
деструкция гепатоцитов
усиленный распад эритроцитов
нарушение липидного обмена
длительный прием продуктов богатых каротином

Ликвидатор аварии на ЧАЭС обратился к врачу с жалобами на выраженную слабость, кровоизлияния на коже, поносы. Анализ крови: СОЭ-25 мм/ч, эритроциты 2,4*1012/л, лейкоциты 2,2*109/л, тромбоциты – 70*109/л. Для какой фазы острой лучевой болезни характерна данная картина заболевания?
=разгара болезни
первичной острой реакции
мнимого клинического благополучия
восстановления
исхода болезни

У новорожденного ребенка с судорожным синдромом и дефектом межжелудочковой перегородки сердца при рентгенологическом исследовании грудной клетки обнаружена гипоплазия тимуса. Какой иммунодефицит можно предположить у данного больного?
=синдром Ди Джорджи
болезнь Брутона
синдром Вискота-Олдрича
синдром Гуда
атаксию-телеангиоэктазию Луи-Бар

У больного Н., 25 лет, после перенесенной инфекции развился несахарный диабет. Дефицит какого гормона привел к развитию данной патологии?
=вазопрессина
альдостерона
кортизола
ренина
инсулина

Больной Д., 50 лет был поставлен диагноз микседема. Нарушение образования каких гормонов приводит к развитию данной патологии?
=тироксина и трийодтиронина
кортизола и альдостерона
АКТГ и СТГ
окситоцина и вазопрессина
инсулина и глюкагона

Больной К., 15 лет поступил в аллергологическое отделение с диагнозом бронхиальная астма. Избыточное образование каких антител обуславливает развитие основных клинических симптомов?
=IgE
IgA

IgM
IgD

Больному Н., 18 лет был поставлен диагноз сахарный диабет I типа, инсулинзависимый. Какой тип аллергических реакций может лежать в основе повреждения бета-клеток?
=цитотоксический
анафилактический
иммунокомплексный
гиперчувствительности замедленного типа
стимулирующий

Хворий після аварії на виробництві зазнав токсичного впливу ціаніду калію, що спричиноло блокаду цитохромоксидази. Який патологічний процес має місце?
=Тканинна гіпоксія
Гемічна гіпоксія
Циркуляторна гіпоксія
Гіпоксична гіпоксія
Дихальна гіпоксія

У хворого з хронічною серцевою недостатністю виявлено підвищення гідростатичного тиску в нижній порожнистій вені, що викликало розвиток патологічного процесу, який має назву:
=Серцевий набряк
Печінковий набряк
Нирковий набряк
Лімфатичний набряк
Токсичний набряк

У хворого на цукровий діабет внаслідок накопичення ?-оксимасляної та ацетооцтової кислот має місце порушення кислотно-лужного стану, яке називається:
=Метаболічний ацидоз
Метаболічний алкалоз
Газовий ацидоз
Газовий алкалоз
Негазовий алкалоз

У хворого після ішемії міокарда розвинувся реперфузійний синдром. Зростання вмісту якого електроліту в цитоплазмі кардіоміоцитів обумовлює посилення ушкодження серця?
=Іонів кальцію
Іонів калію
Іонів магнію
Іонів хлору
Іонів натрію

У хворого має місце гіперглікемія, поліурія, глюкозурія, полідипсія, метаболічний ацидоз. Про який патологічний процес слід думати лікарю?
=Цукровий діабет
Нирковий діабет
Нецукровий діабет
Фосфат-діабет
Сульфат-діабет

При обстеженні в аналізі крові пацієнта виявлено лейкоцитоз, лімфоцитоз, клітини Боткіна-Гумпрехта на фоні анемії. Про яку хворобу слід думати лікарю?
=Хронічний лімфолейкоз
Гострий мієлолейкоз
Лімфогранулематоз
Інфекційний мононуклеоз
Мієломна хвороба

У хворого з патологією нирок виявлено масивну протеїнурію, набряки, гіпопротеїнемію, ретенційну гіперліпідемію. Як називається цей патологічний процес?
=Нефротичний синдром
Гіпертензивний синдром
Сечовий синдром
Анемічний синдром


У хворого з хронічною патологією нирок після проведення проби Зимницького виявлено ізогіпостенурію. Які зміни сечовиділення будуть спостерігатися при цьому?
=Поліурія
Олігурія
Анурія
Полакіурія
Ніктурія

У хворого після травми виникло запалення. Це супроводжувалось підвищенням проникливості судинної стінки для білка з розвитком набряку. Яка речовина викликала ці
=Гістамін
Адреналін
Ангіотензин
Вазопресин
Ацетилхолін

У хворого внаслідок порушення обміну речовин утворилися поліциклічні ароматичні вуглеводні. Їх джерелом може бути:
=Холестерин
Адреналін
Гістамін
Вазопресин
Серотонін

У хворого при обстеженні виявили тромбоцитопенію, що виникла внаслідок підвищеного руйнування тромбоцитів. Який механізм виникнення цієї тромбоцитопенії?
=Імунний
Мієлотоксичний
Дефіцитний
Дизрегуляторний
Підвищеного споживання

У хворого з гіпертонічною кризою виявлено в крові збільшення концентрації ангіотензину ІІ. З чим пов’язаний пресорний ефект ангіотензину?
=Скороченням м’язів артеріол
Активізацією синтезу біогенних амінів
Гіперпродукцією простагландинів
Стимуляцією утворення вазопресину
Активацією калікреїн-кінінової системи

У хворого на гіпертонічну хворобу виявлено в крові збільшення концентрації вазопресину. На функцію якого органу впливає цей гормон?
=Нирок
Печінки
Серця
Легень
Наднирок

При обстеженні хворого в крові виявлено гіперхромію еритроцитів, мегалоцити і мегалобласти. Про яку анемію слід
=В12-фолієводефіцитну
Гіпопластичну
Постгеморагічну
Залізодефіцитну
Гемолітичну

У дитини виявлено дефіцит ферменту фенілаланінгідроксилази, збільшення у крові та сечі концентрації фенілпіровиноградної кислоти. Про яку патологію слід думати лікарю?
=Фенілкетонурія
Алкаптонурія
Тирозиноз
Альбінізм
Ксантинурія

У хворого з патологією легень внаслідок активації перекисного окиснення ліпідів спостерігається істотний дефіцит сурфактанту. Які патологічні зміни будуть мати місце у легеневій тканині в результаті відсутності цієї речовини?
=Ателектаз
Емфізема
Бронхоспазм
Набряк
Лімфостаз

Собака знаходилась в термостаті при t 40 С. У неї спостерігалось значне збільшення частоти дихання. Який вид порушення водно-електролітного обміну виник при цьому?
=Дегідратація гіперосмолярна
Дегідратація гіпоосмолярна
Дегідратація ізоосмолярна
Позитивний водний баланс


Після введення великої дози антитіл до базальної мембрани клубочків нирок у піддослідної тварини розвинувся гострий гломерулонефрит. Який вид алергічної реакції за класифікацією Кумбса і Джелла лежить в основі цієї патології?
=Цитотоксичний
Анафілактичний
Імунокомплексний
Гіперчутливість сповільненого типу
Стимулюючий

Кролів годували їжєю з додаванням холестерину. Через 5 місяця виявлені атеросклеротичні зміни в аорті. Назвіть головну причину атерогенеза в даному випадку.
=Екзогенна гіперхолестеринемія
Переїдання

Гіподінамія
Ендогенна гіперхолестеринемія

У хворого на первинний нефротичний синдром встановлений вміст загального білку крові 40 г/л. Яка причина обумовила гіпопротеїнемію?
=Протеїнурія
Вихід білка з судин у тканини
Зниження синтезу білка у печінці
Підвищений протеоліз
Порушення всмоктування білка у кішечнику

У дитини із гемолітичною хворобою новонародженого розвинулась енцефалопатія. Збільшення якої речовини в крові спричинило ураження ЦНС?
=Не сполученого з альбуміном білірубіну
Комплексу білірубін – альбумін
Білірубін – глюкуроніду
Вердоглобіну
Жовчних кислот

У тварини з недостатністю аортальних клапанів розвинулась гіпертрофія лівого шлуночка серця. В окремих його ділянках визначаються локальні контрактури. Накопичення якої речовини в міокардіоцитах обумовило контрактури?
=Кальцію
Калію
Молочної кислоти
Вуглекислого газу
Натрію

У дівчинки діагностований адреногенітальний синдром (псевдогермофродитизм). Надмірна секреція якого гормону наднирників обумовило дану патологію?
=Андрогенів
Естрогенів
Альдостерону
Кортизолу
Адреналіну

У хворого на хронічний гломерулонефрит швидкість клубочкової фільтрації (ШКФ) знижена до 20\% від нормальної. Що спричинює зниження ШКФ при хронічній нирковій недостатності?
=Зменшення кількості діючих нефронів
Тубулопатія
Обтурація сечовивідних шляхів
Ішемія нирок
Тромбоз ниркових артерій

У щура після 48 годин голодування з водою основний обмін зменшився на 20\%, дихальний коефіцієнт 0,7, наявна ліпемія, негативний азотистий баланс. Для якого періоду повного голодування типові дані зрушення?
=Максимального пристосування
Неекономної витрати енергії
Термінального



У щура після 72 годин голодування з водою маса тіла та основний обмін знизилися на 40\%, дихальний коефіцієнт 0,8, має місце кетонємія, ацидоз, набряки. Для якого періоду повного голодування типові дані зрушення?
=Термінального
Неекономної витрати енергії
Максимального пристосування



На 10-ту годину голодування з водою у щура визначається підвищення основного обміну, дихальний коефіцієнт складає 1,0, в крові збільшений вміст глюкокортикоїдів, катехол-амінів. В сечі підвищений вміст азоту. Для якого періоду повного голодування типові дані зрушення?
=Неекономної витрати енергії
Максимального пристосування
Термінального



У хворого із протеїнурією, гематурією, набряками і артеріальною гіпертензією встановлено діагноз “гострий гломерулонефрит”. Назвіть алергічні реакції за Кумбсом і Джелом, які у 80\% випадків є причиною гломерулонефриту?
=Імунокомплексні
Анафілактичні
Цитотоксичні
Гіперчутливості сповільненого типу
Стимулюючого типу

Епілептичні судоми у хворого ускладнено розвитком асфіксії, внаслідок руйнування зубних протезів та їх аспірації. Визначте тип недостатності дихання у хворого:
=Вентиляційна обструктивна
Вентиляційна рестриктивна
Вентиляційна дисрегуляторна
Перфузійна
Дифузійна

У хворого з кардіосклерозом спостерігалася аритмія з кількістю передсердних скорочень до 400 в 1 хв. При цьому частота пульсу була менше частоти серцевих скорочень. Порушення якої функції серцевого м’язу виявляється в даному випадку?
=Збудливості та провідності
Автоматизму
Збудливості
Скоротливості
Провідності

У хворого з серцевою недостатністю виникла аритмія у вигляді генерації позачергових імпульсів в пучку Гіса. Порушення якої функції серцевого м’язу спостерігається в даному випадку?
=Збудливості
Автоматизму
Провідності
Збудливості та провідності
Скоротливості

У новонародженої дитини віком 1 місяць діагностовано вроджену недостатність аортального клапану. При цьому спостерігається перенавантаження лівого шлуночка. Який механізм забезпечить компенсацію серця в даному випадку?
=Гетерометричний
Гомеометричний
Тахікардія
Гіпертрофія міокарду


У хворого виявлено екстрасистолію. На ЕКГ відсутній зубець Р, комплекс QRS деформований, є повна компенсаторна пауза. Які це екстрасистоли?
=Шлуночкові
Передсердні
Передсердно-шлуночкові
Синусні


У експериментальних щурів, що тривалий час отримували лише вуглеводну їжу, спостерігалось накопичення води в тканинах. Який патогенетичний механізм є головним у розвитку набряку в даному випадку?
=Гіпоонкотичний
Мембраногенний
Дисрегуляторний
Лімфогенний
Гіперосмолярний

У хворого на мієломну хворобу виявили білок в сечі. Яка форма протеїнурії має місце у даного хворого?
=Супраренальна.
Ренальна гломерулярна.
Ренальна тубулярна.
Субренальна пухирна.
Субренальна уретральна.

У підлітка 12 років, який протягом 3 місяців різко схуднув, вміст глюкози у крові становив 50 ммоль/л. У нього розвинулася кома. Який головний механізм її розвитку?
=Гіперосмолярний
Гіпоглікемічний
Кетонемічний
Лактатцидемічний
Гіпоксичний

У підлітка 12 років, який протягом 3 місяців схуднув на 7 кг, вміст глюкози у крові становить 20 ммоль/л, несподівано розвинулася кома. Який вид цукрового діабету найбільш
=Інсулінозалежний (I тип)
Інсулінонезалежний (II тип)
Гіпофізарний
Стероїдний
Гіпертіреоїдний

У хворого на аденому клубочкової зони кори наднирників (хвороба Конна) спостерігаються артеріальна гіпертензія, напади судом, поліурія. Що є головною ланкою в патогенезі цих
=Гіперальдостеронізм
Гіпоальдостеронізм
Гіперсекреція катехоламінів
Гіперсекреція глюкокортикоїдів
Гіпосекреція глюкокортикоїдів

Кролю внутрішньовенно ввели нефротоксичну сироватку гвінейської свинки, яка була попередньо імунізована суспензією нирки кроля. Яка патологія нирок моделюється таким
=Гломерулонефрит
Пієлонефрит

Нефротичний синдром
Тубулярна недостатність

Хворий К., 50 років, страждає на гіпертонічну хворобу. Під час фізичного навантаження у нього з’явились відчуття м’язової слабкості, нестачі повітря, синюшність слизової оболонки губ, шкіри, обличчя, дихання супроводжувалось відчутними на відстані вологими хрипами. Який механізм лежить в основі виникнення такого синдрому?
=Гостра лівошлуночкова недостатність.
Хронічна правошлуночкова недостатність.
Хронічна лівошлуночкова недостатність.
Колапс.
Тампонада серця.

Хвора, 55 років, тривалий час приймає барбітурати, що є несприятливим фактором щодо розвитку кров’яної гіпоксії. Утворення якої патологічної форми гемоглобіну може призвести до розвитку кров’яної гіпоксії у цьому випадку?
=Метгемоглобін.
Сульфгемоглобін.
Карбоксигемоглобін.
F-гемоглобін.
S-гемоглобін.

Людина може протриматись без їжі протягом 40-60 діб. Яка з перерахованих субстанцій може бути перетворена на глюкозу для забезпечення енергетичних потреб головного мозку в цей термін голодування?
=Амінокислоти.
Ацетоацетат.
Ацетон.
Жирні кислоти.
Бета-гідроксібутират.

У хворого 43 років артеріальна гіпертензія є наслідком помірного збільшення об’єму серця за хвилину і загального периферичного опору. Вкажіть гемодинамічний варіант розвитку артеріальної гіпертензії у даному випадку.
=Еукінетичний.
Гіперкінетичний.
Гіпокінетичний.

Змішаний.

У хворого на тлі менінгоенцефаліту з’явились розлади дихання. Вони характеризуються постійною амплітудою, однак дихальні рухи раптово припиняються, а потім також раптово відновлюються. Який патологічний тип дихання має місце у
=Біота.
Апнейстичний.
Стенотичний.
Куссмауля.
Чейн-Стокса.

У чоловіка 65 років на протязі 15 років була виражена артеріальна гіпертензія. Останнім часом систолічний тиск почав знижатися, а діастолічний залишився підвищеним. Який гемодинамічний тип артеріальної гіпертензії у хворого?
=Гіпокінетичний.
Нормокінетичний.
Гіперкінетичний.
Еукінетичний.


У собаки була змодельована артеріальна гіпертензія шляхом звуження ниркових артерій. При цьому збільшилась активність ренін-ангеотензін-альдостеронової системи. Який компонент цієї системи викликає найсильніший пресорний ефект?
=Ангіотензін ІІ.
Ренін.
Ангеотензін І.
Ангіотензін ІІІ.
Альдостерон

У жінки з ішемічною хворобою серця на ЕКГ кількість серцевих скорочень 230 в хвилину, зубець Р деформований, шлуночкові комплекси без змін. Які порушення серцевого ритму у хворої?
=Передсердна пароксизмальна тахікардія.
Миготлива аритмія.
Тремтіння предсердь.
Фибріляція шлуночків.
Шлуночкова екстросистола.

У хворого на мікросфероцитарну гемолітичну анемію (хворобу Минковського-Шоффара) внаслідок підвищення проникливості мембрани еритроцитів у клітину надходять іони натрію й вода. Еритроцити набувають форму сфероцитів і легко руйнуються. Який провідний механізм пошкодження еритроцитів у данному випадку?
=Електролітно-осмотичний.
Кальцієвий.
Ацидотичний.
Протеіновий.
Нуклеіновий.

На прикінці зими студент, який останнім часом відчував психічне перевантаження, захворів на гостре респіраторне захворювання після переохолодження. Що є причиною цього
=Патогенний збудник.
Психічне перевантаження.
Переохолодження.
Нераціональне харчування.
Гіповітаміноз.

Хворому на інсулінзалежний цукровий діабет був введений інсулін. Через деякий час у хворого з *явились слабкість, дратливість, посилення потовиділення. Який основний механізм розвитку гіпоглікемічної коми, що виникла?
=Вуглеводне голодування головного мозку.
Посилення глікогенолізу.
Посилення кетогенезу.
Посилення ліпогенезу.
Зменшення гліконеогенезу.

Чоловік віком 30 років отримав опромінювання дозою біля 3 грей. Яка зміна в крові буде через 8 годин після
=Лімфопенія.
Лейкопенія.
Гранулоцитопенія.
Тромбоцитопенія.
Анемія.

Хворий на рак печінки довгий час контактував з диметиламіноазобензолом (ДАБ). Чим пояснюється виникнення пухлини саме в цьому органі?
=Органотропностю ДАБ
Токсичною дією ДАБ
Порушенням жовчовиділення

Місцевою канцерогенною дією ДАБ

У чоловіка, віком 55 років, який протягом багатьох років страждав на недостатність мітрального клапану, виникла гостра серцева недостатність. Який патофізіологічний варіант недостатності серця спостерігається у цьому випадку?
=перевантаження серця об *ємом
гіпоксичне ушкодження серце.
коронарогенне ушкодження серця.
нейрогенне ушкодження серця.
гостра тампонада серця.

У хворого, віком 58 років, з гострою серцевою недосттністю, спостерігалось зменьшення добової кількості сечі – олігоурія. Який механізм цього явища?
=Зниження клубочкової фільтрації.
Зниження кількості функціонуючих клубочків.
Зниження онкотичного тиску крові.
Підвищення гідростатичного тиску на стінку капілярів.
Зниження проникності клубочкової мембрани.

У хворого, віком 30 років, який потрапив до кліники з діагнозом “гострий гломерулонефрит”, спостерігалась протеінурія. Яке порушення спричинило це явище?
=Підвищення проникності клубочкової мембрани.
Затримка виведення продуктів азотистого обміну.
Зниження онкотичного тиску крові.
Підвищення гідростатичного тиску на стінку капілярів.
Зниження кількості функціонуючих нефронів.

У чоловіка, віком 40 років було встановлено діагноз: серпоподібноклітинна анемія. Який механізм є чинником зменшення кількості еритроцитів в крові у цього хворого?
=Позасудинний гемоліз.
Внутрішньосудинний гемоліз.
Нестача заліза в організмі.
Нестача вітаміну В12 і фолієвої кислоти.
Нестача білка

У чоловіка, віком 50 років при обстеженні було виявлено зниження кількості еритроцитів в крові і підвищення рівня вільного гемоглобіну в плазмі крові (гемоглобінемію). КП становив 0,85. Який вид анемії спостерігається у хворого?
=Набута гемолітична анемія.
Спадкова гемолітична анемія.
Гостра постгеморагічна анемія.
Хронічна постгеморагічна анемія.
Анемія внаслідок порушення еритропоезу.

В результаті землетрусу чоловік 50 років два дні перебував під завалом. Після звільнення з-під завалу рятівниками у нього був встановлений синдром тривалого розчавлення. Виникнення якого ускладнення в подальшому найбільш вірогідне при
=Гостра ниркова недостатність.
Гостра печінкова недостатність.
Гостра серцева недостатність.
Гостра судинна недостатність.
Гостра дихальна недостатність.

В експерименті у адреналектомованої тварини спостерігали значну затримку калію в організмі, що обумовила гіперкаліємію. Яке порушення ритму серця найбільш ймовірне у такої тварини?
=Синусова брадикардія.
Синусова тахікардія.
Предсердна екстрасистола.
Шлункова екстрасистола.
Предсердно-шлункова блокада.

У хворого 56 років з серцевою недостатністю спостерігається набряк стоп та гомілок, шкіра в місці набряку бліда і холодна. Яка провідна ланка патогенеза набряку у хворого?
=Підвищення гідростатичного тиску в венулах.
Зменшення онкотичного тиску в капілярах.
Підвищення проникливості капілярів.
Порушення лімфовідтоку.
Позитивний водний баланс.

У дитини 5 років через 2 тижня після перенесеної ангіни виник гострий дифузний гломерулонефрит, що характеризувався олігурією, протеїнурією, гематурією, гіперазотемією. Порушення якої функції нефронів найбільш суттєве для виникнення цих
=Клубочкової фільтрації.
Канальцевої реабсорбції.
Канальцевої секреції.
Сечевиведення.
Інкреторної функції.

У ліквідатора наслідків аварії на Чорнобильській АЕС під час перебігу гострої променевої хвороби виник геморагічний синдром. Що має найбільше значення в патогенезі цього
=Тромбоцитопенія.
Порушення структури стінки судин.
Підвищення активності факторів фібрінолізу.
Підвищення активноті факторів систем протизсідання крові.
Зменшення активності факторів зсідання крові.

В експерименті на білого щура подіяли стресовим фактором (електричним струмом) і спостерігали після цього гіпотонію м’язів, артеріальну гіпотензію, гіпотермію, гіпоглікемію. Який період загального адаптаційного синдрому у щура?
=Фаза шоку.
Фаза протишоку.
Стадія резистентності.
Стадія виснаження.


Хворий 43 років чотири місяця назад переніс травматичну ампутацію лівої нижньої кінцівки. Зараз він скаржиться на відчуття наявності ампутованої кінцівки і постійний сильний, іноді нестерпний біль у ній. Який вид болю у хворого?
=Фантомний
Каузалгія.
Невралгія.
Таламічний
Рефлекторний

Хворий 32 років поступив в стаціонар зі скаргами на загальне недомагання, тошноту, біль справа внизу живота. Після огляду лікарем був поставлений діагноз – гострий апендицит. Який вид болю у хворого?
=Вісцеральний
Соматичний глибокий.
Соматичний поверхневий ранній
Соматичний поверхневий пізній.
Фантомний

У больного с ОПН в стадии полиурии азотемия не только не уменьшилась, но продолжала нарастать. С нарушением какой функции почек это связано?
=Секреция
Фильтрация
Реабсорбция
Инкреция


У экспериментального животного во втором периоде полного пищевого голодания повышается количество липидов в крови. Какая форма гиперлипемии наблюдается в данном случае?
=Транспортная
Алиментарная
Ретенционная
Метаболическая
-

В результате перенесенной нейроинфекции больной потерял способность определять наощупь знакомый предмет. Как называется нарушенный вид чувствительности?
=Стереогноз
Чувство дискриминации
Двумерно-пространственное чувство
Чувство локализации
Мышечно-суставное чувство

У экспериментального животного удалили мозжечок. В результате движения потеряли плавность, точность, исчезла соразмерность движений. Как называется описанное явление?
=Атаксия
Гиперкинез
Тремор
Ригидность
Атетоз

У больной, длительно страдающей туберкулезом легких, обнаружены признаки бронзовой (аддисоновой) болезни. Какой показатель нарушения обмена углеводов будет при этом наблюдаться?
=Гипогликемия
Гипергликемия
Галактоземия
Фруктоземия


Больной жалуется на постоянную жажду, головную боль, слабость, массивное мочевыделение (до 10л/сут). Перенес черепно-мозговую травму. Относительная плотность мочи 1008, патологических составных частей нет. Каков возможный механизм нарушения водного обмена в этом случае?
=Недостаточная продукция антидиуретического гормона
Гиперпродукция антидиуретического гормона
Недостаточный синтез инсулина
Гиперпродукция альдостерона
Недостаточный синтез альдостерона

Во время диспансерного осмотра у мужчины 36 лет, по профессии - водителя, уровень АД составил 150/90 мм рт ст. К концу рабочего дня появляется шум в ушах, недомогание, проходящее после отдыха. Диагностирована гипертоническая болезнь. Каков ведущий патогенетический механизм при данной форме заболевания?
=Нейрогенный
Почечный
Гуморальный
Эндокринный
Рефлексогенный

У ребенка 14 лет, больного дифтерией, в период кризиса при резком падении температуры на фоне тахикардии АД составляет 70/50 мм рт. ст К какой форме нарушения сосудистого тонуса относится данное явление?
=Острая гипотензия

Хроническая гипотензия
Вегето-сосудистая дистония
Гипотоническая болезнь

У больного с анацидным гастритом проведенное исследование крови показало следующие результаты: Эр. 2,5*1012/л; Hb-50г/л; Ц.П.-0,6; ретикулоцитов-0,02\%; микроцитоз. Какой вид анемии наблюдается у больного?
=Железодефицитная
Белководефицитная
Фолиеводефицитная
Апластическая
Гипопластическая

Мужчина 54 лет обратился к врачу в жалобами на общую слабость, частые простудные заболевания, постоянные “синяки” на теле. В анализе крови: Эр. 2,5*10^12/л; Hb-80г/л; Ц.П.-0,9; ретикулоц.-нет; тромбоц. – 50 *10^9/л; лейкоц. - 58*10^9/л; лейкоц. формула: Б-5, Э-15, мбл-6, мц-10, Ю-18, п/я 26, с/я-10, Л-8, М-2, СОЭ-40 мм час. Каково гематологическое
=Хронический миелоидный лейкоз
Лейкемоидная реакция
Миелобластный лейкоз
Хронический лимфолейкоз
Базофильно-эозинофильный лейкоцитоз

У больного днем внезапно поднялась температура тела до 390С и через 6 часов вернулась к норме. На вторые сутки приступ повторился: в период пароксизма температура достигла 410С, период апирексии наступил через 8 часов. Как называется такой тип температурной кривой?
=Интермиттирующий
Возвратный
Гектический
Септический
Постоянный

Врач-исследователь в составе альпинистской экспедиции поднялся на высоту 5000м. На 3-й день пребывания у него появились признаки горной болезни: одышка, головная боль, потеря аппетита, общая слабость, цианоз. Какой тип гипоксии имеет место в данном случае?
=Гипоксическая
Циркуляторная
Застойная
Гемическая
Тканевая

В клинику профессиональных заболеваний поступил больной с диагнозом пневмокониоз. Нарушение какого компонента внешнего дыхания можно считать ведущим?
=Поражение процесса диффузии газов
Поражение вентиляции легких
Нарушение перфузии легких
Нарушение нервной регуляции внешнего дыхания
Нарушение гуморальной регуляции внешнего дыхания

При обследовании больного, страдающего язвенной болезнью желудка, установлено, что желудочная секреция достигает максимума через 5 минут после приема пищи, а спад ее наступает через 1 час 25 мин. Какой тип желудочной секреции присущ данному бальному?
=Возбудимый
Нормальный
Тормозный
Астенический
Инертный

У больного, страдающего гипертонической болезнью, обнаружены суточные колебания общего периферического сопротивления сосудов току крови. С наибольшим участием каких сосудов это связано?
=Артериол
Аорты
Капилляров
Артериоло-венулярных анастомозов
Вен

При рентгенологическом обследовании у больного язвенной болезнью обнаружен стеноз привратника. Это нарушение является:
=Патологическим состоянием
Патологическим процессом
Заболеванием
Патологической реакцией


После продолжительной и тяжелой болезни у больного снизилось давление (60/40 мм рт.ст.), наблюдается тахикардия, одышка, сознание затемненное. Это состояние можно
=Преагонию
Агонию
Шок
Клиническую смерть


У больного ишемической болезнью сердца отмечается гипертрофия миокарда, тахикардия, снижение МОК. Какой из механизмов является ведущим в повреждении кардиомиоцитов в данном случае?
=Повреждение специфических мембранних насосов
Увеличение числа ( и ( - адренорецепторов
Потеря Мg2+ кардиомиоцитами
Потеря Са2+ кардиомиоцитами
Дегидратация кардиомиоцитов

После массивной кровопотери у больного отмечается олигурия, гиперазотемия, отек мозга, отек легких. В снижении диуреза в данном случае имеет значение:
=Снижение эффективного фильтрационного давления
Увеличение реабсорбции воды в канальцах
Увеличение реабсорбции натрия в канальцах
Перераспределение воды в организме
Снижение проницаемости клубочковой мембраны

У больного желтухой в крови повышено содержание прямого билирубина, желчных кислот, в моче отсутствует стеркобилиноген. При какой желтухе возможно наличие этих признаков?
=Механической
Печеночной
Паренхиматозной
Гемолитической
Надпеченочной

В результате нарушения техники безопасности произошло отравление сулемой (хлористой ртутью). Через 2 дня суточный диурез составил 620 мл. У больного появились головная боль, рвота, судороги, одышка, в легких-влажные хрипы. Как называется такая патология?
=Острая почечная недостаточность
Хроническая почечная недостаточность
Уремическая кома
Гломерулонефрит
Пиелонефрит

В клинику доставили больного в бессознательном состоянии, изо рта-запах ацетона. Сахар крови - 25 ммоль/л, кетоновые тела –0,57 ммоль/л. При недостаточности какого гормона может развиться такое состояние?
=Инсулина
Тироксина
Глюкокортикоидов
Альдостерона
Соматотропного гормона

У длительно лихорадящего больного (субфебрилитет) в крови отмечается лейкоцитоз ( в частности, лимфоцитоз), повышенная СОЭ. Для какого из перечисленных ниже заболеваний характерны такие изменения?
=Туберкулеза
Микседемы
Гельминтоза
Стафилококковой пневмонии
Хронического аппендицита

У новорожденного ребенка с пилоростенозом наблюдается часто повторяющаяся рвота, сопровождающаяся апатией, слабостью, повышением тонуса мышц, иногда судорогами. Какая форма нарушения кислотно-основного состояния развилась у
=Негазовый алкалоз
Газовый алкалоз
Газовый ацидоз
Метаболический ацидоз
Выделительный ацидоз

Во время записи ЭКГ человеку слегка нажали пальцем па глазные яблоки и продолжали запись. Возникло урежение сердечного ритма-брадикардия. Нарушение какого свойства проводящей системы сердца лежит в основе этого явления?
=Автоматизма
Возбудимости
Сократимости
Проводимости


В основе развития иммунных и аллергических реакций организмом используются одинаковые механизмы ответа иммунной системы на антиген. Определите основное отличие аллергических реакций от иммунных?
=Развитие повреждения тканей
Количество попадающего антигена
Особенность строения антигенов
Пути попадания антигенов в организм
Наследственная предрасположенность

Ребенок 10 лет во время игры порезал ногу осколком стекла и был направлен в поликлинику для введения противостолбнячной сыворотки. С целью предупреждения развития анафилактического шока лечебную сыворотку вводили по Безредке. Какой механизм лежит в основе подобного способа гипосенсибилизации организма?
=Связывание фиксированных на тучных клетках IgE
Блокирование синтеза медиаторов тучных клеток
Стимуляция иммунологической толерантности к антигену
Стимуляция синтеза антиген-специфичных IgG2
Связывание рецепторов к IgE на тучных клетках

У больной Б., анализ крови выявил признаки ВИЧ инфекции. Поражение каких иммунокомпетентных клеток характерно для СПИДа?
=Т-хелперов
Т-киллеров
В-лимфоцитов
Макрофагов
Нейтрофилов

У чоловіка 52 років ясна різців верхньої щелепи темно червоного кольору, гноєтеча з ясеневих закутків, розхитування зубів. Діагностований парадонтит. Який період хвороби у цього чоловіка?
=Виражених проявів.
Предхвороба.
Латентний.
Продромальний.
Кінець хвороби.

У больного С. через сутки после апендиктомии при анализе крови выявили нейтрофильный лейкоцитоз с регенеративным сдвигом. Какой наиболее вероятный механизмом развития абсолютного лейкоцитоза в периферической крови больного?
=усиление лейкопоэза
перераспределение лейкоцитов в организме
уменьшение разрушения лейкоцитов
замедление эмиграции лейкоцитов в ткани
активация иммунитета

У студента Г. после экзамена при анализе крови выявили лейкоцитоз без существенного изменения в лейкоцитарной формуле. Выберите наиболее вероятный механизмом развития относительного лейкоцитоза в периферической крови?
=перераспределение лейкоцитов в организме
усиление лейкопоэза
уменьшение разрушения лейкоцитов
замедление эмиграции лейкоцитов в ткани
ускоренный лейкопоэз

У больного Ю., 19 лет, выявлена хронической приобретенной гемолитической анемии. Что является ведущим патогенетическим механизмом развития этой патологии?
=аутоиммунный гемолиз
токсический гемолиз
внутриклеточный гемолиз
гипоосмолярность плазмы
осмотический гемолиз

У больной Н., 49 лет, отмечается ограничение произвольных движений в левых конечностях. Тонус мышц в левой руке и ноге повышен по спастическому типу, усилены местные сухожильные рефлексы, выявляются патологические рефлексы. Какой наиболее вероятный механизм привел к развитию мышечной гипертонии и гиперрефлексии?
=снижение тормозных нисходящих влияний
активация мотонейронов вследствие инсульта
активация возбуждающих влияний из очага инсульта
активация синаптической передачи импульсов
торможение мотонейронов коры головного мозга

У больной Л., 40 лет, объем мышц в области голени правой ноги на 2 см меньше, чем на левой. Ахилов и коленный рефлексы справа отсутствуют. Какой наиболее вероятный механизм возникновения гипорефлексии при периферическом параличе?
=нарушение проведения возбуждения
торможение пирамидных мотонейронов
нарушение синаптической передачи импульсов
активация возбуждающих влияний из ЦНС
нарушение восприятия раздражения

У больного А., 38 лет, на 3-м году заболевания системной красной волчанкой выявилось диффузное поражение почек, сопровождающееся массивными отеками, выраженной протеинурией, гиперлипидемией, диспротеинемией. Каков наиболее вероятный механизм развития протеинурии в данной клинической ситуации?
=аутоиммунное повреждение нефронов
воспалительное повреждение нефронов
ишемическое повреждение канальцев
увеличение уровня протеинов в крови
поражение мочевыводящих путей

Больной А., 27 лет, доставлен в больницу с желудочным кровотечением в тяжелом состоянии. АД - 80/60 мм.рт.ст. Больной выделяет 60 - 80 мл мочи за сутки с относительной плотностью 1,028-1,036. Какой патогенетический механизм вероятнее всего обусловил падение суточного диуреза в данной клинической ситуации?
=снижение гидростатического давления в капиллярах клубочков
повышение осмотического давления мочи
высокий уровень остаточного азота в крови
повышение коллоидно-осмотического давления в крови
повышение гидростатического давления в капсуле Шумлянского-Боумена

Больная Р., 39 лет. поступила в почечный центр в тяжелом прекоматозном состоянии вследствие развития хронической почечной недостаточности. АД - 190/120 мм.рт. ст. Какой первичный патогенетический механизм вероятнее всего обусловил развитие гипертензии у больной?
=увеличение секреции ренина
увеличение секреции глюкокортикоидов
уменьшение секреции альдостерона
увеличение секреции катехоламинов
увеличение концентрации ангиотензинов в крови

Больной с ревматическим миокардитом периодически ощущает перебои в работе сердца. При ЭКГ обследовании выявлено нерегулярное появление идиовентрикулярных экстрасистол. Какой патогенетический механизм обуславливает возникновение компенсаторной паузы при этих экстрасистолах?
=Рефрактерность миокарда к восприятию очередного импульса
Задержка возбуждения в атриовентрикулярном узле
Ретроградное проведение возбуждения к предсердиям
Подавление функции синусового узла
Нарушение сократительной способности миокарда

У больного с хронической сердечной недостаточностью при УЗИ- исследовании выявлена гипертрофия левого желудочка. Какие изменения в гипертрофированном сердце характерны в данной ситуации?
=Уменьшение энергообеспечения миокардиоцитов
Увеличение количества капилляров в миокарде
Активация электрической активности проводящей системы сердца
Увеличение нервных окончаний в миокарде


У больного Б., после развития инфаркта миокарда произошло резкое падение систолического АД до 60 мм.рт.ст. с тахикардией 140 уд/мин, одышкой, потерей сознания. Какой механизм является первичным в патогенезе развившегося
=Уменьшение минутного объема крови
Повышение возбудимости миокарда продуктами некротического распада
Снижение объёма циркулирующей крови
Развитие пароксизмальной тахикардии
Развитие анафилактической реакции на миокардиальные белки

У больной М., с хронической сердечной недостаточностью было проведено плановое исследование показателей кардиогемодинамики. Какие наиболее вероятные показатели свидетельствуют о развитии декомпенсации сердца?
=Миогенная дилятация
Развитие тахикардии
Тоногенная дилятация

Повышение центрального венозного давления

Больной А, 59 лет, директор частного предприятия. После проверки налоговой инспекцией вечером появились интенсивные жгучие боли, локализованные за грудиной, иррадиирующие в левую руку. Через 15 мин состояние больного нормализовалось. Какой из возможных механизмов развития стенокардии является ведущим у данного больного?
=повышение в крови уровня катехоламинов
атеросклероз коронарных сосудов
внутрисосудистая агрегация форменных элементов
тромбоз коронарных сосудов
функциональная перегрузка сердца

У больного А. с обширным инфарктом миокарда развилась сердечная недостаточность. Какой патогенетический механизм способствовал ее развитию?
=Уменьшение массы функционирующих миокардиоцитов
Перегрузка давлением
Перегрузка объемом
Острая тампонада сердца
Реперфузионное поражение миокарда

У больного с длительным приступом стенокардии провели энзимодиагностику. Увеличение уровня содержания в крови какого фермента является решающим для подтверждения диагноза “инфаркт миокарда” в первые 2-4 часа его развития?
=Креатинфосфокинааы
Альдолазы
Липопротеинлипааы
Аланинаминотрансферазы
Ацетилхолинестеразы

В кардиологическом отделении находится больной с диагнозом “атеросклероз, ИБС, стенокардия покоя”. При лабораторном исследовании в плазме крови выявлено повышение уровня липидов. Какой класс липидов плазмы крови играет ведущую роль в патогенезе атеросклероза?
=липопротеиды низкой плотности
хиломикроны
альфа-липопротеиды
липопротеиды высокой плотности
комплексы жирных кислот с альбуминами

У хирурга С. после проведения длительной операции повысилось АД (140/110 мм.рт.ст.). Какие изменения гуморальной регуляции могут быть причиной повышения артериального давления в данном случае?
=Активация симпато-адреналовой системы
Активация образования и выделения альдостерона
Активация ренин-ангиотензивной системы
Активация калликреин-кининовой системы
Торможение симпато-адреналовой системы

Лікар-стоматолог при обстеженні ротової порожнини чоловіка, який 5 років тому назад переніс операцію видалення шлунка, виявив атрофічно-запальні процеси слизової оболонки ротової порожнини, ясен, язика. Чим обумовлені зміни слизової оболонки ротової порожнини хворого?
=Дефіцитом вітаміну В12.
Дефіцитом вітаміну С.
Дефіцитом вітаміну В1.
Порушенням харчування.
Психоемоційним перенапруженням.

У хворого з пошириними опіками шкіри тулуба мають місце ознаки вираженої інтоксикації. Для якої стадії опікової хвороби це характерно?
=Опікової токсемії
Опікового шоку
Опікової інфекції
Опікового виснаження
Термінальної

У хворого з опіковою хворобою як ускладнення розвинувся ДВЗ-синдром. Яку стадію ДВЗ-синдрому можна запідозрити, якщо відомо, що кров хворого згортається менше ніж за 3 хв.?
=Гіперкоагуляції
Перехідну
Гіпокоагуляції
Фібринолізу
Термінальну

Після автомобільної катастрофи у хворого діагностовано травму середньої третини плеча із неповним розривом середнього нерва. Крім рухових та сенсорних розладів нижче місця травми хворий скаржиться на різкі, пекучі, нестерпні болі. Якого хорактеру є такі болі?
=Каузалгія
Проекційний біль
Відбитий
Фантомний
Соматичний

У піддослідної тварини в експерименті видалено одну нирку, та накладено звужуючу лігатуру на артерію іншої нирки. Який вид вторинної гіпертензії виник у неї?
=Реноваскулярна
Ренопривна
Ендокринна
Нейрогенна
Ангіогенна

У хворого має місце недостатність мітрального клапану, внаслідок чого відбувається перевантаження серця кров’ю. Який механізм термінової компенсації є головним при перевантаженні серця об’ємом?
=Гетерометричний
Гомеометричний
Хроноінотропний
Інотропна дія катехоламінів
Гіпертрофія міокарда

В наслідок розвитку венозної гіперемії у тканинах виникає гіпоксія. До розладів яких біохімічних процесів це призведе?
=Порушення аєробного розщеплення вуглеводів
Зниження анаеробного розщеплення
Посилення катаболізму білків
Зниження утворення кетанових тіл
Зниження процесів гліколізу

Одним із проявів гострої променевої хвороби є гематологичний синдром. Які зміни у периферичній крові характерні для періоду первинних реакцій?
=Лейкоцитоз, лімфопенія
Лейкопенія, лімфопенія
Лейкопенія, тромбоцитопенія
Лейкоцитоз, тромбоцитопенія
Лейкопенія, анемія, тромбоцитопенія

У жіночу консультацію на контрольний огляд звернулась жінка на 6 місяці вагітності. При обстеженні виявлена залізодефіцитна анемія. Який механізм розвитку цього стану?
=Підвищене використання заліза
Порушення депонування заліза
Нестача заліза в їжі
Порушення всмоктування заліза
Дефіцит внутрішнього фактору Кастла.

У хворого на хронічний дифузний гломерулонефрит виявлена анемія. З чим пов’язаний її патогенез?
=Зниження продукції еритропоетину
Пригнічення функції червоного кісткового мозгу
Дефіцит внутрішнього фактору Кастла
Посиленим гемолізом еритроцитів
Наявністю антитіл до клітин периферичної крові

У хворого внаслідок травми поперекового відділу хребта відбувся перерив половини спинного мозку. При обстеженні на боці ураження має місце параліч кінцівки, втрата тактильної та глибокої суглобово-м’язевої чутливості, на протилежному боці - втрата больової і температурної чутливості. Для якого синдрому характерний такий симптомокомплекс?
=Броун-Секара
Верніке-Корсакова
Джуліана-Барра
Ламберта-Ітона
Паркінсона

Ухворого із стійкою артеріальною гіпертензією при проведенні ангіографії виявлено атеросклеротичне ураження обох ниркових артерій. Який механізм розвитку артеріальної гіпертензії є первинним?
=Збільшення продукції реніну
Збільшення катехоламінів
Збільшення виділення альдостерону
Збільшення серцевого викиду
Збільшення секреції вазопресину

У хворого із порушенням серцевого ритму при обстеженні на ЕКГ спостерігається: ЧСС 50/хв., ритм синусовий, не правильний; інтервал PQ подовжений; періодичне випадіння комплексу QRS. Яке порушення серцевого ритму має місце?
=Неповна а/v блокада ІІ ст
Повна а/v блокада
Блокада правої ніжки пучка Гіса
Неповна а/v блокада І ст
Синдром слабості синусового вузла

У хворого, що переніс 5 років тому резекцію шлунка, розвинулась В12-фолієводефіцитна анемія. Який механізм є провідним у розвитку такої анемії?
=Відсутність внутрішнього фактора Касла
Відсутність зовнішнього фактора Касла
Порушення всмоктування вітамінуВ12 в тонкій кишці
Дефіцит фолієвої кислоти
Дефіцит транскобаламіну

У жінки 55 років, що страждає подагрою, з*явилися сильні болі в області нирок. При ультразвуковому обстеженні встановлена наявність ниркових каменів. Підвищення концентрації якої речовини є найбільш ймовірною причиною утворення каменів у даному випадку?
=Уратів
Сечовини
Білірубіну
Холестерину
Цистіну

Юнаку 25 років, скаржиться на сухість у роті, спрагу, зниження маси тіла, незважаючи на підвищений апетит. При обстеженні: ріст 170 см, вага – 50 кг, рівень глюкози в крові - 10,5 ммоль/л, глюкозурія. Для якого з зазначених нижче станів найбільш характерні ці симптоми?
=Цукровий діабет
Нирковий діабет
Аліментарна глюкозурія
Стероїдний діабет
Нецукровий діабет

У пацієнта основний обмін знижений на 15\% негативний азотистий баланс. З боку відзначається гіпоглікемія, гіперліпацидемія, кетонемія, ацидоз. Дихальний коефіцієнт дорівнює 0,7. Про який період повного голодування може йти
=Період максимального пристосування

Період неощадливих трат
Термінальний період


У хворого протягом дня підвищення температури тіла змінюється зниженням її до нормального рівня. Таке підвищення температури спостерігається періодично через 3 дні на четвертий. Який тип температурної кривої?
=Febris internuttens
Febris continua
Febris reccurens
Febris hectica
Febris remitens

У пілота на висоті 14000 м виникла аварійна розгерметизація кабіни. Який з видів емболії в нього розвився?
=Газова
Тканинна
Тромбоемболія
Повітрянна
Жирова

На 8 день після введення протиправцевої сироватки з приводу брудної рани стопи в пацієнта підвищилася до 380С, з*явилися болі в суглобах, висипка, сверблячка. З боку крові спостерігається лейкопенія і тромбоцитопенія. Який тип алергічної реакції розвинувся?
=Імунокомплексна
Анафілактична
Цитотоксична
Гіперчутливість уповільненого типу
Стимулююча

Жінка 55 років звернулася зі скаргами на тривалі циклічні маткові кровотечі протягом року, слабкість, запаморочення. При огляді виявлена: блідість шкіри, аналіз крові: Hb – 70 г/л, еритроцити – 3,2 Т/л, колірний показник – 0,6, Le – 6,0 Г/л, ретикулоцити – 1\%. Гіпохромія еритроцитів. Яка анемія у
=Хронічна постгеморагічна
Гемолітична
Апластична
В12-фолієводефіцитна
Залізодефіцитна

Чоловік 56 років потрапив до клініки зі скаргами на загальну слабкість, біль і печіння в області язика, відчуття оніміння в кінцівках. У минулому переніс резекцію кардіального відділу шлунка. В аналізі крові: Hb – 80 г/л; еритроцити – 2,0 Т/л; колірний показник – 1,2 лейкоцити – 3,5Г/л; . Який вид анемії в даного хворого?
=В12-фолієводефіцитна
Гемолітична
Постгеморагічна
Апластична
Залізодефіцитна

Чоловік 40 років скаржиться на загальну слабкість, головний біль, кашель з виділенням мокротиння, задишку. Після клінічного огляду й обстеження поставлений діагноз: осередкова пневмонія. Який тип гіпоксії має місце в хворого?
=Дихальна (респіраторна)
Циркуляторна
Гемічна
Тканинна
Гіпоксична

Чоловік 50 років хворіє на хронічний бронхіт, скаржиться на задишку при фізичному навантаженні, постійний кашель з відходженям мокротиння. При обстеженні діагностовано ускладнення – емфізема легень. Чим вона обумовлена?
=Зниженням еластичних властивостей легень
Зменшенням альвеолярної вентиляції
Зменшенням розтягності легень
Зменшенням перфузії легень
Порушенням вентиляціоно-перфузійного співвідношення в легенях

Хворому 55 років поставлений основний діагноз гострий гломерулонефріт. Вкажіть основний механізм розвитку анемії при цому:
=Зменшення продукції ерітропоетіну
Зменшення клубочкової фільтрації
Зменшення синтезу ниркових простагландинів
Ниркова азотемія
Зменшення канальцевої реабсорбції

На прийом до лікаря прийшов пацієнт дуже високого росту, з довгими товстими пальцями рук, великою нижньою щелепою і відвислою нижньою губою. Підвищену секрецію якого гормону можна підозрювати?
=Соматотропного
Тироксину
Гонадотропного
Альдостерону
Катехоламінів

При клінічному обстеженні в жінки встановлено: підвищення потовідділення, тахікардія, схуднення, тремор. Яка ендокринна патологія може це спричинити?
=Гіпертиреоз
Гітотиреоз
Гіпергонадизм
Гіпогонадизм
Гіпоальдостеронізм

У жінки, віком 55 років, стоматолог виявив кілька зубів уражених карієсом. Як можно визначити наявність цього
=Патологічний процес.
Патологічний стан.
Патологична реакція
Хвороба.
Компенсаторна реакція

Який показник аналізу крові найбільш імовірно свідчить про наявність в організмі вогнища запалення?
=Зростання кількості нейтрофільних лейкоцитів
Зростання кількості ретикулоцитів
Зростання кількості тромбоцитів
Зростання кількості базофільних лейкоцитів
Зниження кількості моноцитів

Хворий після перенесеного епідемічного паротиту почав худнути, постійно відчував спрагу, пив багато води, відмічалось часте сечовиділення, підвищений апетит. Скаржиться на шкірний свербіж, слабкість, фурункульоз. В крові глюкози 16 ммоль/л, кетонові тіла 100 мкмоль/л; глюкозурія. Яке захворювання розвинулось у пацієнта?
=Інсулінозалежний цукровий діабет
Інсулінонезалежний цукровий діабет
Стероїдний діабет
Нецукровий діабет
Цукровий діабет недостатнього харчування

При голодуванні зменшується маса органів і тканин. Назвіть орган, який втрачає більше всього в масі в першому періоді
=Печінка
Нирки
Головний мозок
Серце
М’язи

Одним з механізмів ушкодження кардіоміоцитів при ішемії міокарда є активація перекисного окислення ліпідів. Цей процес стимулюється внаслідок підвищення вмісту в міокарді таких речовин:
=Катехоламінів
АТФ
Глютатіонпероксидази
Супероксиддисмутази
Вільних жирних кислот

При подагрі у хворих часто визначається збільшення та деформація суглобів внаслідок запалення. Який вид запалення знаходиться в основі цих змін?
=Проліферативне
Альтеративне
Ексудативне
Фібринозне
Змішане

Юнак 17 років захворів гостро, температура тіла підвищилась до 38,5 ? С, з’явився кашель, нежить, сльозотеча, виділення з носу. Яке запалення розвинулось у юнака?
=Катаральне
Серозне
Фібринозне
Гнійне
Геморагічне

Людину 30 років вкусила бджола. На місці укусу визначається набряк, гіперемія, підвищення температури. Назвіть ініціальний патогенетичний фактор запального набряку.
=Підвищення проникності мікросудин
Підвищення осмотичного тиску у вогнищі запалення
Зниження онкотичного тиску крові
Підвищення кров’яного тиску в капілярах
Порушення лімфовідтоку

У вогнищі гнильного запалення визначається багато путресцину, кадаверину, гістаміну та ін. Назвіть особливості обміну речовин, які приводять до утворення біогенних амінів у вогнищі
=Підсилення декарбоксилювання амінокислот
Активація дезамінування амінокислот
Стимуляція протеолізу
Порушення трансамінування
Пригнічення декарбоксилювання

Для пухлинної тканини характерна біохімічна анаплазія. Особливе місце в біохімії пухлини займає обмін вуглеводів і вироблення енергії. Назвіть особливості вуглеводного обміну в
=Негативний ефект Пастера
Позитивний ефект Пастера
Ослаблення гліколізу
Негативний ефект Кребтрі
Пригнічення гліколітичних ферментів

Хворий взимку впав в ополонку, замерз на вітрі, захворів. Температура тіла піднялась до 39,7 ? С і коливалась від 39,0 ? С до 39,8 ? С. Назвіть тип температурної кривої у хворого.
=Febris continua
Febris recurrens
Febris hectica
Febris intermittens
Febris remittens

При анемії в периферичній крові визначаються дегенеративні і регенеративні форми еритроцитів. Назвіть регенеративні форми еритроцитів.
=Ретикулоцити
Мікроцити
Сфероцити
Пойкілоцити
Гіперхромні еритроцити

Хвора 25 років, палестинка, скаржиться на слабкість, запаморочення, задишку. В анамнезі анемія, яка періодично загострюється. В крові: Hb – 60 г/л, еритроцитів – 2,5 Т/л, ретикулоцитів – 35 ‰, анізо- та пойкілоцитоз еритроцитів, багато мішеньоподібних еритроцитів, багато поліхроматофілів. Назвіть вид анемії у хворої.
=Таласемія
Серпоподібноклітинна анемія
Хвороба Мінковського-Шоффара
Хвороба Аддісон-Бірмера
Глюкозо-6-фосфатдегідрогеназдефіцитна анемія

У хворого з гіпохромною анемією в еритроцитах знаходиться 45 \% Hb S та 55 \% Hb А1. Яка форма анемії у хворого?
=Серпоподібноклітинна анемія
Альфа-таласемія
Хвороба Аддісон-Бірмера
Глюкозо-6-фосфатдегідрогеназдефіцитна анемія
Мікросфероцитарна анемія

Хворий 21 року скаржиться на слабкість, підвищення температури до 38-40 ? С. Об’єктивно: печінка і селезінка збільшені. В крові: Hb – 100 г/л, еритроцити – 2,9 Т/л, лейкоцити – 4,4 Г/л, тромбоцити – 48 Г/л, нейтрофіли с/я – 17 \%, лімфоцити – 15 \%, бластні клітини – 68 \%. Всі цитохімічні реакції негативні. Дайте гематологічне заключення.
=Недиференційований лейкоз
Хронічний мієлолейкоз
Гострий мієлобластний лейкоз
Гострий лімфобластний лейкоз
Гострий еритромієлоз

Хворий 62 років блідий, всі групи лімфовузлів збільшені. В крові – Hb – 60 г/л, еритроцити – 1,9 Т/л, лейкоцити – 29 Г/л, тромбоцити – 110 Г/л. Лейкоцитарна формула: сегментоядерні лейкоцити – 10 \%, лімфоцити – 8 \%, моноцити – 2 \%, бластних клітин – 80 \%. Цитохімічні дослідження бластних клітин: позитивна реакція на глікоген, негативна - на ліпіди і пероксидазу. Дайте заключення про патологію.
=Гострий лімфобластний лейкоз
Гострий мієлобластний лейкоз
Гострий мегакаріоцитарний лейкоз
Гострий промієлоцитарний лейкоз
Гострий монобластний лейкоз

В експерименті видалили частину головного мозку, внаслідок чого у тварини розвинулись асинергія і дисметрія. Який відділ головного мозку був видалений у тварини?
=Мозочок
Лобну долю
Тім’яну долю
Середній мозок
Сітчастий утвір

Хворому поставлено діагноз: ниркова артеріальна гіпертензія. Назвіть ініціальний патогенетичний фактор розвитку артеріальної гіпертензії в даному випадку?
=Ішемія нирок
Гіпернатріємія
Гіперальдостеронізм
Збільшення синтезу реніну
Збільшення синтезу ангіотензіну

У хворого на цукровий діабет розвинулась діабетична нефропатія з розвитком уремії. Швидкість клубочкової фільтрації 9 мл/хв. Який найбільш вірогідний механізм зниження швидкості клубочкової фільтрації і розвитку ХНН у пацієнта?
=Зменшення маси діючих нефронів
Зниження системного артеріального тиску
Закупорка просвіту канальців нефрона гіаліновими циліндрами
Розвиток ацидозу в тканинах
Спазм приносячих артеріол

В добовій сечі хворого визначені вилужені еритроцити. Для якої патології нирок найбільш характерний виявлений симптом?
=Дифузний гломерулонефрит
Нефротичний синдром
Нирково-кам’яна хвороба
Пієлонефрит
Гостра ниркова недостатність

При патології нирок в сечі з’являються патологічні складові частини. Поява яких патологічних складових частин сечі свідчить про підвищення проникності клубочкової мембрани?
=Протеїнурія
Глюкозурія
Аміноацидурія
Алкаптонурія
Піурія

У хворого на хронічний дифузний гломерулонефрит розвинулась хронічна недостатність нирок. В термінальній стадії ХНН розвивається оліго- та анурія, що спричиняється:
=Зниженням маси діючих нефронів
Ішемією коркової речовини нирок внаслідок спазма судин
Зменшенням фільтраційного тиску і фільтрації
Збільшенням реабсорбції води в дистальних канальцях
Дисемінованим внутрішньосудинним зсіданням крові

У жінки 32 років після перенесеного міокардиту виявлено при електрокардіографічному дослідженні порушення серцевого ритму (ритм не синусний). Функції яких кардіоміоцитів порушені?
=Пейсмекерних клітин
Скоротливих кардіоміоцитів
Перехідних провідних кардіоміоцитів
Провідних кардіоміоцитів пучка Гіса
Провідних кардіоміоцитів ніжок пучка Гіса

У хворого з хронічним мієлолейкозом знайдено ознаки анемії – зменшення кількості еритроцитів і вмісту гемоглобіну, оксифільні і поліхроматофільні нормоцити, мікроцити. Який патогенетичний механізм є провідним в розвитку цієї анемії?
=Заміщення еритроцитарного ростка
Внутрішньосудинний гемоліз еритроцитів
Дефіцит вітаміну В12
Зменшення синтезу еритропоетину
Хронічна кровотрата

При запальних процесах в організмі починається синтез білків “гострої фази”. Які речовини є стимуляторами їх синтезу?
=Інтерлейкін-1
Імуноглобуліни
Інтерферони
Біогенні аміни
Ангіотензини

У хворого з хронічним гломерулонефритом розвинулася нормохромна, нормоцитарна анемія. Механізм її пов’язаний з:
=Зниженням секреції еритропоетина
Підвищенням секреції еритропоетина
Зниженням секреції інгібітора еритропоетина
Стрептококовою інтоксикацією
Ушкоджувальною дією імунних комплексів

У хворого з пораненням кисті почав утворюватися набряк. У якій стадії порушення місцевого кровообігу це відбувається?
=Артеріальна гіперемія
Стаз
Венозна гіперемія
Спазм артеріол
Престаз

У водолаза, що тривалий час перебував на глибині 40 м, при декомпресії розвинулась кесонна хвороба. Основною патогенетичною ланкою стала емболія:
=Газова
Повітряна
Жирова
Парадоксальна
Тканинна

У хворого діагностована спадкова форма коагулопатії, яка проявляється дефектом VIII фактору зсідання крові. Вкажіть в якій фазі зсідання крові виникають первинні порушення коагудяції в даному випадку?
=Утворення тромбопластину
Утворення тромбіну
Утворення фібрину
Ретракція згустку


Хвора поступила в інфекційне відділення зі скаргами на нестримне блювання. Які порушення водно-сольового обміну у
=Гіпоосмолярна дегідратація
Ізоосмолярна дегідратація
Гіперосмолярна дегідратація
Гіпоосмолярна гіпергідратація
Гіперосмолярна гіпергідратація

У голодуючої тварини спостерігається загальне пригнічення, зниження маси тіла на 20\%. Температура тіла 36,2°С, кількість серцевих скорочень 68 /хв., дихальний коефіцієнт 0,7. Який період голодування?
=Максимального пристосування
Неекономного витрачання енергії
Тканинного розпаду



Після хімічного опіку у хворого розвинувся стеноз стравоходу. Наступило різке схуднення від затрудненого прийому їжі. Аналіз крові: 3,0(1012/л, Hb-106 г/л, загальний білок- 57 г/л. Який вид голодування у хворого?
=Неповне
Білкове
Повне
Водне
Абсолютне

У хворих з В12 - дефіцитною анемією виникають дегенеративні процеси в задніх і бокових стовпах спинного мозку (фунікулярний мієлоз). Ураження аксонів пов”язане з
=Мієліну
Ацетилхоліну
Дофаміну
Серотоніну
Норадреналіну

У жінки 40 років, яка систематично вживала ацетилсаліцилову кислоту, з’явилися крововиливи. Виявлено порушення функціональної активності тромбоцитів. Тромбоцитопатія в даному випадку зумовлена пригніченням активності
=Циклооксигенази
Холінестерази
Цитохромоксидази
Глюкозо-6-фосфатдегідрогенази
Na+, K+ - АТФази

Після перенесеного тяжкого інфекційного захворювання у пацієнта з’явилися ознаки нецукрового діабету, про що свідчило збільшення добового діурезу до 10 л. Який механізм є провідним у розвитку зневоднення ?
=Зменшення реабсорбції води в нирках
Підвищення осмолярності ультрафільтрату
Гальмування всмоктування води в кишечнику
Зниження реабсорбції натрію в нирках
Зниження онкотичного тиску плазми

У хворої 43 років після чергового загострення ревмокардиту з’явилися ознаки декомпенсації серцевої діяльності з виникненням набряків на ногах і асциту. Затримці води в організмі хворої сприяло підвищення продукції
=Альдостерону
Інсуліну
Кортизолу
Тироксину
Кортикотропіну

Під час операції на легенях у хворого виникла зупинка серця. Регулярні скорочення його вдалося відновити лише через 10 хв. Найглибші зміни внаслідок гіпоксії сталися в
=Корі головного мозку
Серці
Печінці
Нирках
Селезінці

У хворого з облітеруючим ендартеріїтом проведена гангліонарна симпатектомія. Який вид артеріальної гіперемії виник в результаті операції?
=Нейропаралітична
Нейротонічна
Метаболічна
Робоча
Реактивна

Жінка 42 років із невралгією трійчастого нерва скаржиться на періодичне почервоніння правої половини обличчя та шиї, відчуття припливу тепла та підвищення шкірної чутливості. Ці явища можна пояснити розвитком артеріальної гіперемії -
=Нейротонічної
Нейропаралітичної
Метаболічної
Робочої
Реактивної

Пацієнтка 58 років скаржиться на швидку втому, зниження працездатності, сонливість, задишку при швидкій ході. Аналіз крові: еритроцити – 4,0Т/л, гемоглобін – 92 г/л, колірний показник – 0,6. В мазку крові - велика кількість анулоцитів і мікроцитів. Для якої анемії це характерно?
=Залізодефіцитної
Постгеморагічної
Гемолітичної
Перніціозної
Серповидноклітинної

У хворого 23 років в результаті черепно-мозкової травми виник набряк мозку. Який механізм пошкодження клітин безпосередньо привів до набряку мозку?
=Електролітно-осмотичний.
Ліпідний.
Кальцієвий.
Ацидотичний.
Протеїновий.

У хворого діагностовано септичний ендокардит. Температура тіла протягом 5 днів коливалася в межах 39,5 – 40,2 0С. На 6 день на фоні різкого зниження температури до 35,2 0С розвинувся колапс. Який головний механізм колапсу?
=Вазодилятація
Гіпервентиляція
Посиленно потовиділення
Тахікардія
Поліурія

У хворого на гострий міокардит з’явилися клінічні ознаки кардіогенного шоку. Який із вказаних нижче патогенетичних механізмів є провідним в розвитку шоку?
=Порушення насосної функції серця
Депонування крові в органах
Зниження діастолічного притоку до серця
Зниження судинного тонусу
Збільшення периферичного опору судин

В умовах гострого експерименту у собаки видалили обидві нирки і перевели на гемодіаліз. Внаслідок цього виникло стійке підвищення артеріального тиску. Головним механізмом його виникнення є
=Зниження депресорних простагландинів
Збільшення продукції альдостерону
Збільшення антидіуретичного гормону
Активація ренін – ангіотензинової системи
Збільшення виділення норадреналіну

При дослідженні сечі у хворого з хронічним пієлонефритом виявлено зниження її відносної щільності до 1,010, що поєднувалося з підвищеним діурезом. Яка функція нирок порушена?
=Реабсорбція натрію і води
Клубочкова фільтрація
Канальцева секреція
Реабсорція глюкози
Реабсорбція білка

При обстеженні хворого на хронічний пієлонефрит без порушення азотовидільної функції нирок виявлено нормохромну анемію. Який механізм є провідним у розвитку анемії в даному
=Зменшення еритропоетину
Гематурія
Пригнічення еритропоезу
Дефіцит заліза
Дефіцит ціанокобаламіну

У хворого з гострою нирковою недостатністю на 6-й день проведення терапевтичних заходів виникла поліурія. Зростання діурезу на початку поліуричної стадії гострої ниркової недостатності зумовлене
=Відновленням фільтрації в нефронах
Збільшенням об’єму циркулюючої крові
Збільшенням натрійуретичного фактора
Зменшенням альдостерону в плазмі
Зменшенням вазопресину в плазмі

У жінки 30 років виникли набряки обличчя. При обстеженні виявлено протеїнурію (5,87 г/л), гіпопротеїнемію, диспротеїнемію, гіперліпідемію. Поєднання цих ознак характерне
=Нефротичного синдрому
Нефритичного синдрому
Хронічного пієлонефриту
Гострої ниркової недостатності
Хронічної ниркової недостатності

У пациента в результате длительной рвоты происходит значительная потеря желудочного сока, что является причиной нарушения кислотно-основного состояния в организме. Какая из перечисленных форм нарушения КОС имеет место ?
=Негазовый алкалоз
Газовый ацидоз
Негазовый ацидоз
Газовый алкалоз
Метаболический ацидоз

Група тварин була без їжі на протязі 10 годин, при цьому концентрація глюкози в крові залишалася на нижній межі норми. Чим це пояснюється?
=Активацією глікогенолізу
Активацією глікогенезу
Активацією глюконеогенезу
Пригніиченням глікогенезу
Пригниченням глюконеогенезу

У хворого з нефротичним синдромом спостерігаються масивні набряки обличчя та кінцівок, Який патогенетичнйм механізм є провідним в розвитку набряків?
=Зниження онкотичного тиску крові
Підвищення судинної проникності
Підвищення гідродинамічного тиску крові
Лімфостаз
Підвищення лімфовідтоку

Хвора поступила до клініки зі скаргами на загальну слабкість, запаморочення, задишку. Незадовго до звертання у клініку вона приймала лівоміцетин для профілактики кишкових інфекцій. Аналіз крові: еритроцити - 1,9 Т/л, гемоглобін - 58 г/л , колірний показник - 0,9 , лейкоцити - 2,2 Г/л. Про яку анемію
=Гіпопластичну
Метапластичну
Апластичну
Гемолітичну
Залізодефіцитну

У больного, находящегося в пульмонологическом отделении, диагностирована эмфизема легких, которая сопровождается снижением эластичности легочной ткани. Какой тип дыхания наблюдается при этом?
=Экспираторная одышка
Инспираторная одышка
Поверхностное дыхание
Редкое дыхание
Периодическое дыхание

Установлено, что при возникновении гепатомы в ней часто прекращается синтез желчных кислот. О каком виде анаплазии это свидетельствует?
=Функциональной
Энергетической
Морфологической
Биохимической
Физико-химической

У больного, который был госпитализирован после того, как находился под завалом в течении 4 часов, наблюдается резкое снижение диуреза, гиперазотемия, признаки отека головного мозга. В какой стадии острой почечной недостаточности находится больной?
=Олигоанурической
Начальной
Полиурической
Выздоровления
-

У популяції населення України частота гетерозигот за геном фенілкетонурії є високою і дорівнює 3\%. Який метод дослідження використовується в генетиці для раннього виявлення фенілкетонурії у новонароджених.
=Біохімічний
Популяційно-статистичний
Генеалогічний
Цитогенетичний
Дерматогліфики

Пацієнт 56 років, який хворіє на цукровий діабет, скаржиться на постійну сухість в роті і печію в ділянці губ. Яка основна причина вказаних проявів захворювання?
=дегідратація тканин внаслідок гіперглікемії.
метаболічний ацидоз.

порушення трофіки слизової рота.
розвиток в ротовій порожнині патогенної флори.

У хворого на інсулінзалежний цукровий діабет виявлені такі ознаки: гіперглікемія, гіперкетонемія, глюкозурія, ацетонурія та поліурія. До чого приводить накопичення кетонових тіл в крові?
=Негазового ацидозу і коми
Поліурії
Газового ацидозу
Ангіопатій
Олігурії

Жінка 38 років скаржиться на підвищену пітливість, серцебиття, підвищення температури у вечірні години. Основний обмін +60 \%. Лікар установив діагноз тиреотоксикоз. Які властивості тироксину приводять до підсилення теплопродукції?
=Роз’єднує окисне фосфорування
Підвищує спряження окислення і фосфорування
Зменшує бета-окислення жирових кислот
Зменшує дезамінування амінокислот
Сприяє накопиченню ацетил-КоА

Внаслідок необережності лаборанту в порожнину рота на язик попала хлористоводнева кислота. З якого процесу почнеться глосит у хворого?
=Альтерації
Порушення обміну речовин
Ексудації
Порушення мікроциркуляції
Артеріальної гіперемії

Пародонтит уражає у всьому світі до 50 \% населення віком після 30 років. Вирішальну роль в патогенезі пародонтиту грає:
=Нервово-дистрофічний фактор
Пошкодження калікреїном тканин пародонту
Пошкодження пародонту активними факторами лейкоцитів
Утворення зубного каменю мікрофлорою
Імунні пошкодження тканин

При аналізі ЕКГ виявлено: зубці Р відсутні, замість них чисельні хвилі f, комплекси QRST ідуть через різні проміжки часу (RR неоднакові), зубці R різного вольтажу. Назвіть вид аритмії.
=Миготлива аритмія
Синусова аритмія
Пароксизмальна тахікардія
Синусова тахікардія
Ідіовентрикулярний ритм

У хворого виявлено залізодефіцитну анемію. Наслідком тривалого перебігу цієї патології можуть бути атрофічні та дистрофічні процеси в травному тракті: глосит, гінгівіт, карієс зубів, езофагіт. Чим спричинені такі змніи при даній анемії?
=Зниженням активності залізовмісних ферментів
Підвищеннім активності трансамінування
Підвищенням активності мідьоксидази
Зменшенням активності гліколізу
Підвищенням активності каталази

Хворий Н., 55 років, хворіє на хронічний гломерулонефрит протягом 15 років. Які зміни складу крові або сечі найбільш характерно свідчать про обмеження секреторноі функції нирок?
=Гіперазотемія
Гіперглікемія
Гіпопротеїнемія
Протеїнурія
Гіпо-, ізостенурія

У хворого діагностована кістково-мозкова форма гострої променевої хвороби. В крові виявлено лейкоцитоз та лімфопенія. Для якої стадії хвороби характерні ці зміни?
=Первинних реакцій
Прихованої
Розпалу хвороби
Завершення хвороби
Віддалених наслідків

У хворого 65 років періодичні больові відчуття під лівою лопаткою і в лівому плечі різко посилилися після значного фізичного навантаження. Методом ЕКГ діагностовано гострий інфаркт міокарда. Який вид болю мав місце в даному випадку?
=Іррадіюючий
Вісцеральний
Фантомний
Каузалгія


У 10-и річного хлопчика, хворого на пневмонію, яка супроводжувалась високою температурою, застосування антибактеріальної терапії спричинило критичне падіння температури. Які зміни терморегуляції характерні для цієї стадії гарячки?
=Збільшення тепловіддачі і зниження теплопродукції
Збільшення теплопродукції і збільшення тепловіддачі
Збільшення тепловіддачі при незмінній теплопродукції
Тепловіддача дорівнює теплопродукції
Збільшення теплопродукції і зниження тепловіддачі

До приймального відділення лікарні потрапив юнак в стані несвідомості, внаслідок отруєння морфіном. У нього спостерігається поверхневе та рідке дихання, яке обумовлене пригніченням дихального центру. Який тип недостатності дихання виник при цьому?
=Вентиляційна дисрегуляторна
Вентиляційна обструктивна
Вентиляційна рестриктивна
Перфузійна
Дифузійна

У хворого з серцевою недостатністю виникла аритмія, при якій на ЕКГ частота скорочень передсердь була 70, а шлуночків – 35 скорочень за 1 хв. Порушення якої функції серцевого м’язу спостерігається у хворого?
=Провідності
Збудливості
Автоматизму
Збудливості та провідності
Скоротливості

Після ампутації кінцівки хворий тривалий час відчуває кінцівку та сильний, нестерпний біль у ній. Як називається такий біль?
=Фантомний
Рефлекторний (відбитий)
Вісцеральний
Перший (протопатичний )
Другий (епікритичний)

У хворого, який скаржився на біль у ділянці лівої лопатки, був діагностований інфаркт міокарду. Назвіть вид болю у хворого?
=Ірадіюючий (відбитий)
Вісцеральний
Фантомний
Перший (протопатичний)
Другий (епікритичний)

До лікаря звернувся пацієнт з приводу пожовтіння склер і шкіри. При обстеженні не було виявлено енцефалопатії та ознак холемічного і ахолічного синдромів. Яка жовтяниця розвинулась у цього пацієнта?
=Гемолітична
Паренхіматозна
Ядерна
Механічна
Ензимопатична

У туристів, що піднялись на висоту 2,5 км, розвинулась гіпоксична гіпоксія у вигляді гірської хвороби. Який провідний початковий механізм лежить у розвитку даного виду гіпоксії?
=Зниження рO2 у вдихуваному повітрі
Зниження рCO2 в повітрі
Анемія
Відносна недостатність кровообігу
Інтенсивна м’язова робота

Чоловік тривалий час працює в нафтопереробній промисловості. Який з наведених класів канцерогенів зустрічається в його оточенні?
=Поліциклічні ароматичні вуглеводні
Аміноазосполуки
Нітрозаміни
Канцерогени біологічного походження
Аміни

Хвора 38 років надійшла в реанімаційне відділення в несвідомому стані. Рефлекси відсутні. Цукор крові – 2,1 ммоль/л. В анамнезі – цукровий діабет з 18 років. Яка кома має
=Гіпоглікемічна
Кетоацидотична
Лактацидемічна
Гіперосмолярна
Гіперглікемічна

Жінка 26 років скаржиться на сухість у роті, спрагу. При обстеженні: рівень глюкози в крові – 6,5 ммоль/л, глюкозурія. Для якого з зазначених нижче станів найбільш характерні ці
=Нирковий діабет
Стероїдний діабет
Нецукровий діабет
Аліментарна глюкозурія
Цукровий діабет

Ріст дитини 10 років досягає 178 см, маса – 64 кг. З порушенням діяльності якої ендокринної залози це зв*язано?
=Гіпофізу
Щитовидної залози
Статевих залоз
Надниркових залоз
Паращитовидної залози

До лікаря звернулася жінка зі скаргами на те, що у весняний період у неї з*являється нежить, сиплість голосу, почервоніння вік зі сльозотечею. Який тип алергійної реакції по Джелу і Кумбсу розвивається в пацієнтки?
=Анафилактичний
Уповільненої гіперчутливості
Цитотоксичний
Імунокомплексний
Стімулюючий

У приймально-діагностичне відділення доставили жінку 38-ми років з шлугковою кровотечею, які зміни найбільш можливі з
=Зменшення гематокритного числа
Лейкоцитоз
Еритроцитоз
Лейкопенія
Збільшення гематокритного числа

Пацієнт попередив хірурга-стоматолога, що застосовування знеболюючих препаратів може викликати алергічний шок. Збільшення кількості в крові якого біогенного аміну може бути причиною такого стану?
=Гістаміна;
ГАМК;
Серотоніна;
Дофаміна;
Триптаміна.

У хворого кислотність шлункового соку складає 20 умовних одиниць. Що може спричинити таке зниження кислотності шлункового соку?
=Хронічний атрофічний гастрит;
Хронічний гіпертрофічний гастрит;
Хронічний коліт;
Виразкова хвороба дванадцятипалої кишки;
Виразкова хвороба шлунка.

У хворого струс головного мозку, що супроводжується повторним блюванням і задишкою. При обстеженні відзначено: рН = 7,62; РСО2 = 40 мм рт. ст. Яке порушення кислотно-основного стану хворого Б.?
=негазовий алкалоз
газовий алкалоз
негазовий ацидоз
газовий ацидоз


У хворого відзначається атрофія альвеолярних відростків щелепи після видалення зубів. Це є прикладом:
=патологічного стану
патологічної реакції
патологічного процесу
структурного сліду адаптації
хвороби

http://www.uatest.org.ua/module/krok/?do=base&id=52


Рецензии